SlideShare una empresa de Scribd logo
1 de 21
01 02COLEGIO DE CIENCIAS LORD KELVIN 4to Año Secundaria ALGEBRA 4to Año Secundaria
OBJETIVOS ESPECIFICOS:
Define y reconoce a las cantidades imaginarias
como componentes no reales de los números
complejos.
Opera con las potencias enteras de i.
COMENTARIO PREVIO:
El conjunto І de los números irracionales, junto
con el conjunto Q de los números racionales
constituyen el conjunto R de los números reales.
Por los conocimientos previos que manejamos
vimos que el campo numérico hasta ahora
conocido necesitaba una nueva ampliación que
permitiera hallar raíces pares de números
negativos. Así, por ejemplo, no existe ningún
número real que represente
4
4,1,8,3 −−−− . Estas raíces
reciben el nombre cantidades imaginarias.
Llamamos imaginarios a los números
constituyentes de las componentes no reales de
los números complejos.
En este módulo que consta de dos sesiones
trataremos de realizar un estudio formal y
riguroso de este nuevo sistema numérico, en todo
momento relacionaremos estos conceptos nuevos
con los conocimientos antes conocidos.
CONTENIDO TEÓRICO:
CANTIDADES IMAGINARIAS
Los números imaginarios se originan de la
extracción de la raíz cuadrada a números
negativos.
Definición: Las cantidades imaginarias son
aquellas que se obtienen por la extracción de
raíces pares de números negativos.
Ejemplo: 126
5;64;9 −−− ; son
cantidades imaginarias. Toda expresión de la
forma: n
a2 , donde 2n es par y a es un número
real negativo, es una cantidad imaginaria pura.
Unidad imaginaria:
Recibe este nombre el radical 1− , se le
representa mediante el siguiente símbolo: i =
1− (notación de Gauss) y cumple i 2
= -1.
1− se tomará como referencia para medir
todas las cantidades imaginarias puras.
Operación básica de transformación:
Sea a ∈ ℜ+
, tenemos:
iaaaa =−=−=− 1)1(
Toda raíz imaginaria puede expresarse como el
producto de un número real por la unidad
imaginaria.
Ejemplo:
i39 =− ; i55 =− ;
i
3
2
9
4
=−
Potencias enteras de la unidad imaginaria:
De i1
= i se ha concluido que i2
= -1; conociendo
esto podemos deducir todas las demás potencias de
i.
i 1
= i i 5
= i 4
. i 1
= i i 9
= i
i 2
= -1 i 6
= i 4
. i 2
= -1 i 10
= -1
i 3
= - i i 7
= i 4
. i 3
= - i i 11
= - i
i 4
= 1 i 8
= i 4
. i 4
= 1 i 12
= 1
Generalizando:
i 4q + r
= i r
0 ≤ r < 4 ; q ∈ Ζ
Veamos los siguientes ejemplos:
♦ Sabemos que: i 4q
= 1 ∀ q ∈ Ζ
i 12 448
= 1 ; i 137 956
= 1; i -12 448
= 1
Recuerde que 4q es
0
4 ; ∀ q ∈ Ζ
♦ La unidad imaginaria i, no siempre
estará afectado exactamente de un exponente
0
4 , podría presentarse:
i 4q + r
; 0 ≤ r < 4; q ∈ Ζ
i 15 767
= i 4 (3 941) + 3
= i 3
= - i
i -135
= i 4 (-34) + 1
= i 1
= i
Se concluye: i + i 2
+ i 3
+ i 4
= 0, esta
relación podemos generalizarlo diciendo: la
suma de cuatro potencias consecutivas
cualesquiera de la unidad imaginaria es igual a
cero. Es decir:
i n
+ i n+1
+ i n+2
+ i n+3
= 0 ; ∀ n ∈ Ζ
Ejemplos
♦ i 217
+ i 218
+ i 219
+ i 220
= 0
♦ i - 75
+ i - 76
+ i - 77
+ i - 78
= 0
Resumen:

0
4
41
0
∀=i positivo o negativo

0
4
4
0
∀=+ rr
ii positivo o negativo, r ∈
Ζ
 0432
=+++ iiii ; en general:
 Ζ∈∀=+++ +++
niiii nnnn
;0321
PRÁCTICA DE CLASE
1. Hallar: a) i 4 273
b) i 30 214
2. Siendo, i = 1− , dar el valor de:
I. i –7
II. i –21
III. i-3 224
3. Simplificar:
37912137728249
12123 iiiii +−++
4. Hallar el valor de: i 658
+ i 527
– i 436
+ i 247
5. Simplificar:
161514131211109
8765432
428428
824824
iiiiiiii
iiiiiiii
+++++++
+++++++
6. Reduce a su mínima expresión:
(i –233
– i –232
+ i –231
-... – i –2
+ i –1
– 1) 2
7. Simplificar:
niniiii
iniiii
n
n
4)18(...9753
8...432
4432
168642
−++++++
+++++
EJERCICIOS PROPUESTOS Nº 01
1. Hállese la parte real de efectuar:
2222
100321
..... iiiiK ++++=
a) 1 b) 0 c) 50
d) 100 e) 25
2. Efectuar: E =
3333
5555
2222
iii ++
a) i b) - i c) 1
d) - 1 e) N.A
3. Simplificar:
603593584540527
293302313321328
−
+
−
+
−
−
−
+
−
++++
=
iiiii
iiiii
K
a) 4 b) 3 c) 8
d) - 1 e) N.A
S4AL33B “El nuevo símbolo de una buena educación....” S4AL33B “El nuevo símbolo de una buena educación...."
III
CANTIDADES
01 02COLEGIO DE CIENCIAS LORD KELVIN 4to Año Secundaria ALGEBRA 4to Año Secundaria
4. El valor simplificado de:














21
15
!776
!777 i
i
,es:
a) 0 b) 1 c) -1
d) i e) -i
5. Efectuar: 1 + i + i3
+ i5
+ i7
+…. + i79
a) 1 b) i c) 2i
d) 32i e) i + 1
06.Simplificar:
32824255
800876353875175347
iiii
−−−
++
+++
iii
a) -2 b) 2 c) -2i
d) 32i e) – i
07.Hallar el valor de:
E = i2
+ 2i4
+ 3i6
+ 4i8
+… + (2n – 1)i4n – 2
+
2n.i4n
a) 1 b) ni c) 2ni
d) 32i e) n
08.Siendo: 1−=i .
Simplificar:
1
...
325412
775432
−−+−
+++++
=
iiii
iiiii
G
a) 1 b) 2 c) i
d) 2i e) 3i
09.Reducir: 969798 KELVINWTIUNT
iii ++
Siendo 1−=i
a) 0 b) 1 c) i – 1
d) i e) – i
10.Calcular: S = i + i2
+ i3
+...... + i100
.Donde: i2
= -1
a) 0 b) 1 c) 2
d) 3 e) 5
TAREA DOMICILIARIA
01.Simplificar:
196225361
36100324
−+−−−
−−−+−
02.Calcular:
a) i16
b) i10
c) i4n+3
(n entero) d) i43
03.Calcular:
367
5
i
04.Hallar:
3333
3
i
05.Calcular:
839
6
i
06.Hallar el valor de: E = i1!
+ i3!
+ i5!
+ i7!
+ … +
i59!
OBJETIVOS ESPECIFICOS:
 Realiza un estudio formal de los números
complejos y sus respectivas propiedades.
 Aplica las propiedades antes estudiadas en la
resolución de ejercicios que involucran
números complejos.
COMENTARIO PREVIO:
El matemático francés Descartes fue el primero
que llamó imaginarios a los números
constituyentes de las componentes no reales de
los números complejos. El matemático alemán
Euler contribuyó notablemente a divulgar el uso
de los números complejos, pero quién mayor
auge dio a su utilización fue el matemático danés
Wessel, que suministró una valiosa interpretación
geométrica de los números complejos.
CONTENIDO TEÓRICO:
01.NÚMEROS COMPLEJOS
Son de la forma Z = a + bi, a ∈ IR, b ∈ IR
se llama:
a = Re(z) → parte real de Z
b = Im(z) → parte imaginaria de Z
Complejos conjugados
Son aquellos que difieren únicamente en el
signo de su parte imaginaria:
4 – 3i 4 + 3i
Complejos Opuestos
El opuesto de a + bi es –a - bi
Complejos Nulos
Aquel número que tiene su parte real y su
parte imaginaria iguales a cero:
0 = 0 + 0i
Igualdad de dos números complejos
Dos números complejos son iguales si tienen
iguales sus partes reales y sus partes
imaginarias:
a + bi = c + di ⇒ a = c y b = d
Módulo o norma de un complejo
Se define por medio de la siguiente
relación:
Ejemplo:
5)4()3(43 22
=−+−=−−= ir
02.OPERACIONES CON NÚMEROS
COMPLEJOS
AdiciónAdición:
(a + bi) + (c + di) = (a + c) + (b + d)i
(2 + 7i) + (-7 - 3i) = (2 - 7) + (7 - 3)i = -5 + 4i
SustracciónSustracción
(a + bi) - (c + di) = (a - c) + (b – d)i
iii
1
1
9
1
5
2
)43(
9
1
4
5
2
3 +−=





−+−=





+−





+
MultiplicaciónMultiplicación
(a + bi) (c + di) = (a c-bd) + (ad +bc)i
(2-5i)(3-7i) = 6 - 14i - 15i + 35i2 = -29 - 29i
PotenciaciónPotenciación
Zn
= Z.Z.Z.Z.Z.... Z , ∀ Z∈C, n∈N
n veces
División de números complejosDivisión de números complejos
S4AL33B “El nuevo símbolo de una buena educación....” S4AL33B “El nuevo símbolo de una buena educación...."
22
baibaP +=+=
C
⇔
C
NÚMEROS
01 02
Observación: Cuando el índice es 2 (n
= 2) podría tomarse en cuenta la
transformación de radicales dobles en
simples, paraalgunoscasos.
COLEGIO DE CIENCIAS LORD KELVIN 4to Año Secundaria ALGEBRA 4to Año Secundaria
22
)()(
))((
))((
dc
iadbcbdac
idcidc
idciba
idc
iba
+
−++
=
−+
−+
=
+
+
Efectuar:
i
ii
ii
ii
i
i
25
26
25
7
34
15268
)34)(34(
)34)(52(
34
52
22
2
−
−
=
+
+−
=
−+
−−
=
+
−
Raíz cuadrada de números complejosRaíz cuadrada de números complejos
La radicación de un número complejo
arrojará tantas raíces como lo indique el
índice del signo radical.
Es decir: dado Z = a + bi para calcular las
raíces enésimas o raíces de orden “n” de Z
(n∈ Ν, n ≥ 2), se establece lo siguiente:
yixbian
+=+
Donde: a, b y n son datos, x e y tendrán que
calcularse (x, y ∈ ℜ); para esto se tendrá que
elevar ambos miembros a la “n” y desarrollar
el segundo miembro por fórmula del
binomio de Newton. Se recomienda esto
cuando “n” toma valores pequeños, en caso
contrario téngase en cuenta la fórmula de
Moivre.
Ejemplo: Calcular las raíces cuadradas de 21
– 20i
Resolución
Establecemos la igualdad:
iyxi +=−2021
Elevando al cuadrado:
21 - 20i = (x 2
– y 2
) + 2xy i
Por igualdad de números complejos:
♦ x 2
– y 2
= 21 ....(I)
♦ 2xy = -20 ....(II)
(I) 2
+ (II) 2
: (x2
– y2
) 2
+ 4x2
y2
= (21)2
+ (-20)2
(x2
+ y2
)2
= 841→ x2
+ y2
= 29 .......(III)
(I)+(III): 2x2
= 50 → x2
= 25 → x = 5 ó x = –5
(III)–(I): 2y2
= 8→ y2
= 4→ y = 2 ó y = -2
De (II): x e y tienen signos opuestos, luego:
x = 5 ; y = -2 ó x = -5 ; y = 2
)25(2021 ii −±=−∴
Otra forma: Aplicando transformaciones de
radicales dobles en simples. Veamos:
1002211102212021 −−=−−=− xi
( ) ( )ii 254252021 −±=−−±=−
03.REPRESENTACIÓN GRÁFICA DE UN
NÚMERO COMPLEJO
La representación gráfica de un número
complejo se realiza en un sistema de ejes
coordenadas denominado Diagrama de
Argand, mediante un punto cuyas
coordenadas serán las componentes de un
complejo, al punto se le denomina afijo del
número complejo.
Ejemplo:
Representar gráficamente los siguientes
números:
Número complejo Afijo del complejo
4 + 3i (4 ; 3)
2 - 4i (2 ;-4)
-5 + 2i (-5; 2)
7 (7 ; 0)
-3i (0 ;-3)
eje imaginario
eje real
(4,3)
(-5,2)
(2,-4)
04.FORMA POLAR O TRIGONOMÉTRICA
RELACIONES FUNDAMENTALES
e je im a g in a rio
e je re a l
a + b i = (a , b )
b
p
a
0
1) 22
bar += módulo
2) θ = arc tg (
a
b
) argumento
(0 ≤ θ < 2π), a ≠ 0
3) a = r cos θ
4) b = r sen θ
FORMA POLAR O TRIGONOMÉTRICA DE UN
COMPLEJO
Sea Cos θ + i sen θ = Cis θ
Luego: a + b i = r Cis θ
Ejemplo:
1) Cis 60° = cos 60° + i sen 60° =
2
1
+
2
3
i
2) Expresar en forma polar – 4 – 4 3 i
8)34()4( 22
=−+−=r
°==
−
−
= 2403
4
34
tgarctgarcθ
°=−− 2408344 cisi
e je im a g in a rio
e je re a l
2 4 0 °
6 0 °
05.OPERACIONES CON COMPLEJOS EN
FORMA POLAR
1°)Multiplicación
S4AL33B “El nuevo símbolo de una buena educación....” S4AL33B “El nuevo símbolo de una buena educación...."
a + b i = r (cos θ + i sen θ)
(25) + (-4)
(25) (-4)
01 02COLEGIO DE CIENCIAS LORD KELVIN 4to Año Secundaria ALGEBRA 4to Año Secundaria
(r1 Cis θ1) (r2 Cis θ2) = r1 r2 Cis (θ1 +
θ2)
2°)División
3°)Potenciación
(r cis θ)n
= rn
cis (nθ)
Nota:
Si r = 1 tenemos la fórmula de Moivre
(cis θ)n
= cis (nθ)
4°)Radicación
Donde: k = 0 ; 1 ; 2 ; ... (n-1)
Ejemplos:
1) Hallar las raíces cúbicas de la unidad
)120(
3
360
1011 333
kCis
k
Ciscis °=
°
==
K = 0 ⇒ W1=Cis 0 = 1
K = 1 ⇒ W2=Cis 120° =
i
2
3
2
1
+
−
K = 2 ⇒ W3=Cis 240° =
2
1−
-
2
3
i
2) Resolver x4
+ 1 = 0
4
360180
118011
k
Ciscisx
°+°
==−=
k = 0, 1, 2, 3
K = 0 ⇒ W1=Cis 45° =
2
2
+
2
2
i
K = 1 ⇒ W2=Cis 135° =
2
2−
+
2
2
i
K = 2 ⇒ W3=Cis 225° =
2
2−
-
2
2
i
K = 3 ⇒ W4 = Cis 315° =
2
2
-
2
2
i
PRÁCTICA DE CLASE
01. Efectuar:
i
i
i
i
i
i
K
−
+
−
+
−
+
−
+
−
+
=
1
1
1
1
1
1
1
1
1
1
a) 1 - i b) 1 c) 0
d) 1 + i e) i
02. Si la raíz cuadrada del número complejo: 1 +
i es: x + yi .Hállese el valor de:
x
y
y
x
M −=
a) 0 b) 1 c) 2
d) 3 e) 4
03. Si se cumple: iyixn +=++ 148
Calcular el valor de “x”.
a) 24n b) 24n+2 c) 42n+1 d)
-24n+2 e) -22n+1
04. Calcular:
( )∑=
+
+++
n
n
nn
xxx
1
4644
2652
Para: x = −1
a) 1 b) 5 c) 5(n - 1)
d) 5n e) 5(n+1)
05. Simplificar:
ii
i
2
13
2
3
125
37






+−
a) 1 b) 2 c) 3
d) 4 e) N.A.
06. Si:
2
31 −+−
=w . Calcular el Valor
numérico de: K = (5 + 7w + 7w2
)12
a) 64 b) 512 c) 1024
d) 2048 e) 2048
07. Siendo “W” una de las raíces cúbicas de la
unidad tal que: w ≠ 1, calcule:
S = (a+aw+w2
)4
(1+aw+aw2
)4
(a+w+ aw2
)4
a) a12 b) (a + 1)12 c) -(a - 1)12
d) (a -1)12 e) N.A.
08.Si tenemos que:
52
5
9
1
2
21
3
i
i
i
xii
=
−
+
+
+
+
Hállese el módulo de “Z”:
+
∈+= RxixZ /
a) 1 b) 2 c) 3
d) 4 e) N.A.
09. Hállese “Z” de:
1
8
4
;
3
5
8
12
=
−
−
=
−
−
Z
Z
iZ
Z
a) 6 + 17i b) 4 + 9i c) 6 + 19i
d) 6 + 8i e) a y d
10. Si “z” es un complejo tal que 5=z
.Halle:
22
11 zzK −++=
a) 52 b) 50 c) 48
d) 2 e) 32
11. La suma de los siguientes complejos:
Z1 = 2 + (y + 2)i
Z2 = y + 4 - 3yi / y ∈ R
Origina un número real, calcularlo
a) 7 b) 0 c) 5
d) 2 e) 6
EJERCICIOS PROPUESTOS Nº 02
01.Efectuar:
i
i
ii
Z
−
+
+
−
+
+
=
1
1
1
1
1
1
a) i b) -i c) 1 + i
d) 1 – i e) 0
02.Calcular:
( )
9
9
1
1
i
i
E
+
+
=
a) 8 b) 12 c) 16
d) 4 e) 2
S4AL33B “El nuevo símbolo de una buena educación....” S4AL33B “El nuevo símbolo de una buena educación...."
)( 21
2
1
22
11
θθ
θ
θ
−= Cis
r
r
Cisr
Cisr
n
k
Cisrcisr
)360( °+
=
θ
θ
01 02COLEGIO DE CIENCIAS LORD KELVIN 4to Año Secundaria ALGEBRA 4to Año Secundaria
03.Calcular el valor de:
( ) ( )
( )6
23
1
11
i
ii
E
−
+−+
=
a) i b) 3i c) 4i
d) i/4 e) 2i
04.Efectuar:
( ) ( )
3
311 2
−
++
i
ii
Donde: 1−=i , es igual a:
a) 1 – 3i b) -2 c) 10
d) 2 e) 101
05.Calcular el menor valor de “n” que verifica:
(1 + i)n
= 32 i
a) 5 b) 4 c) 8
d) 10 e) 12
06.Simplificar:
27233581246032442541
33746542152
iiiii
iiiii
E
−+−+
++++
=
a) i b) -i c) 1
d) -1 e) 3
07.Si se cumple:
(1 + i)2
+ (1 + i)4
+ (1 + i)6
+ (1 + i)8
= m + ni
m, n ∈ IR. Hallar: m . n
a) 70 b) 72 c) -72
d) 40 e) -54
08.Hallar “n” en: [(1 + i)7
+ (1 – i)7
]n
= 26!
a) 120 b) 180 c) 240
d) 300 e) 360
09.Simplificar:
42
432
1
1
1
1
1
1
1
1
ii
iiii
i
i
i
i
E
+−
+++
+
−
+
−
+
−
+
+=
a) 1 – i b) 1 + i c) i
d) 2 i e) 0
10.Reducir:
Z = (1+ i)3
+ (1+ i2
)3
+ (1+ i3
)3
+…+ (1+
i200
)3
a) 100 b) 200 c) 300
d) 400 e) 500
11.Sea:
Z = x + (y – 2) i ; 1−=i
W = 5 + (x + 5) i
Calcular x + y para que Z y W sean
conjugados.
a) 13 b) –8 c) –3
d) 1 e) 5
12. Calcular:
1;5
201424
1
−=
−
+
i
i
ii
a) –i b) i c) 2
d) 1 e) 0
TAREA DOMICILIARIA
01.Calcular: R =
4
1
1
1
1




+
−
+
−
+
i
i
i
i
a) 2 b) i c)4
d) 0 e) N.A
02.Si Z = 1 + i ; Calcular: Z8
a) 2 i b) 4 i c) 16
d) 18 e) N.A
03.Si la raíz cuadrada del número complejo 1 + i
es x + y i , Hallar el valor de : M = x/y - y/x
a) 0 b) 1 c) 2
d) 3 e) N.A
04.Sabiendo que E es un valor real, donde el
valor de:
E =
( ) i
i
i
−
+
+
3
43
225
; donde i =
1−
a) 4 b) 2 c) 3
d) 1 e) N.A
05.Sumar:
+
−
+
+
−
+
+
−
+
i
i
i
i
i
i
56
65
34
43
2
21
……………n términos
a) ( n + 1 ) i b) ( 2n + 1 ) I c) n i
d) 2n I e) N.A.
06.Hallar “α - β” en :
( 1 + i ) ( 2 + i ) ( α + i ) = (1 - i ) (2 - i ) (β - i
)
Sabiendo su 1−=i
a) 2 b) 4 c) 0
d) -1 e) N.A
07.Hallar “a + b” si
( )
ibai
i
i
i
i
+=+
−
+
+
−
−
4
21
21
5
3
2
23
a) 2 b) 4 c) 6
d) 8 e) N.A
08.Hallar “a + b” si:
( )
ibai
i
i
i
i
+=+
−
+
+
−
−
4
21
21
5
3
2
23
a) 2 b) 4 c) 6
d) 8 e) N. A
09.Si x; y ∈ IR indicar el valor de y
x
sabiendo que:
x – 2y + xi – yi = 2 + 5i
a)2 b)1 c) 8
d)1/2 e)1/4
10.Si:
( )
1;
93
2
9
1
1
3
33
11
2
−=
+





++−
++





−+
= i
a
i
ai
i
a
i
Z
Calcular: Z4
+ 1
a) 82 b) 81 c) a + 800
d) 80i + 16 e) 81 + a
11.Si: 1−=i indicar el valor de:
32
1003432
2
...
iii
iiiii
E
−+−
+++++
=
a) -1 b) 1 c) 1/2
d) -1/2 e) (1/2)i
12.Halle el módulo del siguiente complejo:
( )( )
( )( )2
4
173
2231
ii
ii
E
−+
++
=
a) 16 b) 4 c) 32
d) 8 e) 64
13.Si: 7=z ; calcular el valor de:
22
11 zzR +++=
a) 15 b) 16 c) 17
d) 18 e) 19
S4AL33B “El nuevo símbolo de una buena educación....” S4AL33B “El nuevo símbolo de una buena educación...."
01 02COLEGIO DE CIENCIAS LORD KELVIN 4to Año Secundaria ALGEBRA 4to Año Secundaria
14.Sumar:
ostérn
i
i
i
i
i
i
min""...
56
65
34
43
2
21
++
−
+
+
−
+
+
−
+
a) i b) -i c) ni
d) -ni e) 0
15.Luego de efectuar:
5
2 iii +− Obtenemos
a) 1 + i b) -i c) i
d) 1 e) 1 – i
16.Si: x, y ∈ IR indique la relación entre ellos
para que: (x + yi) (2 + 3i) sea un número real.
a) x = 2y b) 2y + 3x = 0 c)x = 3y
d) x – y = 1 e) x + y = 2
17.Indicar el cuadrado de:
14091409 −−+−+
a) 80 b) 100 c) 36
d) 625 e) 900
18.Si: {a; b; c; d} ∈ IR / a ≠ b ≠ 0 además:
aib
bia
didiccZ
+
+
=++= ; indicar: z 
a) 1 b) 2 c) 3
d) 4 e) 5
19.Halle el módulo del complejo:
3
4
22
)66cos66)(23(
i
iseni
z +
°+°+
= ;
1−=i
a) 2
13
b) 2
13
c) 4
13
d) 13 e) 8 13
20.Hallar “x + y” en:
1i;y3x
i4x3
iy1
ix
−== +
+
+
a) 3 b) 5/2 c) 3/2
d) 9/2 e) 7/2
OBJETIVOS ESPECIFICOSOBJETIVOS ESPECIFICOS:
 Reconoce y clasifica una ecuación
algebraica desarrollando la percepción y
acumulando experiencias que servirán de
soporte para futuras formalizaciones
 Dado un conjunto de Ecuaciones de
Primer Grado, trabaja creativamente y con
actitud crítica situaciones problemáticas,
utilizando una variedad de técnicas de cálculo
y aplicando correctamente las propiedades
que correspondan.
COMENTARIO PREVIOCOMENTARIO PREVIO:
Hace cinco mil años, en el país de los
sumerios, cerca del Golfo Pérsico, se dieron las
primeras dificultades matemáticas que
necesitaban ser interpretadas bajo ciertas
igualdades. Esto dio inicio a las primeras
relaciones que, posteriormente, los matemáticos
dieron el nombre de Teoría de Ecuaciones.
Con el afán de resolver las ecuaciones se han
creado nuevas teorías, nuevos conceptos, nuevos
conjuntos numéricos. El método de resolución de
las ecuaciones de primer y segundo grado fueron
descubiertos por los matemáticos sumerios y
babilonios (3000 años a.C) y por Diofante (329 –
410 d.C) fundador del Álgebra, por los hindúes
y, finalmente por los árabes (siglo IX). Este
método forma parte del más antiguo patrimonio
matemático de la humanidad. La ecuación de
tercer grado dio ocasión a Cardano (1501–1576)
y a Tartaglia (1499– 1557) para inventar los
números complejos en el siglo XVI. Ludovico
Ferrari (1522–1565), discípulo de Cardano,
encontró el método general de la resolución de la
ecuación de cuarto grado. Posteriormente, René
Descartes (1596–1650), sabio y filósofo francés,
inventor de la geometría analítica descubre otra
forma de resolver la ecuación cuártica.
Como es lógico, los matemáticos trataron de
resolver las ecuaciones de grado superior a cuatro
(quinto grado, sexto grado,…., de grado n). Este
estudio tenía un interés doble, ya que hubiera
constituido un gran logro encontrar un método
general de resolución para todas las ecuaciones de
una incógnita, cualquiera sea su grado.
Tras muchos intentos se llegó a la conclusión
de que las ecuaciones de quinto grado o superior
eran imposibles de resolver sólo usando cálculos
algebraicos. Un médico italiano de Bolonia, Paolo
Ruffini (1765–1822), había tratado de
demostrarlo en 1798, en su teoría general de las
ecuaciones; pero la demostración resultó
incompleta. Al cabo de unos años, el joven
matemático noruego Abel (1802–1829) descubrió
en 1824 el teorema que lleva su nombre y dice:
“Es imposible resolver algebraicamente las
ecuaciones generales de grado superior a
cuatro”.
Este teorema fue reforzado por Evariste
Galois (1811–1832), matemático francés,
fundador de la teoría de los grupos.
Dado que los matemáticos no lograron
encontrar métodos generales de resolución para
ecuaciones de grado superior a cuatro; trataron de
responder ciertas cuestiones como:
 ¿Cuántas raíces positivas posee una ecuación?
 ¿Cuántas raíces reales o complejas posee una
ecuación?
 Dados dos números a y b, ¿cuántas raíces de
una ecuación dada están comprendidas entre a
y b? (problema de la separación de las raíces
de una ecuación).
Desde este punto de vista los dos teoremas
fundamentales son el de René Descartes y el
teorema fundamental del álgebra (K. Gauss –
D′Alambert). Este teorema fue enunciado por
Girard en 1625, sólo realizó una demostración
incompleta por parte de D′Alambert (1746). La
primera demostración completa fue establecida
por K. Gauss (1799). Después Cauchy,
Weierstrass y Kronecker dieron otras
demostraciones.
El teorema de Gauss – D′Alambert se enuncia
“Toda ecuación polinomial de grado n posee
por lo menos una raíz (compleja o real)”.
CONTENIDO TEÓRICO:
S4AL33B “El nuevo símbolo de una buena educación....” S4AL33B “El nuevo símbolo de una buena educación...."
ECUACIONES DE
01 02COLEGIO DE CIENCIAS LORD KELVIN 4to Año Secundaria ALGEBRA 4to Año Secundaria
1. IGUALDAD DE NUMEROS REALES
Es la relación matemática donde nos indica
que dos cantidades tienen el mismo valor. Se
denota por el signo =, que se lee igual.
Veamos: 27 = 27 ; |9| = |- 9| ; A = B
AXIOMAS DE LA IGUALDAD.-
Enunciaremos los siguientes axiomas sobre la
Igualdad de Números Reales.
Axioma de Reflexividad: Todo número
real es igual a si mismo.
Si a ∈ R ⇒ a = a
Axioma de Simetría: Si un número real es
igual a otro, entonces el segundo es igual al
primero.
Si a = b ⇒ b = a, a; b ∈ R
Axioma de Transitividad: Si un número
real es igual a otro, y este otro es igual a un
tercero, entonces el primero es igual al
tercero.
Si a= b ∧ b = c ⇒ a = c; a; b; c ∈ R
2. ECUACIÓN
Una ecuación es una igualdad condicional
entre dos expresiones matemáticas definidas
sobre un mismo conjunto numérico, donde
participa por lo menos una variable (cantidad
desconocida llamada variable). Es todo
enunciado abierto en que aparece el signo “=”
y cuyo valor de verdad se determina mediante
su correspondiente conjunto de valores
admisibles para la variable (conjunto
solución).
Notación: A(x) = B(x)
OBSERVACIÓNES.-
Enunciado abierto: Es toda expresión que
contiene por lo menos una variable, que para
determinados valores de su dominio se
convierte en un enunciado verdadero o falso
llamado proposición.
Variable: Es el símbolo que puede tomar un
valor cualquiera de un determinado conjunto
llamado dominio. A las variables que
intervienen en la ecuación se les llama
incógnitas
Conjunto solución:
El conjunto solución de una ecuación es el
conjunto de valores (soluciones) que permiten
que la ecuación sea una proposición
verdadera.
Si una ecuación no posee solución alguna,
entonces definiremos a su conjunto solución
como el vacío y lo denotaremos por φ o {}
Ejemplo 1. Sea la ecuación: x3
= 4x.
Si x=1: 13
= 4(1) → 1 = 4 Proposición falsa
Si x=2: 23
=4(2)→8=8 Proposición verdadera
Si x=-2: (-2)3
=4(-2)→-8=-8 Proposición verdadera
Si x=0: 03
= 4(0) → 0= 0 Proposición verdadera
De lo expuesto; vemos que 2, - 2, 0 son
soluciones de la ecuación de acuerdo a la
definición, luego:
CS = {2, - 2, 0}
Ejemplo 2: La ecuación 3x – 5 = 0, tiene
como raíz o solución a: x = 5/3.
Luego, su conjunto solución es: C.S.=






3
5
3. CLASIFICACION DE LAS ECUACIONES
ALGEBRAICAS
3.1. DE ACUERDO A SU FORMA:
Ecuación polinomial: Es una ecuación
algebraica racional entera.
P(x) = ax + b= 0
P(x) = ax2
+ bx + c = 0
P(x) = ax3
+ bx2
+ cx + d = 0
P(x) = a0xn
+ a1xn–1
+ a2 xn–2
+ a3 xn–3
+...+ an – 1 x
+ an = 0
n ∈ Z+
∧ {a0; a1
;
a2; a3; ...an - 1;an} ⊂ R ;
a0; a1; a2; a3; ...; ; an - 1; an son los coeficientes.
Ecuación fraccionaria: Es una ecuación
algebraica racional fraccionaria.
P(x)=
2x
7
+
- 5x+11= 0 ....... CVA = R - {-2}
P(x)= 0
1x
4
3x
5
1x
3
=
−
−
+
+
+
.......CVA
=R-{-1,-3,1}
Ecuación Irracional:
P(x)= 03x2x 2 =−+− .Restricción de
la ecuación: x-2≥0→x≥2. Luego CVA=x∈ [2,+∞>
3.2.DE ACUERDO A SU CONJUNTO
SOLUCIÓN:
Ecuaciones consistentes o compatibles:
Son aquellas que tienen o aceptan por lo
menos una solución. A su vez se dividen en:
- Determinadas: Son aquellas que tienen
un número limitado de soluciones.
Ejemplo: x3
= x, CS = {1; 0; - 1}
- Indeterminadas: Son aquellas que tienen
un número ilimitado de soluciones. Ejm:
Ejemplo: x + 1 = x + 1, CS = R
Ecuaciones Inconsistentes o
Incompatibles.- Son aquellas que no tienen
solución, también se les denomina absurdas o
imposibles.
Ejemplo:
x
1
= 0 CS = φ
4. ECUACIONES DE PRIMER GRADO 0
LINEALES EN UNA VARIABLE
Son aquellas ecuaciones que tienen la forma:
P(x) = ax + b = 0
Donde: a, b son los coeficientes, “x” es la
incógnita.
Para obtener la única raíz o solución de la
ecuación, basta con despejar la incógnita, así
tendremos que: x =
a
b
− (presentación
única solución).
5. ANÁLISIS DE LA ECUACIÓN
PARAMETRICA EN VARIABLE “X”.
ax= b.........(*)
Caso I: Si: a ≠ 0 (no importa el valor de b),
reemplazamos en (*), obteniéndose
x = b/a una sola solución, con lo
cual su conjunto solución es finito,
luego (*) es compatible
determinada.
Caso II: Si: a = 0, b = 0, evaluando en (*) se
tiene 0x = 0, indicando que existen
infinitas soluciones, luego (*) es
compatible indeterminada.
CasoIII:Si: a = 0, b ≠ 0, al reemplazar en (*)
se obtiene 0x = b que carece de
soluciones, con lo cual su conjunto
solución es vacío, luego (*) es
incompatible.
S4AL33B “El nuevo símbolo de una buena educación....” S4AL33B “El nuevo símbolo de una buena educación...."
Nota: El conjunto de valores admisibles en una
ecuación polinomial son todos los reales.
Nota: El hecho de haber establecido el
conjunto de valores admisibles (CVA), no
implica haber resuelto la ecuación, sólo se le ha
restringido.
01 02COLEGIO DE CIENCIAS LORD KELVIN 4to Año Secundaria ALGEBRA 4to Año Secundaria
Ejemplo: En la ecuación paramétrica en “x”:
(a – 5) (a + 3) x = (a + 2) (a + 3)
Halle los valores de a para que sea:
I) Determinada II) Indeterminada
III) Incompatible
Resolución
I) (a-5)(a+3)≠



−≠
≠
3a
5a
0 ⇒ ∀a ∈ R -{- 3,
5}
II)(a – 5) (a + 3) = 0 ∧ (a + 2) (a + 3) = 0
(a = 5; a = – 3) ∧ (a = – 2; a = – 3)⇒ ∴a= - 3
III)(a – 5) (a + 3) = 0 ∧ (a + 2) (a + 3) ≠ 0
(a=5; a=- 3) ∧ (a ≠ - 2; a ≠ - 3) ⇒ ∴ a= 5
6. ECUACIONES EQUIVALENTES: Dos o más
ecuaciones de las mismas variables son
equivalentes, si y solo si poseen el mismo
conjunto solución.
Ejemplos:
P1 = 14
3
x2
2
x
=+ → CS = {12}
P2 = 5x – 36= 24 → CS = {12}
Como los conjuntos solución son iguales,
entonces P1 y P2 son equivalentes:
Luego, para resolver ecuaciones en general y de
primer grado en particular es necesario tener en
cuenta lo siguiente:
a) Si se divide ambos miembros de una ecuación
por una misma expresión que contenga a la
incógnita, entonces se perderán soluciones.
Esto se puede evitar si la expresión que se
divide (simplifica) se iguala a cero.
Ejemplo: Resolver: (x+3) (x-2) = 4 (x - 2)
Resolución
Simplificando (x - 2) para no perder
solución: x – 2 = 0 → x = 2
Luego, tendremos: x + 3 = 4 → x = 1
La ecuación tiene 2 soluciones x=2 y x=1 (de
no haber igualado a cero, hubiéramos perdido
la solución x=2).
b) Si se multiplican ambos miembros de una
ecuación por una misma expresión que
contenga a la incógnita, entonces se puede
introducir soluciones extrañas.
Esto se puede evitar si previamente se
simplifica por separado cada miembro de la
ecuación.
Ejemplo: Resolver:
( ) ( )
4
2
23
=
−
−+
x
xx
Resolución
Primero simplificamos (x - 2), y tendremos; x
+ 3 = 4 → x = 1
Observación.- Si hubiésemos trasladado (x -
2) a multiplicar, tendríamos que una solución
sería x = 2, que es una solución extraña, pues
no verifica la igualdad.
c) Si se eleva ambos miembros de una ecuación
a un mismo exponente, entonces se pueden
introducir soluciones extrañas.
Ejemplo: Resolver: 772
−=+ xx
Resolución
Elevando al cuadrado ambos miembros de la
ecuación propuesta:
( ) ( )222
77 −=+ xx
x2
+7=x2
–14x+49⇒ 14x = 42 ⇒ x = 3
Pero si reemplazamos; x = 3 en la ecuación
dada tendremos:
444167373 2
−=→−=→−=+
Proposición Falsa
(No cumple), luego: x = 3 es una solución
extraña, y la ecuación es incompatible, pues
no tiene solución:
Observación: Siempre que se potencie los
dos miembros de una ecuación. El valor o los
valores obtenidos para “x” deben
comprobarse en la ecuación original pues
pueden no ser soluciones verdaderas.
d) Si a ambos miembros de una ecuación le
sumamos un mismo número o un mismo
polinomio, la nueva ecuación es equivalente a
la inicial.
Observación: Si a ambos miembros se suma
o resta una función arbitraria la ecuación
resultante no necesariamente es equivalente a
la inicial.
La ecuación: x2
– 12 = 2x + 3 tiene por raíces:
x = 5; x = - 3
Sumando a los dos miembros de la ecuación
original:
5x
2
−
Obtenemos: x2
–12+
5x
2
−
=2x+3 +
5x
2
−
.
Para lo cual x = 5 no es solución.
Observaciones:
1. El conjunto solución de una ecuación
depende del conjunto numérico en que se
quiere resolver la ecuación, por ejemplo:
Si queremos resolver en el conjunto de los
racionales (Q), entonces el conjunto solución
de la ecuación: x2
= 2, es vacío; pues no existe
número racional cuyo cuadrado es 2. Si
embargo si resolvemos en el conjunto de los
reales (R), entonces el conjunto solución es {
2− , 2 }.
De la misma manera, la ecuación x2
= – 1, no
tiene solución en R, pero si la tiene en el
conjunto C. Al despejar x se obtiene: x =
1− ó x = - 1− .
Si definimos 1− =i (i es la unidad
imaginaria del conjunto C), el conjunto
solución es: {- i; i}.
2. Si p y q son expresiones algebraicas en una
variable “x”, entonces un enunciado de la
forma “p = q” se llama una ecuación
algebraica en “x”. Si obtenemos una
proposición verdadera cuando reemplazamos
x por x0; entonces x0 es llamada una solución
de la ecuación. x0 es un valor del dominio
(conjunto de valores admisibles) para x.
3. Si el conjunto solución de una ecuación es
todo el dominio para x, entonces la ecuación
se llama una IDENTIDAD, por ejemplo:
La ecuación: 2
1 x− =
)1)(1( xx +− es una identidad; pues
es cierta para todo número en el dominio para
x, esto es, en el intervalo cerrado: [- 1, 1].
4. Si en el dominio para “x” existen números
que no son soluciones, entonces la ecuación
se llama ecuación condicional o un enunciado
abierto. Por ejemplo; en la ecuación: x2
=
S4AL33B “El nuevo símbolo de una buena educación....” S4AL33B “El nuevo símbolo de una buena educación...."
Para resolver una ecuación de primer
grado es fácil, bastará con aplicar algunas
propiedades básicas de los números reales
hasta hallar el valor de la incógnita.
Se debe tener cuidado, cuando la variable
aparece en el denominador o cuando se
presenta un término radical; es justamente
en estos casos que aparece una raíz
extraña en algunas ecuaciones.
01 02COLEGIO DE CIENCIAS LORD KELVIN 4to Año Secundaria ALGEBRA 4to Año Secundaria
x , cuyo dominio para x es: [0, ∞>
existen números en el dominio que no son
soluciones, por ejemplo x = 4 ∈ [0, +∞>, y no
es solución, luego se trata de una ecuación
condicional.
PROBLEMAS EXPLICATIVOS
01.Sayumi tenía 120 nuevos soles. Si gastó los
7
5
de lo que no gastó. ¿Cuánto dinero gastó
Sayumi?
Resolución
Sea x la cantidad de nuevos soles que gastó
Sayumi. Entonces (120 - x) nuevos soles es lo
que no gastó.
Luego: Gasto =
7
5
(No gastó)
Entonces: x =
7
5
(120 - x)↔7x=600 – 5x
↔ 7x + 5x = 600
↔ 12x = 600
↔ x =
12
600
↔ x = 50
Respuesta: Sayumi gastó 50 nuevos soles.
02.Walter llega tarde al colegio cuando había
pasado un
8
1
de la clase de álgebra; 6
minutos después llega Jimmi y sólo escucha
los
5
4
de la clase. Si la clase empezó a las
8:00 de la mañana. ¿A que hora terminó?
Resolución
Sea t el tiempo (en minutos) que duró la
clase. Jimmi se pierde ( '6t
8
1
+ ) de la
clase, que equivale a
5
1
t (pues Jimmi sólo
escuchó los
5
4
t).
Luego:
5
1
t =
8
1
t + 6↔
5
1
t –
8
1
t =
6
↔
40
t3
= 6
↔ t =
3
6x40
↔ t = 80’
Respuesta: Como la clase empezó a las 8:00
a.m. y duró 80 minutos entonces terminó a las
9:20 a.m.
03.Un río tiene una corriente de 3 kilómetros por
hora. Si el bote de Aly Boydi tarda el mismo
tiempo en ir 18 kilómetros río abajo y 15 km.
río arriba. Calcule la velocidad del bote en
aguas tranquilas.
Resolución
Sea V la velocidad del bote en aguas
tranquilas, entonces (V + 3) es la velocidad
del bote río abajo (con la corriente a favor) y
(V - 3) es la velocidad del bote río arriba
(contra la corriente), entonces tenemos:
Distancia Velocidad Tiempo
Río Abajo 18 V+3
3V
18
+
Río Arriba 15 V – 3
3V
15
−
Como el tiempo es el mismo:
3V
18
+
=
3V
15
−
↔ 18 (V – 3)
= 15 (V + 3)
↔ 18V – 54 = 15 V + 45
↔ 18V – 15V= 45 + 54
↔ 3V = 99
↔ V =
3
99
↔ V = 33
Respuesta: La velocidad del bote en aguas
tranquilas es 33 kilómetros por hora.
PRÁCTICA DE CLASE
01.Clasificar las siguientes ecuaciones
algebraicas de acuerdo a su forma.
S4AL33B “El nuevo símbolo de una buena educación....” S4AL33B “El nuevo símbolo de una buena educación...."
Clasificación de las Ecuaciones
De acuerdo
a su forma
De acuerdo
a su C.S.
Algebraica
Poseer grado
No Algebraica Compatible Incompatible
(CS= O )
Polinomial
P(x)=a
o
xn + a x n-1
1
+ a x n-2
2
+ ...+ a = 0
n
Fraccionaria
3
x+2
= 5
Irracional
x - 7 = 7 - x
Se considera
Se caracterizan
por:
Exponencial
xx- 256= 0
Logarítmico
Log
6
x - 1= 0
Trigonométrico
Senx - x= 0
Puede ser
Determinada
(C.S. Finito)
Indeterminada
(C.S. Infinito)
Se enfoca
01 02COLEGIO DE CIENCIAS LORD KELVIN 4to Año Secundaria ALGEBRA 4to Año Secundaria
ECUACIÓN
ALGEBRAICA
CLASIFICACIÓN
7xx2x 23 −+−
= 0
x2
2x
5
−
−
= 0
ECUACIÓN
ALGEBRAICA
CLASIFICACIÓN
x24x −−− =
0
3x
3
5
2x
x3
−
−+
+
=
0
02.Clasificar las siguientes ecuaciones
algebraicas de acuerdo a sus soluciones:
ECUACIÓN
ALGEBRAICA
CLASIFICACIÓN
x3
= 9x
2x + 5 = 2x + 5
x +
x
1
x
1
=
ECUACIÓN
ALGEBRAICA
CLASIFICACIÓN
x(x - 2) = (x - 1)2
5x = 5x
5x − -
2x +−
03.Encierra en una circunferencia V (Verdadero)
o F (Falso).
- El conjunto de valores admisibles en una
ecuación algebraica implica que la ecuación
ha sido resuelta.
V - F
- En una ecuación polinomial sus
coeficientes son números naturales
V - F
- Una ecuación es una proposición
matemática
V - F
- Una ecuación compatible indeterminada
tiene infinitas soluciones.
V - F
04.Una ecuación compatible:
a) Tiene 2 incógnitas
b) No tiene solución
c) Tiene un número finito de soluciones
d) Tiene un número infinito de soluciones
e) c y d
05.Toda ecuación lineal presenta:
a) 1 solución b) 2 soluciones
c) 3 soluciones d) 4 soluciones
e) N.A.
06.Se llama ecuación polinomial a la:
a) Ecuación algebraica racional entera
b) Ecuación algebraica racional fraccionaria
c) Ecuación trascendente
d) Ecuación irracional
e) N.A.
07.Una ecuación se llama incompatible si:
a) Tiene infinitas soluciones
b) Tiene 3 incógnitas
c) Tiene un número finito de soluciones
d) Es irracional
e) No admite solución
08.Resolver: x + 5 +
6x
4
x7
6x
4
−
+−=
−
a) 6 b) – 6
c) 6 y – 6 d) Indeterminado
e) Incompatible
09.Resolver: x– 4 + 2
x420x8x5 −+−=−
a) 6 b) – 6
c) 6 y – 6 d) Indeterminado
e) Incompatible
10.Resolver:
2xx
9x
2x
3x
2
2
−+
−
=
+
−
. Marque lo
correcto:
a) Tiene una raíz b) Tiene dos raíces
c) Tiene tres raíces d) Indeterminado
e) Incompatible
11.Respecto a la ecuación en x, a (a2
– 1) x = 0,
establezca el valor de verdad de cada
proposición:
I. Es compatible para cualquier valor de
a.
II. Si a = –1, tiene infinitas soluciones.
III. Si a = 0, tiene solución única.
IV. Si a ∈ {0; 1; –1}, tiene una única
solución e igual a cero.
a) VVVV b) VFVF c) FFVV
d) FFFV e) FVFF
12.Determine el valor de verdad de las siguientes
proposiciones con respecto a la ecuación en
x:
( ) 21)2(2
−=−−+−− axaaaaaa
.
I. Es determinado cuando a ≠ 1∧a ≠ -1
II. Es indeterminado cuando a =1∨a= -1
III. Es incompatible cuando a = 2
a) VVV b) VVF c) VFV
d) FFV e) VFF
13.Luego de resolver la ecuación en “x”:
a
axaxax
ax 2
15
423
5
3
3
2
−
−
=
−
+
−
++
Es cierto que:
a) La solución depende de a (a ∈ ℜ)
b) Tiene una sola solución
c) No tiene solución
d) Tiene infinitas soluciones
e) Tiene dos soluciones
14.Luego de resolver la ecuación en “x”.






++=
−
+
−
+
−
cbaab
cx
ac
bx
bc
ax 111
2
I. Si a + b + c = 0 la ecuación tiene
infinitas soluciones con abc ≠ 0.
II. Si a + b + c ≠ 0 siempre existe solución
y es única.
III. Siempre la solución es a + b + c.
a) VVV b) VFV c) VFF
d) FVV e) FFV
EJERCICIOS PROPUESTOS Nº 03
01.Resolver: xx −=− 9)1( 2 .
a) Incompatible b) 0 c) 5
d) 5, - 5 e) Indeterminado
02.Resolver: 116513 +=++ xxx .
Indique la suma de sus raíces.
a) 0 b) 5 c) 6
d) 7 e) 9
S4AL33B “El nuevo símbolo de una buena educación....” S4AL33B “El nuevo símbolo de una buena educación...."
01 02COLEGIO DE CIENCIAS LORD KELVIN 4to Año Secundaria ALGEBRA 4to Año Secundaria
03.Resolver:
6
10
1
3
3
7
2
5
+
+
+
−
+
=
+
+
+
+
+
x
x
x
x
x
x
x
x
Indique: x3)x27( ++
a) 4 b) 2 c) - 27
d)
3 3 e) 2
04.Dada la ecuación en x:
6
75
3
21 +
=
+
+
+ xx
x
x
Dar el valor de verdad:
I. La ecuación dada es lineal
II. La ecuación tiene infinitas soluciones
III. La ecuación tiene solución única
IV. x = 32 + es solución de la
ecuación
V. La ecuación dada es ecuación
polinomial
a) FVFVV b) FVFVF c) VVVFF
d) FFVVV e) VFVFV
05.Para que valor real del parámetro “n”, la
ecuación del primer grado “x”: (2n – 1)x + 2
= nx – 3n2
será compatible y determinada.
a) ∀n ∈ R b) 2 c) 3
d) ∀n ∈ R+ e) ∀ n ∈ R - {+1}
06.En la siguiente ecuación:
(x+1) + (x+2)+(x+3) + ... + (x+n)= n2
, n
entero positivo, el valor de x es:
a)
2
)1( −n
b)
2
)1( +n
c)
2
n
d)
2
3n
e)
2
)12( +n
07.Si se define: P(n) = n + 3; f (m) = 3m. Calcular
“x” en:
f (P (f (P (2)))) – P (f (P (x))) = 75.
a) 4 b) -11 c) 12
d) -15 e) -1
08.Resolver:
1
22
1
22
1
=
−−+
+
−++ xxxx
a) 1 b) 4 c) 5
d) 2 e) - 1
09.Resolver: x + 2
25 x− = 7. ¿Cuántas
soluciones tiene?
a) 1 b) 2 c) 3
d) 4 e) 0
10.Hallar x en: x+++ 141221 = 5
a) 1 b) 4 c) 9
d) 16 e) 0
11.Si |– 9x| = 72. Calcular: |x – 3|.
a) 0 b) {1, 2} c) {5, 11}
d) 11 e) 5
12.Sea la ecuación en “x”:
a3
x – a4
+ 6a2
= (3a – 2)x + 8a – 3 e indicar el
valor de “a” para el cual la ecuación presenta
infinitas soluciones:
a) 1 b) 2 c) 3
d) 4 e) 0
13.Resolver:
3
)81...941(285
306
1
...
20
1
12
1
6
1
285
x
x +++++=





++++
a) 1 b) 7 c) 8
d) 9 e) 570
14.Si a ≠ b, resolver en x: a (x - a2
) - b(x -b2
) =
0.
a) φ b) {0} c) {1}
d) {a + b} e) {a2 + ab + b2}
15.Determinar el cardinal del conjunto solución
de la ecuación: =− 2
)2(x - 9.
a) 0 b) 1 c) 2
d) 4 e) N.A.
16.Resolver la ecuación:
(x+1) + (x+2)+(x+) +... +(x+20) = 420 – x.
a) 0 b) 5 c) 10
d) 12 e) 21
17.Hallar m y p para que la ecuación:
3mx–4p=2x+m. Sea:
I) Incompatible II) Indeterminada
Señalar la suma de soluciones de m:
a) 2/3 b) 1/3 c) 1
d) 4/3 e) 5/3
18.Si: 2
)1( −x = 1 - x. El conjunto solución
de la ecuación es:
a) x = 1 b) x = 3 c) x > 1
d) x < 1 e) x = 2
19.Resolver la ecuación de primer grado:
(m – 3)x2
+ 5m + (m – 2)x – 14 = 0
a) 1 b) - 1 c) 0
d) 19 e) 15
20.Resolver: x – 7+
5
11
−x
= 3 – x +
5
11
−x
a) 5 b) 5; - 5 c) - 5
d) Indeterminado e) Incompatible
TAREA DOMICILIARIA
01.Compre cierto número de folletos de álgebra
por 100 nuevos soles. Si el precio por el
ejemplar me hubiese costado un nuevo sol
menos, tendría 5 ejemplares más por el
mismo dinero. ¿Cuántos folletos compre?
a) 5 b) 4 c) 25
d) 20 e) 15
02.José tiene tres veces los años que tenía
Ricardo cuando el tenía 16 años. Ricardo
tiene 24. Hallar la edad de José.
a) 25 b) 20 c) 40
d) 30 e) 35
03.En un reloj se lee: 8: 48 cuando en realidad
son: 8:52, más tarde a las 9:42 se lee 9:34, y
según esto; ¿A que hora se tenía una lectura
correcta?
a) 8:02 b) 8:00 c) 8:04
d) 8:25 e) 9:11
04.En una sala de juego para entrar se paga 1
dólar y para salir 1 dólar. Una persona juega
en 3 salas y pierde en cada una la mitad de lo
que tiene. ¿Cuánto tenía antes de empezar a
jugar si al final se queda sin dinero?
a) 20 b) 21 c) 22
d) 23 e) N.A.
05.El conjunto solución de:
9x
15x6x
3x
4x
3x
1x2
2
2
−
+−
=
−
−
−
+
−
, es:
a) IR b) {3, - 3} c) {4, - 4}
d) IR - {3, - 3} e) N.A.
06.Resolver: x+++ 133210 = 4
S4AL33B “El nuevo símbolo de una buena educación....” S4AL33B “El nuevo símbolo de una buena educación...."
01 02
ax2
+bx + c = 0 , {a; b; c} ⊂ R / a≠ 0
COLEGIO DE CIENCIAS LORD KELVIN 4to Año Secundaria ALGEBRA 4to Año Secundaria
a) 12 b) 16 c) 25
d) 36 e) 9
07.Resolver la ecuación:
2
1
65
5
3
4
2
=
+−
−
+
−
−
xx
x
x
x
a) 1 b) 2 c) 3
d) 4 e) 5
08.Resolver la ecuación:
3
11
3
1
1
=
+
−
−
x
x
x
a) 18 b) 9 c) 25
d) 16 e) 4
09.Resolver:
2
4
2
+
=++
x
xx .
a) 1/2 b) 4/5 c) 2/3
d) 3/2 e) 3/4
10.Resolver:
ab
baax
a
bx
b
ax 2
)(32 −+
=
−
−
+
.
a)
2
ba +
b)
ba
ab
+
c)
ba
ba
−
+
d)
ba
ab
+
2
e)
ba
ba
+
−
11.Indique que pares de ecuaciones son
equivalentes:
I. x = 4; x2
= 16 II. x = 4; x2
= 4x
III. x = 4; x = 16 IV. x = 4; 4x = 16
a) Las 4 posibilidades planteadas
b) Sólo I y II
c) Sólo II y III
d) Sólo III y IV
e) Sólo I y IV
12.El valor de x que satisface la ecuación
fraccionaria:
3
1
1
1
1
1
2
1
1
1
1
+
+
=
+
+
x
.
a) 3/4 b) 1/2 c) 2/3
d) 5/6 e) 7/6
13.Hallar el valor de x en:
64......
3 3 32424
=∞÷÷÷÷ xxxx
a) 2 b) 32 c) 16
d) 4 e) 64
14.Hallar el valor del parámetro “a” de modo
que la ecuación a2
x + 2x + 2 = a2
+ a + 3ax
sea:
 Compatible determinado
 Compatible indeterminado
 Incompatible
BIBLIOGRAFIA
1. PERELMAN : Álgebra Recreativa
2. POTAPOV : Álgebra
3. SWOKOVSKI : Álgebra Universitaria
4. ACADEMIA CESAR VALLEJO: Compendio
Académico
5. RYBNIKOV: Historia de la Matemáticas
6. ALFONSO MORALES: Matemática
Resumida
OBJETIVOS ESPECIFICOSOBJETIVOS ESPECIFICOS:
 Dado un conjunto de Ecuaciones de segundo
Grado, resolverlos aplicando correctamente
las propiedades que corresponden.
COMENTARIO PREVIOCOMENTARIO PREVIO:
La historia del álgebra comenzó en el antiguo
Egipto y Babilonia donde fueron capaces de
resolver ecuaciones lineales (ax = b) y
cuadráticos (ax2
+ bx + c) así como ecuaciones
indeterminadas como x2
+ y2
= z2
con varias
incógnitas. Los antiguos Babilonios resolvían
cualquier ecuación cuadrática empleando
esencialmente los mismos métodos que hoy se
enseñan.
También fueron capaces de resolver algunas
ecuaciones indeterminadas.
Los matemáticos alejandrinos Heron y
Diofante continuaron con la tradición de Egipto y
Babilonia aunque el libro "La aritmética de
Diofante" es de bastante más nivel y presenta
muchas soluciones es sorprendente para
ecuaciones indeterminadas difíciles. Esta antigua
sabiduría sobre resolución de ecuaciones
encontró a su vez, acogida en el mundo islámico
en donde se le llamó "ciencia de reducción y
equilibrio" (La palabra árabe "ál - jabru" que
significa reducción es el origen de la palabra
álgebra).
En el siglo IX el matemático al-juarizmi escribió
uno de los primeros libros árabes de álgebra, una
presentación sistemática de la teoría fundamental
de ecuaciones con ejemplos y demostraciones
incluidas
CONTENIDO TEÓRICO:CONTENIDO TEÓRICO:
ECUACIONES DE SEGUNDO GRADOECUACIONES DE SEGUNDO GRADO
Llamadas también ecuaciones polinomiales de
segundo grado, cuya forma general es:
Frecuentemente a dicha ecuación se le llama:
Ecuación Cuadrática y se caracteriza por
presentar 2 soluciones (su incógnita “x” asume
dos valores)
MÉTODOS DE RESOLUCIÓN DE LA
ECUACIÓN.
Toda ecuación de 2do
grado podrá resolverse al
menos por una de las siguientes formas:
A) Por Factorización
Este método se aplica únicamente si el
trinomio:
ax2
+ bx + c es factorizable por aspa simple,
para lo cual se debe tener en cuenta la
siguiente propiedad:
Si: m . n = 0 ⇒ m = 0 ∨ n = 0
Resolver: ax2
+ bx + c = 0
Factorizando se obtiene: a(x–x1) (x–x2) = 0
De donde:x–x1=0∨x–x2 =0⇒x= x1∨x= x2
∴C.S = {x1; x2}, x1; x2 se llaman raíces de la
ecuación polinomial cuadrática.
Ejemplo: Resolver la siguiente ecuación: x2
–
x – 12 = 0
Resolución
La ecuación dada es: x2
– x – 12=0
Factoricemos al trinomio: x2
– x – 12
Según el criterio del aspa
x2
– x–12=(x–4)(x+3)
simple tendremos: x -4
x 3
luego la ecuación dada será: (x–4) (x+3) = 0
Finalmente de acuerdo a la propiedad señalada
líneas arriba; se tendrá:
S4AL33B “El nuevo símbolo de una buena educación....” S4AL33B “El nuevo símbolo de una buena educación...."
ECUACIONES DE
01 02COLEGIO DE CIENCIAS LORD KELVIN 4to Año Secundaria ALGEBRA 4to Año Secundaria
x – 4 = 0 ∨ x + 3 = 0 ⇒ x = 4 ∨ x= -3
Es decir el conjunto solución de la ecuación:
x2
– x – 12= 0, es : C.S. = {4; -3}
B) Por la Fórmula de Carnot
Dada la ecuación: ax2
+ bx + c = 0, sus raíces
se obtienen utilizando la fórmula deducida
por Sadi Carnot:
a
acbb
x
2
42
−±−
=
Donde las raíces son:
a2
ac4bb
x
2
1
−+−
= ;
a2
ac4bb
x
2
2
−−−
=
Luego el conjunto solución es:







 −−−−+−
=
a
acbb
a
acbb
SC
2
4
;
2
4
..
22
Ejemplo: Resolver la siguiente ecuación: x2
+ 3x
– 1 = 0
Resolución
De la ecuación se deduce que: a = 1 ∧ b = 3 ∧ c
= –1
Reemplazando en la fórmula tenemos:
)1(2
)1)(1(433
x
2
−−±−
=
Efectuando y reduciendo:
2
133
x
±−
=
Finalmente las raíces de la ecuación son:
2
133
x1
+−
= ;
2
133
x2
−−
=
En consecuencia el conjunto solución es:







 −−+−
=
2
133
;
2
133
.S.C
ANÁLISIS DE LA ECUACIÓN.
Para la ecuación: ax2
+ bx + c = 0, se tiene:
I) Si: a ≠ 0 ∧ {b ; c} ⊂ R , la ecuación es :
Compatible Determinada
II) Si: a = 0 ∧ b = 0 ∧ c = 0, la ecuación es
compatible Indeterminada.
III) Si: a = 0 ∧ b = 0 ∧ c ≠ 0, la ecuación es
Incompatible.
NATURALEZA DE LAS RAÍCES.
A) DISCRIMINANTE (∆)
Llamamos discriminante a la expresión
subradical contenida en la fórmula de Carnot:
∆ = b2
– 4ac
De este modo la fórmula que da solución a
una ecuación de segundo grado queda así :
a2
b
x
∆±−
=
B) ANÁLISIS DEL DISCRIMINANTE
Observando la relación anterior, resulta
previsible que el valor y/o signo del
discriminante determinará la naturaleza de las
raíces de una ecuación de 2do
grado. Veamos
los siguientes casos:
Primero: Si: ∆ > 0
En este caso las raíces de la ecuación serán
reales y diferentes.
Segundo: Si: ∆ = 0
En este caso las raíces de la ecuación
cuadrática serán reales e iguales. Este caso
se presenta cuando el trinomio “ax2
+ bx + c”
es un cuadrado perfecto.
Tercero: Si : ∆ < 0
En este caso las raíces de la ecuación serán
imaginarias y conjugadas.
Debe notarse que las raíces imaginarias
siempre se presentan en parejas, siendo una la
conjugada de la otra.
Cuarto: Si:∆ = k2
(cuadrado perfecto)
Siendo a, b ∧ c números racionales, las raíces
de la ecuación serán reales racionales. Pero
si ∆ ≠ k2
, las raíces de la ecuación serán
reales irracionales y conjugadas.
PROPIEDADES DE LAS RAÍCES.
Para la ecuación: ax2
+ bx + c = 0 / a ≠ 0, de
raíces x1 ∧ x2 , tenemos:
I) Suma de Raíces: s = x1 + x2 =
a
b
−
II) Producto de Raíces: p = x1 . x2 =
a
c
III) Diferencia de Raíces: d= x1-x2=
a
∆
Para determinar la diferencia de las raíces se
recomienda utilizar la siguiente identidad
21
2
21
2
21 4)()( xxxxxx =−−+
A) RAÍCES PARTICULARES
En algunas ecuaciones las raíces se
condicionan de tal modo que efectuando
alguna operación elemental entre ellas, se
podrá deducir alguna propiedad particular,
como por ejemplo:
Raíces Simétricas: Si x1 ∧ x2 son raíces
simétricas, se podrá establecer lo siguiente:
x1 = m ∧ x2 = –m ⇒ x1 + x2 = 0
Raíces Recíprocas: Si x1 ∧ x2 son raíces
recíprocas, se podrá establecer lo siguiente:
x1 = m ∧ x2 =
m
1
⇒ x1 . x2 = 1
B) RAÍCES ESPECIALES
Llamaremos así a las siguientes raíces:
Raíz Nula:
Dada la ecuación cuadrática ax2
+bx +c=0 / a
≠ 0, si ésta presenta una raíz nula (x=0), se
cumplirá que: c = 0.
Raíz Unidad:
Dada la ecuación cuadrática ax2
+bx+c=0 / a ≠
0, si ésta presenta una raíz unidad (x =1), se
cumplirá que:
a + b + c = 0.
RECONSTRUCCIÓN DE UNA ECUACIÓN
CUADRÁTICA.
Considerando a x1 ∧ x2 como raíces de la
ecuación tal que:
S = Suma de raíces
P = Producto de raíces
Entonces la ecuación que originó a dichas raíces
se determina así:
x2
– Sx + P = 0
PROPIEDADES IMPORTANTES.
A. De las Ecuaciones Equivalentes
S4AL33B “El nuevo símbolo de una buena educación....” S4AL33B “El nuevo símbolo de una buena educación...."
01 02COLEGIO DE CIENCIAS LORD KELVIN 4to Año Secundaria ALGEBRA 4to Año Secundaria
Sean:
a1 x2
+ b1 x + c1 = 0 ...... (1)
a2 x2
+ b2 x + c2 = 0 ...... (2)
Dos ecuaciones equivalentes, luego entre
ellas se cumplirá la siguiente relación:
2
1
2
1
2
1
c
c
b
b
a
a
==
PRÁCTICA DE CLASEPRÁCTICA DE CLASE
01.Resolver las siguientes ecuaciones
a) +
+
3
1x
5
2x −
=
15
1x8 +
b) x - 7212
=−x
c)
34
4
9
1
32
2
222
+−
=
−
+
−
−+
−
xxx
x
xx
x
02.Resuelve las siguientes ecuaciones
cuadráticas:
a) (x + 1)(x + 2)(x+3) = x(x + 4)(x + 5)
b) 3
2
2
1
1
=
+
−
+
−
+
x
x
x
x
c) 3
72 x− - 3
16 x− = 2
d)
2
5
2
3
3
2
=
−
−
+
−
−
x
x
x
x
03.Completar:
a) 2x2
– 7x – 3 = 0 ∆ = …………………
b) 7x2
– 11x – 14 = 0 S = …………………
c) x2
– 5x + 6 = 0 =+
21 x
1
x
1
………..
d) 2x2
+ 7x + 1 = 0 1
2
1
1 xx −−
+ =
………
e) 2x2
+ x – 1 = 0 ( )( )1x1x 21 ++ =
….
f) x2
+ 2x – 1 = 0 ( )( )1x1x 21 ++ =
….
04.Relaciona correctamente:
I) x2
– 4 3 x + 12=0 a)Raíces reales
iguales
II) x2
– 2x – 1 = 0 b)Raíces reales
diferentes
III) x2
– 2x + 3 = 0 c)Raíces complejas
a) I A – II B – III C d) I C – II B – III A
b) I B – II C – III A e) I A – II C – III B
c) I C – II A – III B
05.Calcular “m” para cada uno de los siguientes
casos, siendo la ecuación cuadrática:
(m + 1)x2
– (3m – 5)x + 2m – 5 = 0
a) Suma de raíces es 5/2 m=……......
b) Producto de raíces es 9/4 m=……......
c) Raíces recíprocas. m=……......
d) Raíces simétricas m=……......
e) Una raíz es – 2 m=……......
06.Calcular “n” para cada uno de los siguientes casos,
siendo la ecuación cuadrática:
(2n - 5)x2
+ (3n - 5)x + n + 1 = 0
a) Raíces iguales
b) Suma de las inversas de las raíces es –5/2
c) Diferencia de raíces es 0,5
d) Suma de los cuadrados de las raíces es 5/4
07.Formar una ecuación cuadrática con
coeficientes enteros para cada uno de los
siguientes casos:
a) x1 = 7 x2 = 4
b) x1 = 2/3 x2 = - 3/5
c) x1 = 3 - 2
d) x1 = 4 + i
e) x1 + x2 = - 7/3 ∧ x1 . x2 = 5/9
08.¿Para qué valor de “m” las raíces de la
ecuación:
x2
–(m+3)x+
4
m 2
+1=0; se diferencian en 2?
a) –
6
1
b)
3
1
c) -
3
1
d)
6
1
e)
3
2
09.La ecuación de segundo grado una de cuyas
raíces es la fracción:
x =

1
2
1
3
1
2
1
3
1
1
+
+
+
+
+
; está dada por:
a) 3x2
– 5 = 0 b) 5x2
– 3 = 0 c)3x2
–x–5=0
d) 5x2
– x – 3=0 e) 2x2
- 4 = 0
10.Determine la suma de los valores que puede
tomar “a” para que la ecuación: (a + 1) x2
+
ax + 1 = 0; tenga una sola solución si “a” es
un número real y diferente de –1.
a) 12 b) 4 2 c) 4
d) 5 e) 6
11.Sea: {x1; x2} el conjunto solución de:
3x2
– x – 1 = 0. A continuación se establece
que:
P(n) = n n
2
n
1
xx + ; calcular: P(2)
a) 7 b)
3
7 c) 3
d) 7 e) 3
12.Si la ecuación: x2
– 6x + n + 1 = 0, admite
como raíces a x1 ∧ x2 , tal que :
5
3
x2
1
x2
1
21
=+ ;
Encontrar el valor de n:
a) 1 b) 2 c) 3
d) 4 e) 5
13.¿Para qué valor de “n” el discriminante de la
ecuación: x2
– 8x + n = 0, es igual a 20?
a) 44 b) 11 c) 33
d) 22 e) 17
14.Sabiendo que las ecuaciones:
x2
+ mx + n = 0
x2
+ nx + m = 0
Presentan una raíz común, formar otra
ecuación cuadrática cuyas raíces sean las no
comunes de las anteriores
a) x2
+ x – 1 = 0
b) x2
+ (m – n) x + mn = 0
c) x2
– x + 1 = 0
d) x2
– (m + n) x + mn = 0
e) x2
– mn = 0
S4AL33B “El nuevo símbolo de una buena educación....” S4AL33B “El nuevo símbolo de una buena educación...."
01 02COLEGIO DE CIENCIAS LORD KELVIN 4to Año Secundaria ALGEBRA 4to Año Secundaria
15.En la ecuación: 2x2
– (m - 1) x + m + 1 = 0,
¿qué valor positivo debe darse a “m” para que
las raíces difieran en uno?
a) 7 b) 8 c) 9
d) 10 e) 11
16.Sabiendo que: (p + q)2
y (p – q)2
son raíces de
cierta ecuación cuadrática recíproca donde
“p” y “q” son raíces de la ecuación: ax2
+ bx
+c = 0; a > b > 0, calcular a4
– b4
a) 2abc b) – 2abc2
c) 4abc2
d) – 4 ab2
c e) – 4abc2
17.Sabiendo que la ecuación: x4
– 9x + λ = 0
admite dos raíces que suman 3, calcular el
producto de todas las raíces
a) 3 b) 6 c) 9
d) 12 e) 18
18.Si las raíces de la ecuación en “x”
x2
– 3x + m + 1 = 0
3x2
+ 5x + m = 0
Son imaginarias y reales respectivamente
determine el valor entero de “m”
a) 0 b) 1 c) - 1
d) 4 e) 2
19.Determine a + b +c de modo que la ecuación:
x3
– ax2
+ bx + c = 0
Admita por raíces: a, b, c; abc ≠ 0
a) 1 b) - 1 c) 0
d) 4 e) 8
20.Resolver:
18
x
1
x3
x
1
x
3
=





+−





+
Indicar la raíz de mayor valor
a) 2 +1 b) 3-2 2 c) ( 2 +1)2
d) 2+3 2 e) (3+ 5 )/2
EJERCICIOS PROPUESTOS Nº 04
01.Indicar la mayor raíz de la ecuación: x2
- 3x +
2,16 = 0
a) 1,2 b) 0,8 c) 1,8
d) 0,3 e) 1,2
02.Si : x = ....221 +++ , puede
decirse que:
a) x = 3 b) 0<x<1 c) x>2
d) x =2 e)x es infinitamente grande
03.Cuál o cuáles de las siguientes ecuaciones:
I. x2
– x – 1 = 0 II. x2
– 2x + 3 = 0
III. 3x2
+ x – 2 = 0
No admite raíces reales.
a) Solo I b) Solo II c) Solo III
d) II y III e) I y II
04.Halle la menor raíz de la siguiente ecuación
mónica de segundo grado: (m - 2) x2
– (3m -
8) x + m – 9 = 0
a) -2 b) -3 c) 2
d) 3 e) -1
05.Calcular el valor de “m-2n” si la ecuación
cuadrática:
5 (m + n +18) x2
+ 4(m - n) x + 3mn = 0
Es incompatible.
a) -9 b) -18 c) 9
d) 18 e) -13
06.Calcular la mayor solución de la ecuación:
(m - 2) x2
– (2m - 1) x + m – 1 = 0
Sabiendo que su discriminante es 25.
a) 3 b) 0,5 c) 2,5
d) 1,5 e) N.A.
07.Calcular “m” para que la ecuación:
6x2
+ (2m + 3) x + m = 0. Tenga única
solución.
a) 3 b) 3/4 c) 1/2
d) 3/2 e) 5/3
08.Indicar el valor de verdad de las siguientes
proposiciones, en base a la ecuación:
x (x - 1)2
(2x - 3)3
(x2
- 3 )2
= 0
( ) Posee 4 raíces o soluciones
( ) Su conjunto solución posee 5 elementos
( ) Posee a x = 0 como raíz simple y a x =
3/2 como raíz triple.
a) VVV b) FVV c) FFV
d) VFV e) VVF
09.En la ecuación cuadrática: ax2
+bx+c = 0
Afirmamos:
I. Si la suma de sus raíces es igual a su
producto entonces b + c = 0
II) Si una raíz es la opuesta de la otra
entonces b = 0
III) Si una raíz es el doble de la otra, entonces
2b2
= 9ac
a) Las 3 afirmaciones son verdaderas
b) I y II son verdaderas
c) I y III son verdaderas
d) II y III son verdaderas
e) Sólo II es verdadera
10.Si “r” y “s” son las raíces de la ecuación:
ax2
+ bx + c = 0 ; el valor de :
22
s
1
r
1
+ , es:
a) b2
- 4ac b)
a2
ac4b2
−
c)
2
2
c
ac4b −
d)
2
2
c
ac2b −
e) b2
+ 4ac
11.Si la ecuación: x2
– nx + 36 = 0, admite como
raíces a : x1 ∧ x2, tal que:
12
5
x
1
x
1
21
=+ ; encontrar el valor de “n”.
a) 25 b) 18 c) 12
d) 24 e) 15
12.Siendo : x1 ∧ x2 las raíces de la ecuación :
5x2
– 23x + 11 = 0 , el valor de:
9x2
1x3
.
9x2
1x3
2
2
1
1
−
+
−
+
; es:
a)
35
17
b)
35
143
c)
35
153
d)
35
183
e)
35
173
13.¿Para qué valores de “m” la ecuación:
x2
- 2(3m+1) x + 7(2m+3) = 0, tendrá sus dos
raíces iguales?
a) 5 ; 2 b) 1 ; -3/2 c) 4 ; -2
d) 3 ; -1 e) 2 ; -10/9
14.La ecuación cuadrática cuyas raíces son:
2+ 2 ∧ 2- 2 , es:
a) x2
+ 2x – 1= 0 d) x2
+ 4x +2= 0
b) 2x2
- 4x + 1= 0 e) x2
- 4x + 2= 0
S4AL33B “El nuevo símbolo de una buena educación....” S4AL33B “El nuevo símbolo de una buena educación...."
01 02COLEGIO DE CIENCIAS LORD KELVIN 4to Año Secundaria ALGEBRA 4to Año Secundaria
c) x2
- 8x + 2= 0
15.Si “α” y “θ” son las raíces de la
ecuación:
x2
- 2x – 5 = 0, encontrar una ecuación
cuadrática cuyas raíces sean: α2
y θ2
.
a) x2
+14x + 25=0 d) x2
+14x+15= 0
b) x2
- 2x - 1= 0 e) x2
- 14x - 25= 0
c) x2
- 14x + 25= 0
16.Si x1 y x2 son raíces reales de: ax2
+bx+c=0 (a
≠ 0), calcular el valor de “m” para que la
ecuación de raíces (x1 + m) y (x2 + m);
carezca de término lineal
a) – b /2a b) b/2a c) b/a
d) – b/a e) b/3a
17.Determinar la ecuación de segundo grado
cuyas raíces sean: una la suma y la otra el
producto de las raíces de: ax2
+bx+c=0; a≠ 0
a) a2
x2
– a(b - c)x – bc = 0
b) a2
x2
– a (b + c)x – bc = 0
c) a2
x2
– a (b + c)x + bc = 0
d) a2
x2
+ a (b - c)x + bc = 0
e) a2
x2
+ a (b - c)x – bc = 0
18.Siendo x1 y x2 las raíces de la ecuación:
ax2
+ bx + b = 0; a ∧ b ≠ 0
Tales que x1 es a x2 como “b” es a “a”
calcular:
21 xx
a
b
b
a
R ++=
a) 0 b) 1 c) 2
d) 3 e) 4
19.La ecuación P(x) = x2
+ bx + c = 0; tiene raíces
reales positivas distintas, entonces de las
raíces de la ecuación:
F(x) =
2
x + b x + c = 0; se puede
afirmar:
a) Son las mismas de P(x)
b) Algunas son negativas
c) Algunas son complejas
d) Son todas positivas
e) Son todas negativas
20.Hallar la ecuación de segundo grado de
coeficiente principal 1 y de raíces m y n se
sabe que:
 x2
+ (m – 1) x + m – 2 = 0; tiene una sola
solución real.
 x2
– (n + 1) x + 2n = 0; tiene una raíz
igual a 3.
a) x2
+ 9x + 18 = 0
b) x2
– 6x + 18 = 0
c) x2
– 9x – 18 = 0
d) x2
– 9x + 18 = 0
e) x2
– 6x – 18 = 0
TAREA DOMICILIARIATAREA DOMICILIARIA
01.Resolver las ecuaciones:
1) x2
= 7
2) (x + 1) (x – 3) = 12
3) 15x2
– 34x + 15 = 0
4) (x + 3) (x + 5) = 13x2
5) x(x - 1997) = (x - 1997)
Indicar la ecuación que posee la menor raíz
a) 1 b) 2 c) 3
d) 4 e) 5
02.Sea la ecuación:
[(m + n)2
– (m - n)2
] x2
+ (m - 1)2
x – [(m + n)2
+ (m - n)2
] = 0 siendo m ≠ 0 ∧ n ≠ 0 y x1 y x2
son sus raíces. ¿En cuántas unidades es
necesario disminuir dichas raíces para que
sean simétricas?
a) 1/n b) – 1/n c) 1/2 n
d) – 2n e) – 1/2 n
03.Hallar una de las raíces de la ecuación:
a (b - c)x2
+ b (c - a) x + c (a - b) = 0
Si x es la incógnita
a)
ba
cb
−
−
b)
cb
ac
−
−
c)
( )
( )cba
cab
−
−
d)
ac
ba
−
−
e)
( )
( )cba
cab
−
−
04.Dada la ecuación: x2
- 2x + m = 0. Calcular
“m” si una de las raíces es 1 + 2i, (i = 1−
); m ∈ R
a) 2 b) 3 c) 4
d) 5 e) 8
05.Si la ecuación: x2
+ px + q = 0; tiene por
conjunto solución {r, s} si: r – s = 4 y r3
– s3
= 208; entonces p/q es:
a) 2/3 b) 3/2 c) 2/5
d) 2/7 e) 1/7
06.Hallar el valor de “a” para que las raíces de la
ecuación: x2
– (a + 3) + 1
4
2
+
a
= 0 se
diferencien en 5
a) 5/3 b) 7/3 c) 10/3
d) 5/6 e) 20/3
07.Resolver e indicar la solución:
275232522 =−+++−+− xxxx
a) 7 b) 13 c) 15
d) 5 e) 16
08.Calcular “m” para que la ecuación:
6x2
+ (2m + 3) x + m = 0 tenga una raíz
solamente
a) 3 b) 3/4 c) 1/2
d) 3/2 e) 5/3
09.Sea la ecuación: 021 =++ xx
Indicar el valor de verdad de las
proposiciones:
( ) Si la ecuación admite solución, ésta debe
estar comprendido en [-1; 0]
( ) La ecuación tiene dos soluciones reales
( ) La ecuación tiene una única solución
a) VFV b) VFF c) VVF
d) VVV e) FVV
10.Resolver: (1 + x) (1 + 2x) (1 + 3x) = - 15
Indicar la suma de las raíces no reales:
a) 0 b) 1/2 c) – ½
d) - 1 e) 1/6
11.Sea el polinomio cuadrático:
P(x) ≡ (n + 1)! x + n! (x) + (n - 1)!; n ∈ N,
indicar verdadero o falso, si P(x) = 0, según
corresponda:
( ) P(x) tiene raíces reales y diferentes ∀ n ∈ N
( ) P(x) tiene siempre raíces imaginarias y
conjugadas
( ) Para algún n ∈ N, P(x) tiene raíces iguales
a) FFV b) FVV c) VFV
d) VVV e) FVF
12.Si r y s son raíces de la ecuación cuadrática:
mx2
– 2(m – 1) x + m = 0 y cumplen
r
s
s
r
+
=4, halle la suma de todos los valores “m”
que satisfacen la condición
a) 1 b) - 4 c) - 1
S4AL33B “El nuevo símbolo de una buena educación....” S4AL33B “El nuevo símbolo de una buena educación...."
01 02COLEGIO DE CIENCIAS LORD KELVIN 4to Año Secundaria ALGEBRA 4to Año Secundaria
d) 0 e) 4
13.El producto de multiplicar el término
independiente con el coeficiente del término
cuadrático de la ecuación que tiene por raíces
el cuadrado de la inversa de las raíces de
ax2
+ bx + c = 0, a ≠ 0, es:
a) ac b) a2c2 c) a/c
d) 1/a2c2 e) c/a
14.Hallar la suma de los cuadrados de las raíces
de la ecuación polinomial:
F(x) = x3
– 3x + 6 = 0
a) 1 b) - 1 c) 4
d) 8 e) 6
15.Si x1, x2, x3 son las raíces de la ecuación:
4x3
+ mx2
- 4x + m2
= 0
Además:
cba
cba
x
++
++
=
43
1 ;
ca
cba
x
+
++
=2
;
c
cba
x
2
3
++
= , calcule un valor de “m”
a) 0 b) - 1 c) 2
d) - 2 e) 1
OBJETIVOS ESPECIFICOSOBJETIVOS ESPECIFICOS:
 Reconocer una ecuación polinomial
e indicar la relación existente entre solución y
raíz.
 Resolver ecuaciones de cualquier
grado aplicando los teoremas y técnicas
adecuadas.
COMENTARIO PREVIOCOMENTARIO PREVIO
Al - Guarismi, el año 1 100 estudia ecuaciones
del tipo:
ax2
+e=bx, ax2
+bx=e , ax2
+ bx + c = d; etc y da
soluciones para cada caso.
La época de oro de las matemáticas Italianas se da
en el siglo XVI, con Scipiene del Ferro, Nicola
Tartaglia, Girolamo Cardano, Ludovico Ferrari,
Frencois Viette, etc, quienes resolvieron las
ecuaciones de tercer y cuarto grado. Hecho de
trascendental importancia en esa época.
La historia da cuenta de que el profesor Scipiene
del Ferro logré resolver la ecuación de tercer
grado en 1515, pero no la dio a conocer siguiendo
las normas científicas de su época. Aún así,
confió sus resultados a Antonio Fiore.
En 1541 Antonio Fiore se bate en duelo
matemático con el profesor Nicola Trataglia
para ver quién resuelve la ecuación de tercer
grado, saliendo vencedor este último.
Cardano quien era médico, adivino y matemático
logra con tretas y promesas, que Tartaglia le
hiciera conocer la solución de la ecuación de
tercer grado. El mismo año Cardano publica su
libro “Arte Mayor” en donde da la solución de la
ecuación de tercer grado como suya y menciona
que Tartaglia no es sino un redescubridor ya que
del Ferro había dado la primera prueba hace 30
años.
En la misma obra aparece la solución de la
ecuación de cuarto grado, debido a Ludovico
Ferrari, discípulo de Cardano. Posteriormente se
dieron otras pruebas tanto de la ecuación de tercer
grado (F. Viette) como de la ecuación de cuarto
grado (R- Descartes)
Después de los rotundos éxitos de los
matemáticos Italianos viene nuevamente un largo
periodo de estancamiento en la tarea de la
solución de ecuaciones de quinto grado. Recién
en 1825, el joven matemático noruego Niels
Henrick Abel demostró que la ecuación general
de quinto grado no es resoluble mediante la
extracción de raíces y las operaciones aritméticas
conocidas.
Por otro lado en 1929 Evaristo Galois, probaría
que las ecuaciones de grado superior a cuatro no
son resolubles por radicales y dio las condiciones
necesarias y suficientes para que una ecuación de
cualquier grado sea resoluble por radicales.
Actualmente existen técnicas que permiten
resolver ecuaciones de cualquier grado.
CONTENIDO TEÓRICO:CONTENIDO TEÓRICO:
ECUACION POLONOMIAL EN UNA
INCÓGNITA
Es aquella ecuación que tiene la siguiente forma
general:
P(X) = a0 xn
+ a1xn – 1
+ a2 xn – 2
.... + an-1 x + an = 0
Donde: a0; a1; a2;...... : an – 1 ; an : son sus
coeficientes
Si: a 0 ≠ 0→el grado de la ecuación es“n”(n ∈N)
x → es la incógnita
RAIZ DE UN POLINOMIO
Dado el polinomio P(x). Se denomina raíz o cero
del polinomio, al número “a” si y solo si el
polinomio P(x) es divisible entre (x - a).
El polinomio P(x) tiene una raíz de valor “a”
P(x) = (x - a) q(x)
Ejemplo: hallar las raíces de: P(x) = x3
– 6x2
+ 11x
– 6
Resolución
Factorizando se tiene: P(x) = (x –1) (x–2)(x – 3)
Luego las raíces o ceros de P(x). Son: {1; 2; 3}
Observación:
S4AL33B “El nuevo símbolo de una buena educación....” S4AL33B “El nuevo símbolo de una buena educación...."
ECUACIONES
01 02COLEGIO DE CIENCIAS LORD KELVIN 4to Año Secundaria ALGEBRA 4to Año Secundaria
Una manera práctica de hallar las raíces de un
polinomio P(x), es formar la ecuación: P(x) = 0.
Así:
P(x) =(x –1) (x – 2) (x–3)= 0. CS = {1; 2; 3}
En este ejemplo las raíces del polinomio P(x)
coinciden con las soluciones de la ecuación P(x) =
0, lo cual no ocurrirá siempre.
Raíz de Multiplicidad “k”:
Dado el polinomio P(x) se denomina raíz de
multiplicidad “k” (k ∈ Z+
) del polinomio P(x). Al
número “a”, si y sólo si el polinomio P(x) es
divisible entre (x – a)k
, pero no es divisible entre
(x – a)k+1
, es decir si:
P(x) = x4
– x3
– 3x2
+ 5x – 2
Factorizando se tiene: P(x) = (x – 1)3
(x + 2)
Luego las raíces de P(x) son: {1; 1; 1; –2} y se
dice que:
 “1” es una raíz de multiplicidad 3 (raíz triple)
 “2” es una raíz de multiplicidad 1 (raíz
simple)
Formemos la ecuación: P(x)=0⇒P(x)=(x– l)3
(x+ 2)
= 0
⇒ (x – 1)3
= 0 ∨ x + 2 = 0
⇒ x = 1 ∨ x = – 2
Luego: CS {1; –2}
Observación:
La ecuación antes expuesta tiene 4 raíces y dos
elemento en su conjunto solución.
Cuando un polinomio tiene raíces múltiples el
número de raíces y el número de soluciones no
coincide.
Ejercicio:
En la ecuación polinomial: x3
(x – 2)2
(x2
+ 9) (x
+ 3 3 ) = 0
Señale:
a) El número de raíces
b) El número de soluciones
c) Su conjunto solución
TEOREMA FUNDAMENTAL DEL ALGEBRA
Toda ecuación polinomial con cualquier tipo de
coeficientes numéricos tiene por lo menos una
raíz que generalmente es compleja.
Corolario:
Toda ecuación polinomial de grado n > 1. Tiene
exactamente “n” raíces complejas en general.
Luego dada la ecuación polinomial:
P(x)=a0 xn
+a1xn – 1
+.......+an–1x+an = 0: a0 ≠ 0
Se tiene: P(x) = a0(x – x1) (x – x2)...... (x–xn)= 0
Donde: {x1; x2; x3;..........; xn) son raíces de P(x)
TEOREMA DE CARDANO – VIETTE
Sea la ecuación polinomial:
P(x) = a0 xn
+ a1xn – 1
+ a2xn – 2
+...+ an – 1x + an = 0
a0 ≠ 0. Cuyas raíces son: {x1; x2; x3;............; xn}
Se cumple las siguientes relaciones
• Suma de Raíces:
S1 = x1 + x2 + x3 + ............ + xn = –
0
1
a
a
• Suma de Productos Binarios:
S2 =x1x2+x1 x3 + x2 x3 +...... + xn -1 xn = –
0
2
a
a
• Suma de Productos Ternarios:
S3 = x1 x2 x3 + x1 x2 x4 + ...... + xn – 2 xn – 1 xn =
–
0
3
a
a
• Producto de Raíces:
Sn = x1 x2 x3 .............. xn – 1 xn =(-1)n
0
n
a
a
Ejemplo:
01.En: 4x4
+ 3x3
– 2x2
+ 3x – 1 = 0
Calcular:
4
3
S
S
02.En: 3x5
+ 10x12
- 2x10
- 25x5
+ 15 = 0
Calcular: S10
TEOREMAS SOBRE LA ECUACIÓN
POLINOMIAL
1. Toda ecuación polinomial de coeficientes
racionales y de grado n ≥ 2. Que tenga una
raíz de la forma: “a + b ”, donde:
a y b ∈ Q (b > 0) ∧ b ∈ I ; tendrá como
raíz necesariamente al número (a – b ).
2. Toda ecuación polinomial de coeficientes
racionales y de grado n ≥ 4: que tenga una
raíz de la forma ba + , donde: a y b ∈
Q+
∧ Iab,b,a ∈ . Tendrá como
raíces necesariamente a los números:
ba:ba:ba −−+−−
3. Toda ecuación polinomial de coeficientes
reales y de grado n ≥ 2 que tenga una raíz
compleja de la forma, a + bi. Donde a y b
∈ R (b ≠ 0). Tendrá necesariamente como
raíz al complejo conjugado de dicha raíz es
decir otra raíz será: a - bi
Observación:
Q: conjunto de los números racionales
I: conjuntos de los números irracionales
Ejemplos:
 En la siguiente ecuación:
07 23
)( =++−= baxxxP x .
a, b ∈ Q
Hallar (a + b) si su raíz es: 3 + 5
 Formar la ecuación de menor grado
posible sabiendo que una raíz es
35 + y además sus coeficientes
son racionales.
 Dadas la ecuación: x3
+ x2
+ mx + n = 0.
m, n ∈ R
Donde: 1 + 7 i es una de las raíces.
Hallar La suma de coeficientes de la
ecuación.
TRANSFORMACIONES DE ECUACIONES
Sea la ecuación polinomial:
:01
2
2
1
1 =+++++ −
−−
nn
nnn
o aaxaxaxaxa 
Con raíces: {x1; x2; x3;................; xn} entonces:
1. La ecuación de raíces aumentados o
disminuidos en un valor “k”, es decir con
raíces:
{ }kxkxkxkx n ±±±± ;;;; 321 
es:
( ) ( ) ( )1
1
1
=++++ −
−
nn
nn
o akxakxakxa 
Ejemplos:
 Halle la ecuación cuyas raíces son las de la
ecuación:
x2
– 2x – 8 = 0, pero aumentadas en 1.
La ecuación es: (x – 1)2
– 2(x – 1) – 8 = O
S4AL33B “El nuevo símbolo de una buena educación....” S4AL33B “El nuevo símbolo de una buena educación...."
01 02COLEGIO DE CIENCIAS LORD KELVIN 4to Año Secundaria ALGEBRA 4to Año Secundaria
 Encuentre la ecuación cuyas raíces son los de
la ecuación x3
– 2x2
+ x – 5 = 0 disminuidas
en 2.
La ecuación es:
(x + 2 )3
- 2(x + 2)2
+ (x + 2) - 5 = 0.
Efectuando se obtiene: x3
+ 4x2
+ 5x – 3 = 0.
También se puede usar el siguiente método:
x = 2
1 - 2 1 - 5
↓ 2 0 2
x = 2
1 0 1 - 3
↓ 2 4
x = 2
1 2 5
↓ 2
1 4
Luego la ecuación es:
03x5x4x 23 =−++
• Encontrar la ecuación cuyas raíces son las de
la ecuación: x5
– 3x3
+ 2x2
+ 1 = 0,
disminuidas en 1.
2. La ecuación de raíces multiplicadas por un
valor “k” (k ≠ 0); es decir con raíces:
0a
k
x
a
k
x
a
k
x
a n1n
1n
1
n
o =+





++





+





−
−

O también:
0kaxkaxkaxa n
n
2n2
2
1n1
1
n
o =++++ −− 
Ejemplos:
• Encuentre la ecuación, cuyas raíces son las
de la ecuación: x2
– x – 6 = 0. Multiplicadas
por 2
La ecuación es: x2
- 21
x - 22
. 6 = 0
024x2x 2 =−−
• Halle la ecuación cuyas raíces son las de la
ecuación: x3
+ 2x2
- 5 x 6 = 0 multiplicadas
por 3. La ecuación es:
x3
+2 . 31
x2
+ 5 . 32
x - 6 . 33
= 0
x3
+ 6x2
- 45x - 162 =O
3. La ecuación de raíces invertidas es decir con
raíces:






nXxxx
1
;;
1
;
1
;
1
321
 Es:
0axaxa o
1n
1n
n
n =+++ −
− 
Ejemplo:
Dada la ecuación: x3
- 5x2
+ 7x + 2 = 0.
De raíces {a; b; c} entonces la ecuación cuyas
raíces son:






cba
1
;
1
;
1
es 2x3
+ 7x2
-
5x + 1 = 0
TEOREMA DE BOLZANO
Dada la ecuación polinomial F(x) = 0. Donde F(x)
es una función continua definida en [a; b]
Si F(a). F (b) < 0. Entonces existe al menos una
solución real: x0 ∈ < a; b > / 0)( 0
=xF
F(b)
F(a) F
b
a x 0
x
y
PRÁCTICA DE CLASEPRÁCTICA DE CLASE
01. Sean: x1; x2; x3 raíces de la ecuación:
2x3
– x + 5 = 0
Calcular:
321
1
3
1 xxx
3x
1x
+
−
+
a) 1 b) 2 c) -2
d) -3/2 e) 4/3
02.Sean: a, b, y c raíces de la ecuación:
x3
– 4x2
+ 2x + 4 = 0
Calcular:
ab
c
ac
b
bc
a 222
++
a) 5 b) - 5 c) - 4
d) - 7 e) 2
03. En la ecuación: x3
- 63x + α = 0. Determinar
un valor de α para que una de las raíces sea el
doble de otra.
a) 162 b) 180 c) 400
d) 800 e) N.A.
04.En la ecuación polinomial:
P(x) = x3
+ (m + 2) x2
+ (m2
– 3) x + m2
+ 2 =
0
De raíces x1 ; x2 ; x3. Calcular el valor de “m”
de tal manera que la expresión: A =
2
3
2
2
2
1
xxx ++ tenga el máximo valor.
a) l b) 2 c) 3
d) 4 e) 5
05.Hallar la relación que debe existir entre los
coeficientes de la ecuación: ax3
+ bx2
+ cx + d
= 0: a ≠ 0 . Si una de sus raíces es el negativo
de la otra
a) ab = cd b) ac = bd c)ad=bc
d) a + b = c + d e) a+d=b+c
06.Sabiendo que: x = c es una raíz de la
ecuación:
ax5
+ (b-ac)x4
- bcx3
- bx2
-(a-bc)x + ac = 0:
(a>0)
¿Qué condición deben cumplir a; b y c para
que las otras raíces sean reales?
a) |b| ≥ a b) |b| ≤ a c) |b| ≥ 2a
d) |b| ≤ 2a e) 2c=a+ b
07.Indicar el menor valor que debe tener el grado
del polinomio P(x). Con coeficientes reales, tal
que:
(2 + 3 ) Sea una raíz simple, (3 + 2i) sea
una raíz de multiplicidad 2 y ( 3 + 2 )
sea una raíz triple.
a) 5 b) 6 c) 7
d) 8 e) 9
08.Hallar un polinomio mónico P(x) con
coeficientes enteros y de menor grado posible
una de cuyas raíces sea: 3
32 + .
Indicar la suma de los coeficientes de este
polinomio.
a) 34 b) 24 e) - 24
d) 62 e) - 34
EJERCICIOS PROPUESTOS Nº 05
01.Hallar el valor de “k” si las raíces de la
ecuación:
x3
- 9x2
+ kx - 24 = 0
Están en progresión aritmética.
a) 12 b) 13 c) 24
d) 26 e) 28
02. Sea el polinomio: F(x) = x3
+ 3x2
– 9
S4AL33B “El nuevo símbolo de una buena educación....” S4AL33B “El nuevo símbolo de una buena educación...."
01 02COLEGIO DE CIENCIAS LORD KELVIN 4to Año Secundaria ALGEBRA 4to Año Secundaria
Además: F(m) = F(n) = F(p) = 0
Calcular: 





++
mn
p
mp
n
np
m
F
a) - 5 b) - 1 c) 2
d) - 2 e) 4
03.Si: (2 + i) es una raíz de multiplicidad dos del
siguiente polinomio: P(x) = x5
+ ax4
+ bx3
+
cx2
+ dx + 25
Hallar: a + b + c + d. Además: a ; b ; c ; d ∈
R.
a) 17 b) 18 c) 19
d) -18 e) -17
04. La ecuación: x4
– 12x – 5 = 0. Contiene 2
raíces cuya suma es 2. Calcular la suma de las
inversas de las otras dos.
a) 0,2 b) 0,4 c) - 0,2
d) - 0,4 e) 5
05. Sea la ecuación polinomial:
P(x) = ax3
+ x2
+ x + b = 0: a ≠ 0
Determinar los valores de “a” de modo que
P(x) admita una raíz real “r” de multiplicidad
2.
a) { }4
3
1
; −−∞−∈α
b)
3
1
; −∞−∈α
c)
3
1
;∞−∈α
d) { }0
3
1
; −∞−∈α
e) α ∈ R
06.Si la ecuación: x4
+ mx3
+ 2x + n = 0 m ∧ n ∈
R; admite una raíz triple. Hallar: m2
+ n3
a) 3 b) 4 c) 5
d) - 3 e) -1
07.Se sabe que: x1 ; x2 y x3 son las raíces de la
ecuación. x3
– x2
– 1 = 0. Encontrar una
nueva ecuación cuyas raíces son: x1 + x2 ; x2 +
x3 ; x3 + x1
a) 01yy2y 23 =−+−
b) 01yy2y 23 =++−
c) 01yyy 23 =−−−
d) 01yy2y 23 =+−−
e) 01yy2y 23 =−+−
08. ¿Cuál será la ecuación cúbica cuyas raíces
sean el doble de los recíprocos de cada una de
las raíces de la ecuación polinomial?
Ax3
– Bx + C = 0 ; C ≠ 0
a) Cx3
- Bx + A = 0
b) Cx3
+ 2Bx2
+ 4A = 0
c) Cx3
+ 2Bx2
– 4A = 0
d) Cx3
– 2Bx2
+ 8A = 0
e) Ax3
– 2Bx + 4C =O
09.El producto de los coeficientes de la función
polinomial de menor grado que pasa por los
puntos: (0; 0); (1; 1) ; (2; 0) y (3; -1) es:
a) -15/4 b) -14/9 c) 5/9
d) -15/9 e) -16/9
10. Sabiendo que: a b y c son raíces de la
ecuación:
x3
- 7x2
+ 5x + 6 = 0
Calcular:
M = (a+b-c)-1
+(b+c- a)-1
+ (c + a - b)-1
a) 31/55 b) 9/55 c) 7/155
d) 29/155 e) 27/55
11.Si la ecuación: x5
- 10a3
x2
+ b4
x + c5
= 0 tiene
3 raíces iguales. Hallar el valor de: ab4
- 9a5
a) c b) - c5
c) 0
d) c2
e) 1
12.Encontrar un polinomio mónico en "x" de
coeficientes en Z que acepte a 33
32 −
como raíz. Hallar la suma de coeficientes de
dicho polinomio.
a) 165 b) 168 c) 170
d) 174 e) 162
13.Formar la ecuación de menor grado posible
con coeficientes racionales, en la que una de
sus raíces sea. 2i3 +
a) x4
– 2x2
+ 25 = 0 d) x4
+2x2
-25=0
b) x4
+ 2x2
+ 25 = 0 e) x4
+x2
+25=0
c) x4
+ 2x2
+ 5 = 0
14.Calcular la suma de las raíces de: x3
+2x2
=x –
1
a) 2 b) –2 c) 3
d) – 1 e) 1
15.Calcular el producto de las raíces de:
2x3
+ 6x2
= 5x + 8
a) –1 b) –2 c) 4
d) – 4 e) –6
16.Resolver: x3
+ 2x2
– 11x = 12. E indicar una
de sus raíces.
a) 1 b) 2 c) 3
d) 4 e) 6
TAREA DOMICILIARIATAREA DOMICILIARIA
01.Si:
( )23)(
1
1
−
=
x
F x y además a, b y c
son raíces de la ecuación: x3
- 3x - 1= 0.
Calcular S = F(a) + F(b) + F(c)
a) 1 b) 3 c) 1/3
d) 9 e) N.A.
02. Halle las raíces r1 , r2 , r3 , r4 de la ecuación:
4x4
- ax3
+ bx2
- cx + 5 = 0
a) 1/2 b) 1/4 c) 5/4
d) 1 e) N.a.
03. Halle las raíces r1 , r2 , r3 , r4 de la ecuación:
4x4
- ax3
+ bx2
- cx + 5 = 0
Sabiendo que son reales positivos y que:
1
8
r
5
r
4
r
2
r 4321 =+++
Indique el valor de: r4
a) 1/2 b) 1/4 c) 5/4
d) 1 e) 2
04.Determinar el polinomio P(x) de grado 7.
Sabiendo que:
I)Para: x = 3: P(x) =PI
(x) = PII
(x) = PIII
(x) =
0 y PIV
(x) ≠ 0
II) Para: x = - 2 : P(x) = 0 : PI
(x) ≠ 0
III) Para: x = 4 : P(x) = 0 : PI
(x) =0 : PII
(x) ≠ 0
IV) P(2) = - 32
Dar como respuesta el valor de P(5)
a) - 1 12 b) 224 e) 32
d) - 32 e) - 224
05.Sean a . b y c raíces de la ecuación:
x3
+ px + q = 0 (a, b, c diferentes) expresar en
términos de p y q a:
S4AL33B “El nuevo símbolo de una buena educación....” S4AL33B “El nuevo símbolo de una buena educación...."
01 02COLEGIO DE CIENCIAS LORD KELVIN 4to Año Secundaria ALGEBRA 4to Año Secundaria
M=(a -b)2
(b - c)2
(a - c)2
a) 23 q27p4 + b)
23 q27p4 −−
c) 43 q2p + d) 23 q9p +−
e) 23 q27p4 −
06. Sobre la ecuación:
P(x) = x5
+ ax4
+ bx3
+ cx2
+ dx + c = 0
Donde: 2a2
< 3b ∧ {a; b; c; d; e}⊂ R
Indicar verdadero (V) o falso (F)
I) Todas sus raíces son reales
II) Al menos dos raíces son complejas
III) Una raíz es real
a) VFF b) FFV c) FVF
d) FFF e) VVV
07. Si: P(x) = (x - 1)(x - 3)(x - 5)+(x - 2)(x - 4)
Indicar la alternativa más correcta:
a) Tiene 3 raíces reales
b) Tiene 3 raíces reales negativas
c) Tiene 3 raíces reales positivas
d) Tiene 2 raíces reales positivas y una es
negativa
e) N.A
08. Sea el polinomio: P(x) = x3
- 3x2
+ 5
Indicar si es verdadero o falso:
I. Sólo tiene una raíz real positiva
II. Tiene 2 raíces complejas
III. Tiene una raíz comprendida entre <-2; -
1>
IV. Tiene un mínimo absoluto en x= 2
a) VVVF b) VFVF c) VFFF
d) FVVF e) FFFV
09. Dada la función: P(x) = x4
+ 2x3
+ x2
- 8
Decir verdadero (V) o falso (F) en:
I) Tiene un mínimo relativo en x = -1
II) Tiene 2 raíces reales
III) Su menor raíz está ubicada en <-3;2>
a) VVF b) VFV c) VFF
d) VVV e) FFV
10. La única raíz real de: x5
+ x - 10 = 0 se
encuentran en:
a) < 3/2; 7/4 > b) < 7/4; 2 > c) <1;2>
d) <5/4; 3/2> e) < 1; 5/4 >
SOLUCIONARIO
Nº
Ejercicios Propuestos
01 02 03 04 05
01. C C C C D
02. C C B A A
03. B D E B D
04. D D B A D
05. A D E C D
06. A C A A A
07. E C B D B
08. A B D B D
09. D B B A E
10. A B B A D
11. C C E B
12. B A E E
13. D E A
14. D E B
15. A C C
16. C B C
17. A D
18. D B
19. B C
20. E D
GRUPO EDUCATIVO INTEGRAL
copyright 2003
S4AL33B “El nuevo símbolo de una buena educación....” S4AL33B “El nuevo símbolo de una buena educación...."

Más contenido relacionado

La actualidad más candente

Matematica
MatematicaMatematica
Matematica349juan
 
Algebra(4) 5° 1 b
Algebra(4) 5° 1 bAlgebra(4) 5° 1 b
Algebra(4) 5° 1 b349juan
 
solucionario del examen de álgebra
solucionario del examen de álgebrasolucionario del examen de álgebra
solucionario del examen de álgebraMarco Antonio
 
Algebra 4to...Ecuaciones e inecuaciones de 2º Grado
Algebra 4to...Ecuaciones e inecuaciones de 2º GradoAlgebra 4to...Ecuaciones e inecuaciones de 2º Grado
Algebra 4to...Ecuaciones e inecuaciones de 2º GradoCamilo Solis Yanac
 
Ecuacion cuadrática
Ecuacion cuadráticaEcuacion cuadrática
Ecuacion cuadráticaπ -
 
Guía Productos y Cocientes Notables
Guía Productos y Cocientes NotablesGuía Productos y Cocientes Notables
Guía Productos y Cocientes Notablesmatbasuts1
 
2 ecuaciones e inecuaciones con valor absoluto
2 ecuaciones e inecuaciones con valor absoluto2 ecuaciones e inecuaciones con valor absoluto
2 ecuaciones e inecuaciones con valor absolutoGino León
 
Factorización de polinomios
Factorización de polinomiosFactorización de polinomios
Factorización de polinomiosChristiam3000
 
Clases de algebra 3°
Clases de algebra 3°Clases de algebra 3°
Clases de algebra 3°jaguinaga
 
Algebra pre ecuacion cuadratica (resueltos)
Algebra pre ecuacion cuadratica (resueltos)Algebra pre ecuacion cuadratica (resueltos)
Algebra pre ecuacion cuadratica (resueltos)Lukas Gallardo
 
Algebra 5° 3 b
Algebra 5° 3 bAlgebra 5° 3 b
Algebra 5° 3 b349juan
 
Algebra 3 expresiones algebraicas
Algebra 3 expresiones algebraicasAlgebra 3 expresiones algebraicas
Algebra 3 expresiones algebraicascmcoaquira
 
Multiplicación de-monomios-para-segundo-de-secundaria
Multiplicación de-monomios-para-segundo-de-secundariaMultiplicación de-monomios-para-segundo-de-secundaria
Multiplicación de-monomios-para-segundo-de-secundariaAnaliGianellaCapchaM
 
Matematica
MatematicaMatematica
Matematica349juan
 

La actualidad más candente (20)

Matematica
MatematicaMatematica
Matematica
 
Algebra(4) 5° 1 b
Algebra(4) 5° 1 bAlgebra(4) 5° 1 b
Algebra(4) 5° 1 b
 
solucionario del examen de álgebra
solucionario del examen de álgebrasolucionario del examen de álgebra
solucionario del examen de álgebra
 
Algebra 4to...Ecuaciones e inecuaciones de 2º Grado
Algebra 4to...Ecuaciones e inecuaciones de 2º GradoAlgebra 4to...Ecuaciones e inecuaciones de 2º Grado
Algebra 4to...Ecuaciones e inecuaciones de 2º Grado
 
2014 iii 09 factorización
2014 iii 09 factorización2014 iii 09 factorización
2014 iii 09 factorización
 
2014 iii 06 teorema de resto 1
2014 iii 06 teorema de resto 12014 iii 06 teorema de resto 1
2014 iii 06 teorema de resto 1
 
Ecuacion cuadrática
Ecuacion cuadráticaEcuacion cuadrática
Ecuacion cuadrática
 
Guía Productos y Cocientes Notables
Guía Productos y Cocientes NotablesGuía Productos y Cocientes Notables
Guía Productos y Cocientes Notables
 
Metodo de Horner
Metodo de HornerMetodo de Horner
Metodo de Horner
 
Sintitul 3
Sintitul 3Sintitul 3
Sintitul 3
 
2 ecuaciones e inecuaciones con valor absoluto
2 ecuaciones e inecuaciones con valor absoluto2 ecuaciones e inecuaciones con valor absoluto
2 ecuaciones e inecuaciones con valor absoluto
 
Factorización de polinomios
Factorización de polinomiosFactorización de polinomios
Factorización de polinomios
 
Clases de algebra 3°
Clases de algebra 3°Clases de algebra 3°
Clases de algebra 3°
 
Algebra pre ecuacion cuadratica (resueltos)
Algebra pre ecuacion cuadratica (resueltos)Algebra pre ecuacion cuadratica (resueltos)
Algebra pre ecuacion cuadratica (resueltos)
 
Algebra 5° 3 b
Algebra 5° 3 bAlgebra 5° 3 b
Algebra 5° 3 b
 
Divisibilidad algebraica
Divisibilidad algebraicaDivisibilidad algebraica
Divisibilidad algebraica
 
Expresiones algebráicas
Expresiones algebráicasExpresiones algebráicas
Expresiones algebráicas
 
Algebra 3 expresiones algebraicas
Algebra 3 expresiones algebraicasAlgebra 3 expresiones algebraicas
Algebra 3 expresiones algebraicas
 
Multiplicación de-monomios-para-segundo-de-secundaria
Multiplicación de-monomios-para-segundo-de-secundariaMultiplicación de-monomios-para-segundo-de-secundaria
Multiplicación de-monomios-para-segundo-de-secundaria
 
Matematica
MatematicaMatematica
Matematica
 

Destacado

Tema 01 relaciones en ir
Tema 01 relaciones en irTema 01 relaciones en ir
Tema 01 relaciones en ir349juan
 
Geometria(2 parte)
Geometria(2 parte)Geometria(2 parte)
Geometria(2 parte)349juan
 
Tema 04 función logaritmica
Tema 04 función logaritmicaTema 04 función logaritmica
Tema 04 función logaritmica349juan
 
Geometria
GeometriaGeometria
Geometria349juan
 
Tema 02 funciones en ir
Tema 02 funciones en irTema 02 funciones en ir
Tema 02 funciones en ir349juan
 
Tema 03 grafico de funciones en ir
Tema 03 grafico de funciones en irTema 03 grafico de funciones en ir
Tema 03 grafico de funciones en ir349juan
 
Rm (parte ii)
Rm (parte ii)Rm (parte ii)
Rm (parte ii)349juan
 
Aritmetica
AritmeticaAritmetica
Aritmetica349juan
 
Raz matematico 2
Raz matematico 2Raz matematico 2
Raz matematico 2349juan
 
Geometria(3 parte) 4° 1 b
Geometria(3 parte) 4° 1 bGeometria(3 parte) 4° 1 b
Geometria(3 parte) 4° 1 b349juan
 
Geometria
GeometriaGeometria
Geometria349juan
 
Raz. matematico
Raz. matematicoRaz. matematico
Raz. matematico349juan
 
Rm2 4° 1 b
Rm2 4° 1 bRm2 4° 1 b
Rm2 4° 1 b349juan
 
Geometria(2 parte) 4° 1 b
Geometria(2 parte) 4° 1 bGeometria(2 parte) 4° 1 b
Geometria(2 parte) 4° 1 b349juan
 

Destacado (20)

Tema 01 relaciones en ir
Tema 01 relaciones en irTema 01 relaciones en ir
Tema 01 relaciones en ir
 
Algebra de 1º sec
Algebra de 1º secAlgebra de 1º sec
Algebra de 1º sec
 
Operaciones con conjuntos
Operaciones con conjuntosOperaciones con conjuntos
Operaciones con conjuntos
 
Teoria de conjuntos
Teoria de conjuntosTeoria de conjuntos
Teoria de conjuntos
 
Geometria(2 parte)
Geometria(2 parte)Geometria(2 parte)
Geometria(2 parte)
 
Tema 04 función logaritmica
Tema 04 función logaritmicaTema 04 función logaritmica
Tema 04 función logaritmica
 
Algebra
AlgebraAlgebra
Algebra
 
Geometria
GeometriaGeometria
Geometria
 
Tema 02 funciones en ir
Tema 02 funciones en irTema 02 funciones en ir
Tema 02 funciones en ir
 
Tema 03 grafico de funciones en ir
Tema 03 grafico de funciones en irTema 03 grafico de funciones en ir
Tema 03 grafico de funciones en ir
 
Rm (parte ii)
Rm (parte ii)Rm (parte ii)
Rm (parte ii)
 
Algebra
AlgebraAlgebra
Algebra
 
Aritmetica
AritmeticaAritmetica
Aritmetica
 
Raz matematico 2
Raz matematico 2Raz matematico 2
Raz matematico 2
 
Geometria(3 parte) 4° 1 b
Geometria(3 parte) 4° 1 bGeometria(3 parte) 4° 1 b
Geometria(3 parte) 4° 1 b
 
Geometria
GeometriaGeometria
Geometria
 
Algebra
AlgebraAlgebra
Algebra
 
Raz. matematico
Raz. matematicoRaz. matematico
Raz. matematico
 
Rm2 4° 1 b
Rm2 4° 1 bRm2 4° 1 b
Rm2 4° 1 b
 
Geometria(2 parte) 4° 1 b
Geometria(2 parte) 4° 1 bGeometria(2 parte) 4° 1 b
Geometria(2 parte) 4° 1 b
 

Similar a Algebra 4° 3 b

República bolivariana de venezuela
República bolivariana de venezuelaRepública bolivariana de venezuela
República bolivariana de venezuelakatherin yanez
 
Números complejos
Números complejos Números complejos
Números complejos ITSLP
 
Unidad I Números Complejos.pptx
Unidad I Números Complejos.pptxUnidad I Números Complejos.pptx
Unidad I Números Complejos.pptxCsarTrianaFlores
 
4) TEORÍA - Conjunto de Números Complejos .pdf
4) TEORÍA - Conjunto de Números Complejos .pdf4) TEORÍA - Conjunto de Números Complejos .pdf
4) TEORÍA - Conjunto de Números Complejos .pdfSoloMel1
 
Capitulo 6 nf
Capitulo 6 nfCapitulo 6 nf
Capitulo 6 nfBleakness
 
Numeros complejos
Numeros complejosNumeros complejos
Numeros complejosfgonz02
 
Ecuaciones cuadráticas
Ecuaciones cuadráticasEcuaciones cuadráticas
Ecuaciones cuadráticasRosa E Padilla
 
Teoria numeros complejos
Teoria numeros complejosTeoria numeros complejos
Teoria numeros complejosbelplater
 
Números complejos.pdf
Números complejos.pdfNúmeros complejos.pdf
Números complejos.pdfDamyVale
 
Los números complejos
Los números complejosLos números complejos
Los números complejoscsoguero
 
Álgebra 1. Actividades - Intelectum.pdf
Álgebra 1. Actividades - Intelectum.pdfÁlgebra 1. Actividades - Intelectum.pdf
Álgebra 1. Actividades - Intelectum.pdfJL Veliz
 
Tareas números complejos
Tareas números complejosTareas números complejos
Tareas números complejosSpacetoshare
 

Similar a Algebra 4° 3 b (20)

República bolivariana de venezuela
República bolivariana de venezuelaRepública bolivariana de venezuela
República bolivariana de venezuela
 
Números complejos
Números complejos Números complejos
Números complejos
 
Unidad I Números Complejos.pptx
Unidad I Números Complejos.pptxUnidad I Números Complejos.pptx
Unidad I Números Complejos.pptx
 
UNIDAD 1 NUMEROS COMPLEJOS
UNIDAD 1 NUMEROS COMPLEJOSUNIDAD 1 NUMEROS COMPLEJOS
UNIDAD 1 NUMEROS COMPLEJOS
 
4) TEORÍA - Conjunto de Números Complejos .pdf
4) TEORÍA - Conjunto de Números Complejos .pdf4) TEORÍA - Conjunto de Números Complejos .pdf
4) TEORÍA - Conjunto de Números Complejos .pdf
 
Capitulo 6 nf
Capitulo 6 nfCapitulo 6 nf
Capitulo 6 nf
 
Numeros complejos
Numeros complejosNumeros complejos
Numeros complejos
 
Ecuaciones cuadráticas
Ecuaciones cuadráticasEcuaciones cuadráticas
Ecuaciones cuadráticas
 
Teoria numeros complejos
Teoria numeros complejosTeoria numeros complejos
Teoria numeros complejos
 
Números complejos.pdf
Números complejos.pdfNúmeros complejos.pdf
Números complejos.pdf
 
Los números complejos
Los números complejosLos números complejos
Los números complejos
 
Álgebra 1. Actividades - Intelectum.pdf
Álgebra 1. Actividades - Intelectum.pdfÁlgebra 1. Actividades - Intelectum.pdf
Álgebra 1. Actividades - Intelectum.pdf
 
Números complejos
Números complejosNúmeros complejos
Números complejos
 
Numeros complejos
Numeros complejosNumeros complejos
Numeros complejos
 
Mat+I+T08+Números+complejos.pdf
Mat+I+T08+Números+complejos.pdfMat+I+T08+Números+complejos.pdf
Mat+I+T08+Números+complejos.pdf
 
Mat+I+T08+Números+complejos.pdf
Mat+I+T08+Números+complejos.pdfMat+I+T08+Números+complejos.pdf
Mat+I+T08+Números+complejos.pdf
 
Números imaginarios
Números imaginariosNúmeros imaginarios
Números imaginarios
 
guia complejos tercero.pdf
guia complejos tercero.pdfguia complejos tercero.pdf
guia complejos tercero.pdf
 
Tareas números complejos
Tareas números complejosTareas números complejos
Tareas números complejos
 
2 eso matematicas - ud01
2 eso matematicas - ud012 eso matematicas - ud01
2 eso matematicas - ud01
 

Más de 349juan

Taller de estrategias de comunicación y matemática
Taller de estrategias de comunicación y matemáticaTaller de estrategias de comunicación y matemática
Taller de estrategias de comunicación y matemática349juan
 
Taller de estrategias de comunicación y matemática en el marco de las rutas d...
Taller de estrategias de comunicación y matemática en el marco de las rutas d...Taller de estrategias de comunicación y matemática en el marco de las rutas d...
Taller de estrategias de comunicación y matemática en el marco de las rutas d...349juan
 
Razonamiento matemtico nivel primaria
Razonamiento matemtico nivel primariaRazonamiento matemtico nivel primaria
Razonamiento matemtico nivel primaria349juan
 
Procesos didácticos y pedagógicos de una sesión de matemática
Procesos didácticos y pedagógicos de una sesión de matemáticaProcesos didácticos y pedagógicos de una sesión de matemática
Procesos didácticos y pedagógicos de una sesión de matemática349juan
 
Manual de tutoria y orientacion
Manual de tutoria y orientacionManual de tutoria y orientacion
Manual de tutoria y orientacion349juan
 
Cartilla minedu
Cartilla mineduCartilla minedu
Cartilla minedu349juan
 
Aprendiendo a-resolver-conflictos-en-las-instituciones-educativas
Aprendiendo a-resolver-conflictos-en-las-instituciones-educativasAprendiendo a-resolver-conflictos-en-las-instituciones-educativas
Aprendiendo a-resolver-conflictos-en-las-instituciones-educativas349juan
 
Simulacro examen docente a 2015
Simulacro examen docente a 2015Simulacro examen docente a 2015
Simulacro examen docente a 2015349juan
 
Simulacro de examen docente 2015
Simulacro de examen docente 2015Simulacro de examen docente 2015
Simulacro de examen docente 2015349juan
 
6 rutas del aprendizaje roxana dias (1)
6  rutas del aprendizaje   roxana dias (1)6  rutas del aprendizaje   roxana dias (1)
6 rutas del aprendizaje roxana dias (1)349juan
 
5 marco buen desempeño docente fidel soria cuellar
5  marco buen desempeño docente   fidel soria cuellar5  marco buen desempeño docente   fidel soria cuellar
5 marco buen desempeño docente fidel soria cuellar349juan
 
4 dcn y rutas de aprendizaje oscar tinoco gómez
4  dcn y rutas de aprendizaje   oscar tinoco gómez4  dcn y rutas de aprendizaje   oscar tinoco gómez
4 dcn y rutas de aprendizaje oscar tinoco gómez349juan
 
3 mapas de progreso gelvert estrada advíncula
3  mapas de progreso   gelvert estrada advíncula3  mapas de progreso   gelvert estrada advíncula
3 mapas de progreso gelvert estrada advíncula349juan
 
2 pen y marco curricular mauricio acevedo carrillo
2  pen y marco curricular   mauricio acevedo carrillo2  pen y marco curricular   mauricio acevedo carrillo
2 pen y marco curricular mauricio acevedo carrillo349juan
 
1 enfoque por competencias antuanet chirinos mendoza
1  enfoque por competencias   antuanet chirinos mendoza1  enfoque por competencias   antuanet chirinos mendoza
1 enfoque por competencias antuanet chirinos mendoza349juan
 
0 rutas emotivo afectivas - tania bornaz
0  rutas emotivo   afectivas - tania bornaz0  rutas emotivo   afectivas - tania bornaz
0 rutas emotivo afectivas - tania bornaz349juan
 
Orientaciones ebr
Orientaciones ebrOrientaciones ebr
Orientaciones ebr349juan
 
Formato de sesión de aprendizaje
Formato de sesión de aprendizajeFormato de sesión de aprendizaje
Formato de sesión de aprendizaje349juan
 
Fasciculo secundaria-matematica-vii
Fasciculo secundaria-matematica-viiFasciculo secundaria-matematica-vii
Fasciculo secundaria-matematica-vii349juan
 

Más de 349juan (20)

Taller de estrategias de comunicación y matemática
Taller de estrategias de comunicación y matemáticaTaller de estrategias de comunicación y matemática
Taller de estrategias de comunicación y matemática
 
Taller de estrategias de comunicación y matemática en el marco de las rutas d...
Taller de estrategias de comunicación y matemática en el marco de las rutas d...Taller de estrategias de comunicación y matemática en el marco de las rutas d...
Taller de estrategias de comunicación y matemática en el marco de las rutas d...
 
Razonamiento matemtico nivel primaria
Razonamiento matemtico nivel primariaRazonamiento matemtico nivel primaria
Razonamiento matemtico nivel primaria
 
Procesos didácticos y pedagógicos de una sesión de matemática
Procesos didácticos y pedagógicos de una sesión de matemáticaProcesos didácticos y pedagógicos de una sesión de matemática
Procesos didácticos y pedagógicos de una sesión de matemática
 
Manual de tutoria y orientacion
Manual de tutoria y orientacionManual de tutoria y orientacion
Manual de tutoria y orientacion
 
Etd esc
Etd escEtd esc
Etd esc
 
Cartilla minedu
Cartilla mineduCartilla minedu
Cartilla minedu
 
Aprendiendo a-resolver-conflictos-en-las-instituciones-educativas
Aprendiendo a-resolver-conflictos-en-las-instituciones-educativasAprendiendo a-resolver-conflictos-en-las-instituciones-educativas
Aprendiendo a-resolver-conflictos-en-las-instituciones-educativas
 
Simulacro examen docente a 2015
Simulacro examen docente a 2015Simulacro examen docente a 2015
Simulacro examen docente a 2015
 
Simulacro de examen docente 2015
Simulacro de examen docente 2015Simulacro de examen docente 2015
Simulacro de examen docente 2015
 
6 rutas del aprendizaje roxana dias (1)
6  rutas del aprendizaje   roxana dias (1)6  rutas del aprendizaje   roxana dias (1)
6 rutas del aprendizaje roxana dias (1)
 
5 marco buen desempeño docente fidel soria cuellar
5  marco buen desempeño docente   fidel soria cuellar5  marco buen desempeño docente   fidel soria cuellar
5 marco buen desempeño docente fidel soria cuellar
 
4 dcn y rutas de aprendizaje oscar tinoco gómez
4  dcn y rutas de aprendizaje   oscar tinoco gómez4  dcn y rutas de aprendizaje   oscar tinoco gómez
4 dcn y rutas de aprendizaje oscar tinoco gómez
 
3 mapas de progreso gelvert estrada advíncula
3  mapas de progreso   gelvert estrada advíncula3  mapas de progreso   gelvert estrada advíncula
3 mapas de progreso gelvert estrada advíncula
 
2 pen y marco curricular mauricio acevedo carrillo
2  pen y marco curricular   mauricio acevedo carrillo2  pen y marco curricular   mauricio acevedo carrillo
2 pen y marco curricular mauricio acevedo carrillo
 
1 enfoque por competencias antuanet chirinos mendoza
1  enfoque por competencias   antuanet chirinos mendoza1  enfoque por competencias   antuanet chirinos mendoza
1 enfoque por competencias antuanet chirinos mendoza
 
0 rutas emotivo afectivas - tania bornaz
0  rutas emotivo   afectivas - tania bornaz0  rutas emotivo   afectivas - tania bornaz
0 rutas emotivo afectivas - tania bornaz
 
Orientaciones ebr
Orientaciones ebrOrientaciones ebr
Orientaciones ebr
 
Formato de sesión de aprendizaje
Formato de sesión de aprendizajeFormato de sesión de aprendizaje
Formato de sesión de aprendizaje
 
Fasciculo secundaria-matematica-vii
Fasciculo secundaria-matematica-viiFasciculo secundaria-matematica-vii
Fasciculo secundaria-matematica-vii
 

Último

Informatica Generalidades - Conceptos Básicos
Informatica Generalidades - Conceptos BásicosInformatica Generalidades - Conceptos Básicos
Informatica Generalidades - Conceptos BásicosCesarFernandez937857
 
DE LAS OLIMPIADAS GRIEGAS A LAS DEL MUNDO MODERNO.ppt
DE LAS OLIMPIADAS GRIEGAS A LAS DEL MUNDO MODERNO.pptDE LAS OLIMPIADAS GRIEGAS A LAS DEL MUNDO MODERNO.ppt
DE LAS OLIMPIADAS GRIEGAS A LAS DEL MUNDO MODERNO.pptELENA GALLARDO PAÚLS
 
La Función tecnológica del tutor.pptx
La  Función  tecnológica  del tutor.pptxLa  Función  tecnológica  del tutor.pptx
La Función tecnológica del tutor.pptxJunkotantik
 
DECÁGOLO DEL GENERAL ELOY ALFARO DELGADO
DECÁGOLO DEL GENERAL ELOY ALFARO DELGADODECÁGOLO DEL GENERAL ELOY ALFARO DELGADO
DECÁGOLO DEL GENERAL ELOY ALFARO DELGADOJosé Luis Palma
 
ACERTIJO DE LA BANDERA OLÍMPICA CON ECUACIONES DE LA CIRCUNFERENCIA. Por JAVI...
ACERTIJO DE LA BANDERA OLÍMPICA CON ECUACIONES DE LA CIRCUNFERENCIA. Por JAVI...ACERTIJO DE LA BANDERA OLÍMPICA CON ECUACIONES DE LA CIRCUNFERENCIA. Por JAVI...
ACERTIJO DE LA BANDERA OLÍMPICA CON ECUACIONES DE LA CIRCUNFERENCIA. Por JAVI...JAVIER SOLIS NOYOLA
 
cortes de luz abril 2024 en la provincia de tungurahua
cortes de luz abril 2024 en la provincia de tungurahuacortes de luz abril 2024 en la provincia de tungurahua
cortes de luz abril 2024 en la provincia de tungurahuaDANNYISAACCARVAJALGA
 
RETO MES DE ABRIL .............................docx
RETO MES DE ABRIL .............................docxRETO MES DE ABRIL .............................docx
RETO MES DE ABRIL .............................docxAna Fernandez
 
RAIZ CUADRADA Y CUBICA PARA NIÑOS DE PRIMARIA
RAIZ CUADRADA Y CUBICA PARA NIÑOS DE PRIMARIARAIZ CUADRADA Y CUBICA PARA NIÑOS DE PRIMARIA
RAIZ CUADRADA Y CUBICA PARA NIÑOS DE PRIMARIACarlos Campaña Montenegro
 
Caja de herramientas de inteligencia artificial para la academia y la investi...
Caja de herramientas de inteligencia artificial para la academia y la investi...Caja de herramientas de inteligencia artificial para la academia y la investi...
Caja de herramientas de inteligencia artificial para la academia y la investi...Lourdes Feria
 
PRIMER SEMESTRE 2024 ASAMBLEA DEPARTAMENTAL.pptx
PRIMER SEMESTRE 2024 ASAMBLEA DEPARTAMENTAL.pptxPRIMER SEMESTRE 2024 ASAMBLEA DEPARTAMENTAL.pptx
PRIMER SEMESTRE 2024 ASAMBLEA DEPARTAMENTAL.pptxinformacionasapespu
 
Clasificaciones, modalidades y tendencias de investigación educativa.
Clasificaciones, modalidades y tendencias de investigación educativa.Clasificaciones, modalidades y tendencias de investigación educativa.
Clasificaciones, modalidades y tendencias de investigación educativa.José Luis Palma
 
Planificacion Anual 4to Grado Educacion Primaria 2024 Ccesa007.pdf
Planificacion Anual 4to Grado Educacion Primaria   2024   Ccesa007.pdfPlanificacion Anual 4to Grado Educacion Primaria   2024   Ccesa007.pdf
Planificacion Anual 4to Grado Educacion Primaria 2024 Ccesa007.pdfDemetrio Ccesa Rayme
 
Neurociencias para Educadores NE24 Ccesa007.pdf
Neurociencias para Educadores  NE24  Ccesa007.pdfNeurociencias para Educadores  NE24  Ccesa007.pdf
Neurociencias para Educadores NE24 Ccesa007.pdfDemetrio Ccesa Rayme
 
OLIMPIADA DEL CONOCIMIENTO INFANTIL 2024.pptx
OLIMPIADA DEL CONOCIMIENTO INFANTIL 2024.pptxOLIMPIADA DEL CONOCIMIENTO INFANTIL 2024.pptx
OLIMPIADA DEL CONOCIMIENTO INFANTIL 2024.pptxjosetrinidadchavez
 
Plan Refuerzo Escolar 2024 para estudiantes con necesidades de Aprendizaje en...
Plan Refuerzo Escolar 2024 para estudiantes con necesidades de Aprendizaje en...Plan Refuerzo Escolar 2024 para estudiantes con necesidades de Aprendizaje en...
Plan Refuerzo Escolar 2024 para estudiantes con necesidades de Aprendizaje en...Carlos Muñoz
 
EXPANSIÓN ECONÓMICA DE OCCIDENTE LEÓN.pptx
EXPANSIÓN ECONÓMICA DE OCCIDENTE LEÓN.pptxEXPANSIÓN ECONÓMICA DE OCCIDENTE LEÓN.pptx
EXPANSIÓN ECONÓMICA DE OCCIDENTE LEÓN.pptxPryhaSalam
 
texto argumentativo, ejemplos y ejercicios prácticos
texto argumentativo, ejemplos y ejercicios prácticostexto argumentativo, ejemplos y ejercicios prácticos
texto argumentativo, ejemplos y ejercicios prácticosisabeltrejoros
 

Último (20)

Informatica Generalidades - Conceptos Básicos
Informatica Generalidades - Conceptos BásicosInformatica Generalidades - Conceptos Básicos
Informatica Generalidades - Conceptos Básicos
 
DE LAS OLIMPIADAS GRIEGAS A LAS DEL MUNDO MODERNO.ppt
DE LAS OLIMPIADAS GRIEGAS A LAS DEL MUNDO MODERNO.pptDE LAS OLIMPIADAS GRIEGAS A LAS DEL MUNDO MODERNO.ppt
DE LAS OLIMPIADAS GRIEGAS A LAS DEL MUNDO MODERNO.ppt
 
La Función tecnológica del tutor.pptx
La  Función  tecnológica  del tutor.pptxLa  Función  tecnológica  del tutor.pptx
La Función tecnológica del tutor.pptx
 
Sesión de clase: Defendamos la verdad.pdf
Sesión de clase: Defendamos la verdad.pdfSesión de clase: Defendamos la verdad.pdf
Sesión de clase: Defendamos la verdad.pdf
 
DECÁGOLO DEL GENERAL ELOY ALFARO DELGADO
DECÁGOLO DEL GENERAL ELOY ALFARO DELGADODECÁGOLO DEL GENERAL ELOY ALFARO DELGADO
DECÁGOLO DEL GENERAL ELOY ALFARO DELGADO
 
ACERTIJO DE LA BANDERA OLÍMPICA CON ECUACIONES DE LA CIRCUNFERENCIA. Por JAVI...
ACERTIJO DE LA BANDERA OLÍMPICA CON ECUACIONES DE LA CIRCUNFERENCIA. Por JAVI...ACERTIJO DE LA BANDERA OLÍMPICA CON ECUACIONES DE LA CIRCUNFERENCIA. Por JAVI...
ACERTIJO DE LA BANDERA OLÍMPICA CON ECUACIONES DE LA CIRCUNFERENCIA. Por JAVI...
 
cortes de luz abril 2024 en la provincia de tungurahua
cortes de luz abril 2024 en la provincia de tungurahuacortes de luz abril 2024 en la provincia de tungurahua
cortes de luz abril 2024 en la provincia de tungurahua
 
RETO MES DE ABRIL .............................docx
RETO MES DE ABRIL .............................docxRETO MES DE ABRIL .............................docx
RETO MES DE ABRIL .............................docx
 
Defendamos la verdad. La defensa es importante.
Defendamos la verdad. La defensa es importante.Defendamos la verdad. La defensa es importante.
Defendamos la verdad. La defensa es importante.
 
RAIZ CUADRADA Y CUBICA PARA NIÑOS DE PRIMARIA
RAIZ CUADRADA Y CUBICA PARA NIÑOS DE PRIMARIARAIZ CUADRADA Y CUBICA PARA NIÑOS DE PRIMARIA
RAIZ CUADRADA Y CUBICA PARA NIÑOS DE PRIMARIA
 
Caja de herramientas de inteligencia artificial para la academia y la investi...
Caja de herramientas de inteligencia artificial para la academia y la investi...Caja de herramientas de inteligencia artificial para la academia y la investi...
Caja de herramientas de inteligencia artificial para la academia y la investi...
 
PRIMER SEMESTRE 2024 ASAMBLEA DEPARTAMENTAL.pptx
PRIMER SEMESTRE 2024 ASAMBLEA DEPARTAMENTAL.pptxPRIMER SEMESTRE 2024 ASAMBLEA DEPARTAMENTAL.pptx
PRIMER SEMESTRE 2024 ASAMBLEA DEPARTAMENTAL.pptx
 
Clasificaciones, modalidades y tendencias de investigación educativa.
Clasificaciones, modalidades y tendencias de investigación educativa.Clasificaciones, modalidades y tendencias de investigación educativa.
Clasificaciones, modalidades y tendencias de investigación educativa.
 
Planificacion Anual 4to Grado Educacion Primaria 2024 Ccesa007.pdf
Planificacion Anual 4to Grado Educacion Primaria   2024   Ccesa007.pdfPlanificacion Anual 4to Grado Educacion Primaria   2024   Ccesa007.pdf
Planificacion Anual 4to Grado Educacion Primaria 2024 Ccesa007.pdf
 
Neurociencias para Educadores NE24 Ccesa007.pdf
Neurociencias para Educadores  NE24  Ccesa007.pdfNeurociencias para Educadores  NE24  Ccesa007.pdf
Neurociencias para Educadores NE24 Ccesa007.pdf
 
Presentacion Metodología de Enseñanza Multigrado
Presentacion Metodología de Enseñanza MultigradoPresentacion Metodología de Enseñanza Multigrado
Presentacion Metodología de Enseñanza Multigrado
 
OLIMPIADA DEL CONOCIMIENTO INFANTIL 2024.pptx
OLIMPIADA DEL CONOCIMIENTO INFANTIL 2024.pptxOLIMPIADA DEL CONOCIMIENTO INFANTIL 2024.pptx
OLIMPIADA DEL CONOCIMIENTO INFANTIL 2024.pptx
 
Plan Refuerzo Escolar 2024 para estudiantes con necesidades de Aprendizaje en...
Plan Refuerzo Escolar 2024 para estudiantes con necesidades de Aprendizaje en...Plan Refuerzo Escolar 2024 para estudiantes con necesidades de Aprendizaje en...
Plan Refuerzo Escolar 2024 para estudiantes con necesidades de Aprendizaje en...
 
EXPANSIÓN ECONÓMICA DE OCCIDENTE LEÓN.pptx
EXPANSIÓN ECONÓMICA DE OCCIDENTE LEÓN.pptxEXPANSIÓN ECONÓMICA DE OCCIDENTE LEÓN.pptx
EXPANSIÓN ECONÓMICA DE OCCIDENTE LEÓN.pptx
 
texto argumentativo, ejemplos y ejercicios prácticos
texto argumentativo, ejemplos y ejercicios prácticostexto argumentativo, ejemplos y ejercicios prácticos
texto argumentativo, ejemplos y ejercicios prácticos
 

Algebra 4° 3 b

  • 1. 01 02COLEGIO DE CIENCIAS LORD KELVIN 4to Año Secundaria ALGEBRA 4to Año Secundaria OBJETIVOS ESPECIFICOS: Define y reconoce a las cantidades imaginarias como componentes no reales de los números complejos. Opera con las potencias enteras de i. COMENTARIO PREVIO: El conjunto І de los números irracionales, junto con el conjunto Q de los números racionales constituyen el conjunto R de los números reales. Por los conocimientos previos que manejamos vimos que el campo numérico hasta ahora conocido necesitaba una nueva ampliación que permitiera hallar raíces pares de números negativos. Así, por ejemplo, no existe ningún número real que represente 4 4,1,8,3 −−−− . Estas raíces reciben el nombre cantidades imaginarias. Llamamos imaginarios a los números constituyentes de las componentes no reales de los números complejos. En este módulo que consta de dos sesiones trataremos de realizar un estudio formal y riguroso de este nuevo sistema numérico, en todo momento relacionaremos estos conceptos nuevos con los conocimientos antes conocidos. CONTENIDO TEÓRICO: CANTIDADES IMAGINARIAS Los números imaginarios se originan de la extracción de la raíz cuadrada a números negativos. Definición: Las cantidades imaginarias son aquellas que se obtienen por la extracción de raíces pares de números negativos. Ejemplo: 126 5;64;9 −−− ; son cantidades imaginarias. Toda expresión de la forma: n a2 , donde 2n es par y a es un número real negativo, es una cantidad imaginaria pura. Unidad imaginaria: Recibe este nombre el radical 1− , se le representa mediante el siguiente símbolo: i = 1− (notación de Gauss) y cumple i 2 = -1. 1− se tomará como referencia para medir todas las cantidades imaginarias puras. Operación básica de transformación: Sea a ∈ ℜ+ , tenemos: iaaaa =−=−=− 1)1( Toda raíz imaginaria puede expresarse como el producto de un número real por la unidad imaginaria. Ejemplo: i39 =− ; i55 =− ; i 3 2 9 4 =− Potencias enteras de la unidad imaginaria: De i1 = i se ha concluido que i2 = -1; conociendo esto podemos deducir todas las demás potencias de i. i 1 = i i 5 = i 4 . i 1 = i i 9 = i i 2 = -1 i 6 = i 4 . i 2 = -1 i 10 = -1 i 3 = - i i 7 = i 4 . i 3 = - i i 11 = - i i 4 = 1 i 8 = i 4 . i 4 = 1 i 12 = 1 Generalizando: i 4q + r = i r 0 ≤ r < 4 ; q ∈ Ζ Veamos los siguientes ejemplos: ♦ Sabemos que: i 4q = 1 ∀ q ∈ Ζ i 12 448 = 1 ; i 137 956 = 1; i -12 448 = 1 Recuerde que 4q es 0 4 ; ∀ q ∈ Ζ ♦ La unidad imaginaria i, no siempre estará afectado exactamente de un exponente 0 4 , podría presentarse: i 4q + r ; 0 ≤ r < 4; q ∈ Ζ i 15 767 = i 4 (3 941) + 3 = i 3 = - i i -135 = i 4 (-34) + 1 = i 1 = i Se concluye: i + i 2 + i 3 + i 4 = 0, esta relación podemos generalizarlo diciendo: la suma de cuatro potencias consecutivas cualesquiera de la unidad imaginaria es igual a cero. Es decir: i n + i n+1 + i n+2 + i n+3 = 0 ; ∀ n ∈ Ζ Ejemplos ♦ i 217 + i 218 + i 219 + i 220 = 0 ♦ i - 75 + i - 76 + i - 77 + i - 78 = 0 Resumen:  0 4 41 0 ∀=i positivo o negativo  0 4 4 0 ∀=+ rr ii positivo o negativo, r ∈ Ζ  0432 =+++ iiii ; en general:  Ζ∈∀=+++ +++ niiii nnnn ;0321 PRÁCTICA DE CLASE 1. Hallar: a) i 4 273 b) i 30 214 2. Siendo, i = 1− , dar el valor de: I. i –7 II. i –21 III. i-3 224 3. Simplificar: 37912137728249 12123 iiiii +−++ 4. Hallar el valor de: i 658 + i 527 – i 436 + i 247 5. Simplificar: 161514131211109 8765432 428428 824824 iiiiiiii iiiiiiii +++++++ +++++++ 6. Reduce a su mínima expresión: (i –233 – i –232 + i –231 -... – i –2 + i –1 – 1) 2 7. Simplificar: niniiii iniiii n n 4)18(...9753 8...432 4432 168642 −++++++ +++++ EJERCICIOS PROPUESTOS Nº 01 1. Hállese la parte real de efectuar: 2222 100321 ..... iiiiK ++++= a) 1 b) 0 c) 50 d) 100 e) 25 2. Efectuar: E = 3333 5555 2222 iii ++ a) i b) - i c) 1 d) - 1 e) N.A 3. Simplificar: 603593584540527 293302313321328 − + − + − − − + − ++++ = iiiii iiiii K a) 4 b) 3 c) 8 d) - 1 e) N.A S4AL33B “El nuevo símbolo de una buena educación....” S4AL33B “El nuevo símbolo de una buena educación...." III CANTIDADES
  • 2. 01 02COLEGIO DE CIENCIAS LORD KELVIN 4to Año Secundaria ALGEBRA 4to Año Secundaria 4. El valor simplificado de:               21 15 !776 !777 i i ,es: a) 0 b) 1 c) -1 d) i e) -i 5. Efectuar: 1 + i + i3 + i5 + i7 +…. + i79 a) 1 b) i c) 2i d) 32i e) i + 1 06.Simplificar: 32824255 800876353875175347 iiii −−− ++ +++ iii a) -2 b) 2 c) -2i d) 32i e) – i 07.Hallar el valor de: E = i2 + 2i4 + 3i6 + 4i8 +… + (2n – 1)i4n – 2 + 2n.i4n a) 1 b) ni c) 2ni d) 32i e) n 08.Siendo: 1−=i . Simplificar: 1 ... 325412 775432 −−+− +++++ = iiii iiiii G a) 1 b) 2 c) i d) 2i e) 3i 09.Reducir: 969798 KELVINWTIUNT iii ++ Siendo 1−=i a) 0 b) 1 c) i – 1 d) i e) – i 10.Calcular: S = i + i2 + i3 +...... + i100 .Donde: i2 = -1 a) 0 b) 1 c) 2 d) 3 e) 5 TAREA DOMICILIARIA 01.Simplificar: 196225361 36100324 −+−−− −−−+− 02.Calcular: a) i16 b) i10 c) i4n+3 (n entero) d) i43 03.Calcular: 367 5 i 04.Hallar: 3333 3 i 05.Calcular: 839 6 i 06.Hallar el valor de: E = i1! + i3! + i5! + i7! + … + i59! OBJETIVOS ESPECIFICOS:  Realiza un estudio formal de los números complejos y sus respectivas propiedades.  Aplica las propiedades antes estudiadas en la resolución de ejercicios que involucran números complejos. COMENTARIO PREVIO: El matemático francés Descartes fue el primero que llamó imaginarios a los números constituyentes de las componentes no reales de los números complejos. El matemático alemán Euler contribuyó notablemente a divulgar el uso de los números complejos, pero quién mayor auge dio a su utilización fue el matemático danés Wessel, que suministró una valiosa interpretación geométrica de los números complejos. CONTENIDO TEÓRICO: 01.NÚMEROS COMPLEJOS Son de la forma Z = a + bi, a ∈ IR, b ∈ IR se llama: a = Re(z) → parte real de Z b = Im(z) → parte imaginaria de Z Complejos conjugados Son aquellos que difieren únicamente en el signo de su parte imaginaria: 4 – 3i 4 + 3i Complejos Opuestos El opuesto de a + bi es –a - bi Complejos Nulos Aquel número que tiene su parte real y su parte imaginaria iguales a cero: 0 = 0 + 0i Igualdad de dos números complejos Dos números complejos son iguales si tienen iguales sus partes reales y sus partes imaginarias: a + bi = c + di ⇒ a = c y b = d Módulo o norma de un complejo Se define por medio de la siguiente relación: Ejemplo: 5)4()3(43 22 =−+−=−−= ir 02.OPERACIONES CON NÚMEROS COMPLEJOS AdiciónAdición: (a + bi) + (c + di) = (a + c) + (b + d)i (2 + 7i) + (-7 - 3i) = (2 - 7) + (7 - 3)i = -5 + 4i SustracciónSustracción (a + bi) - (c + di) = (a - c) + (b – d)i iii 1 1 9 1 5 2 )43( 9 1 4 5 2 3 +−=      −+−=      +−      + MultiplicaciónMultiplicación (a + bi) (c + di) = (a c-bd) + (ad +bc)i (2-5i)(3-7i) = 6 - 14i - 15i + 35i2 = -29 - 29i PotenciaciónPotenciación Zn = Z.Z.Z.Z.Z.... Z , ∀ Z∈C, n∈N n veces División de números complejosDivisión de números complejos S4AL33B “El nuevo símbolo de una buena educación....” S4AL33B “El nuevo símbolo de una buena educación...." 22 baibaP +=+= C ⇔ C NÚMEROS
  • 3. 01 02 Observación: Cuando el índice es 2 (n = 2) podría tomarse en cuenta la transformación de radicales dobles en simples, paraalgunoscasos. COLEGIO DE CIENCIAS LORD KELVIN 4to Año Secundaria ALGEBRA 4to Año Secundaria 22 )()( ))(( ))(( dc iadbcbdac idcidc idciba idc iba + −++ = −+ −+ = + + Efectuar: i ii ii ii i i 25 26 25 7 34 15268 )34)(34( )34)(52( 34 52 22 2 − − = + +− = −+ −− = + − Raíz cuadrada de números complejosRaíz cuadrada de números complejos La radicación de un número complejo arrojará tantas raíces como lo indique el índice del signo radical. Es decir: dado Z = a + bi para calcular las raíces enésimas o raíces de orden “n” de Z (n∈ Ν, n ≥ 2), se establece lo siguiente: yixbian +=+ Donde: a, b y n son datos, x e y tendrán que calcularse (x, y ∈ ℜ); para esto se tendrá que elevar ambos miembros a la “n” y desarrollar el segundo miembro por fórmula del binomio de Newton. Se recomienda esto cuando “n” toma valores pequeños, en caso contrario téngase en cuenta la fórmula de Moivre. Ejemplo: Calcular las raíces cuadradas de 21 – 20i Resolución Establecemos la igualdad: iyxi +=−2021 Elevando al cuadrado: 21 - 20i = (x 2 – y 2 ) + 2xy i Por igualdad de números complejos: ♦ x 2 – y 2 = 21 ....(I) ♦ 2xy = -20 ....(II) (I) 2 + (II) 2 : (x2 – y2 ) 2 + 4x2 y2 = (21)2 + (-20)2 (x2 + y2 )2 = 841→ x2 + y2 = 29 .......(III) (I)+(III): 2x2 = 50 → x2 = 25 → x = 5 ó x = –5 (III)–(I): 2y2 = 8→ y2 = 4→ y = 2 ó y = -2 De (II): x e y tienen signos opuestos, luego: x = 5 ; y = -2 ó x = -5 ; y = 2 )25(2021 ii −±=−∴ Otra forma: Aplicando transformaciones de radicales dobles en simples. Veamos: 1002211102212021 −−=−−=− xi ( ) ( )ii 254252021 −±=−−±=− 03.REPRESENTACIÓN GRÁFICA DE UN NÚMERO COMPLEJO La representación gráfica de un número complejo se realiza en un sistema de ejes coordenadas denominado Diagrama de Argand, mediante un punto cuyas coordenadas serán las componentes de un complejo, al punto se le denomina afijo del número complejo. Ejemplo: Representar gráficamente los siguientes números: Número complejo Afijo del complejo 4 + 3i (4 ; 3) 2 - 4i (2 ;-4) -5 + 2i (-5; 2) 7 (7 ; 0) -3i (0 ;-3) eje imaginario eje real (4,3) (-5,2) (2,-4) 04.FORMA POLAR O TRIGONOMÉTRICA RELACIONES FUNDAMENTALES e je im a g in a rio e je re a l a + b i = (a , b ) b p a 0 1) 22 bar += módulo 2) θ = arc tg ( a b ) argumento (0 ≤ θ < 2π), a ≠ 0 3) a = r cos θ 4) b = r sen θ FORMA POLAR O TRIGONOMÉTRICA DE UN COMPLEJO Sea Cos θ + i sen θ = Cis θ Luego: a + b i = r Cis θ Ejemplo: 1) Cis 60° = cos 60° + i sen 60° = 2 1 + 2 3 i 2) Expresar en forma polar – 4 – 4 3 i 8)34()4( 22 =−+−=r °== − − = 2403 4 34 tgarctgarcθ °=−− 2408344 cisi e je im a g in a rio e je re a l 2 4 0 ° 6 0 ° 05.OPERACIONES CON COMPLEJOS EN FORMA POLAR 1°)Multiplicación S4AL33B “El nuevo símbolo de una buena educación....” S4AL33B “El nuevo símbolo de una buena educación...." a + b i = r (cos θ + i sen θ) (25) + (-4) (25) (-4)
  • 4. 01 02COLEGIO DE CIENCIAS LORD KELVIN 4to Año Secundaria ALGEBRA 4to Año Secundaria (r1 Cis θ1) (r2 Cis θ2) = r1 r2 Cis (θ1 + θ2) 2°)División 3°)Potenciación (r cis θ)n = rn cis (nθ) Nota: Si r = 1 tenemos la fórmula de Moivre (cis θ)n = cis (nθ) 4°)Radicación Donde: k = 0 ; 1 ; 2 ; ... (n-1) Ejemplos: 1) Hallar las raíces cúbicas de la unidad )120( 3 360 1011 333 kCis k Ciscis °= ° == K = 0 ⇒ W1=Cis 0 = 1 K = 1 ⇒ W2=Cis 120° = i 2 3 2 1 + − K = 2 ⇒ W3=Cis 240° = 2 1− - 2 3 i 2) Resolver x4 + 1 = 0 4 360180 118011 k Ciscisx °+° ==−= k = 0, 1, 2, 3 K = 0 ⇒ W1=Cis 45° = 2 2 + 2 2 i K = 1 ⇒ W2=Cis 135° = 2 2− + 2 2 i K = 2 ⇒ W3=Cis 225° = 2 2− - 2 2 i K = 3 ⇒ W4 = Cis 315° = 2 2 - 2 2 i PRÁCTICA DE CLASE 01. Efectuar: i i i i i i K − + − + − + − + − + = 1 1 1 1 1 1 1 1 1 1 a) 1 - i b) 1 c) 0 d) 1 + i e) i 02. Si la raíz cuadrada del número complejo: 1 + i es: x + yi .Hállese el valor de: x y y x M −= a) 0 b) 1 c) 2 d) 3 e) 4 03. Si se cumple: iyixn +=++ 148 Calcular el valor de “x”. a) 24n b) 24n+2 c) 42n+1 d) -24n+2 e) -22n+1 04. Calcular: ( )∑= + +++ n n nn xxx 1 4644 2652 Para: x = −1 a) 1 b) 5 c) 5(n - 1) d) 5n e) 5(n+1) 05. Simplificar: ii i 2 13 2 3 125 37       +− a) 1 b) 2 c) 3 d) 4 e) N.A. 06. Si: 2 31 −+− =w . Calcular el Valor numérico de: K = (5 + 7w + 7w2 )12 a) 64 b) 512 c) 1024 d) 2048 e) 2048 07. Siendo “W” una de las raíces cúbicas de la unidad tal que: w ≠ 1, calcule: S = (a+aw+w2 )4 (1+aw+aw2 )4 (a+w+ aw2 )4 a) a12 b) (a + 1)12 c) -(a - 1)12 d) (a -1)12 e) N.A. 08.Si tenemos que: 52 5 9 1 2 21 3 i i i xii = − + + + + Hállese el módulo de “Z”: + ∈+= RxixZ / a) 1 b) 2 c) 3 d) 4 e) N.A. 09. Hállese “Z” de: 1 8 4 ; 3 5 8 12 = − − = − − Z Z iZ Z a) 6 + 17i b) 4 + 9i c) 6 + 19i d) 6 + 8i e) a y d 10. Si “z” es un complejo tal que 5=z .Halle: 22 11 zzK −++= a) 52 b) 50 c) 48 d) 2 e) 32 11. La suma de los siguientes complejos: Z1 = 2 + (y + 2)i Z2 = y + 4 - 3yi / y ∈ R Origina un número real, calcularlo a) 7 b) 0 c) 5 d) 2 e) 6 EJERCICIOS PROPUESTOS Nº 02 01.Efectuar: i i ii Z − + + − + + = 1 1 1 1 1 1 a) i b) -i c) 1 + i d) 1 – i e) 0 02.Calcular: ( ) 9 9 1 1 i i E + + = a) 8 b) 12 c) 16 d) 4 e) 2 S4AL33B “El nuevo símbolo de una buena educación....” S4AL33B “El nuevo símbolo de una buena educación...." )( 21 2 1 22 11 θθ θ θ −= Cis r r Cisr Cisr n k Cisrcisr )360( °+ = θ θ
  • 5. 01 02COLEGIO DE CIENCIAS LORD KELVIN 4to Año Secundaria ALGEBRA 4to Año Secundaria 03.Calcular el valor de: ( ) ( ) ( )6 23 1 11 i ii E − +−+ = a) i b) 3i c) 4i d) i/4 e) 2i 04.Efectuar: ( ) ( ) 3 311 2 − ++ i ii Donde: 1−=i , es igual a: a) 1 – 3i b) -2 c) 10 d) 2 e) 101 05.Calcular el menor valor de “n” que verifica: (1 + i)n = 32 i a) 5 b) 4 c) 8 d) 10 e) 12 06.Simplificar: 27233581246032442541 33746542152 iiiii iiiii E −+−+ ++++ = a) i b) -i c) 1 d) -1 e) 3 07.Si se cumple: (1 + i)2 + (1 + i)4 + (1 + i)6 + (1 + i)8 = m + ni m, n ∈ IR. Hallar: m . n a) 70 b) 72 c) -72 d) 40 e) -54 08.Hallar “n” en: [(1 + i)7 + (1 – i)7 ]n = 26! a) 120 b) 180 c) 240 d) 300 e) 360 09.Simplificar: 42 432 1 1 1 1 1 1 1 1 ii iiii i i i i E +− +++ + − + − + − + += a) 1 – i b) 1 + i c) i d) 2 i e) 0 10.Reducir: Z = (1+ i)3 + (1+ i2 )3 + (1+ i3 )3 +…+ (1+ i200 )3 a) 100 b) 200 c) 300 d) 400 e) 500 11.Sea: Z = x + (y – 2) i ; 1−=i W = 5 + (x + 5) i Calcular x + y para que Z y W sean conjugados. a) 13 b) –8 c) –3 d) 1 e) 5 12. Calcular: 1;5 201424 1 −= − + i i ii a) –i b) i c) 2 d) 1 e) 0 TAREA DOMICILIARIA 01.Calcular: R = 4 1 1 1 1     + − + − + i i i i a) 2 b) i c)4 d) 0 e) N.A 02.Si Z = 1 + i ; Calcular: Z8 a) 2 i b) 4 i c) 16 d) 18 e) N.A 03.Si la raíz cuadrada del número complejo 1 + i es x + y i , Hallar el valor de : M = x/y - y/x a) 0 b) 1 c) 2 d) 3 e) N.A 04.Sabiendo que E es un valor real, donde el valor de: E = ( ) i i i − + + 3 43 225 ; donde i = 1− a) 4 b) 2 c) 3 d) 1 e) N.A 05.Sumar: + − + + − + + − + i i i i i i 56 65 34 43 2 21 ……………n términos a) ( n + 1 ) i b) ( 2n + 1 ) I c) n i d) 2n I e) N.A. 06.Hallar “α - β” en : ( 1 + i ) ( 2 + i ) ( α + i ) = (1 - i ) (2 - i ) (β - i ) Sabiendo su 1−=i a) 2 b) 4 c) 0 d) -1 e) N.A 07.Hallar “a + b” si ( ) ibai i i i i +=+ − + + − − 4 21 21 5 3 2 23 a) 2 b) 4 c) 6 d) 8 e) N.A 08.Hallar “a + b” si: ( ) ibai i i i i +=+ − + + − − 4 21 21 5 3 2 23 a) 2 b) 4 c) 6 d) 8 e) N. A 09.Si x; y ∈ IR indicar el valor de y x sabiendo que: x – 2y + xi – yi = 2 + 5i a)2 b)1 c) 8 d)1/2 e)1/4 10.Si: ( ) 1; 93 2 9 1 1 3 33 11 2 −= +      ++− ++      −+ = i a i ai i a i Z Calcular: Z4 + 1 a) 82 b) 81 c) a + 800 d) 80i + 16 e) 81 + a 11.Si: 1−=i indicar el valor de: 32 1003432 2 ... iii iiiii E −+− +++++ = a) -1 b) 1 c) 1/2 d) -1/2 e) (1/2)i 12.Halle el módulo del siguiente complejo: ( )( ) ( )( )2 4 173 2231 ii ii E −+ ++ = a) 16 b) 4 c) 32 d) 8 e) 64 13.Si: 7=z ; calcular el valor de: 22 11 zzR +++= a) 15 b) 16 c) 17 d) 18 e) 19 S4AL33B “El nuevo símbolo de una buena educación....” S4AL33B “El nuevo símbolo de una buena educación...."
  • 6. 01 02COLEGIO DE CIENCIAS LORD KELVIN 4to Año Secundaria ALGEBRA 4to Año Secundaria 14.Sumar: ostérn i i i i i i min""... 56 65 34 43 2 21 ++ − + + − + + − + a) i b) -i c) ni d) -ni e) 0 15.Luego de efectuar: 5 2 iii +− Obtenemos a) 1 + i b) -i c) i d) 1 e) 1 – i 16.Si: x, y ∈ IR indique la relación entre ellos para que: (x + yi) (2 + 3i) sea un número real. a) x = 2y b) 2y + 3x = 0 c)x = 3y d) x – y = 1 e) x + y = 2 17.Indicar el cuadrado de: 14091409 −−+−+ a) 80 b) 100 c) 36 d) 625 e) 900 18.Si: {a; b; c; d} ∈ IR / a ≠ b ≠ 0 además: aib bia didiccZ + + =++= ; indicar: z  a) 1 b) 2 c) 3 d) 4 e) 5 19.Halle el módulo del complejo: 3 4 22 )66cos66)(23( i iseni z + °+°+ = ; 1−=i a) 2 13 b) 2 13 c) 4 13 d) 13 e) 8 13 20.Hallar “x + y” en: 1i;y3x i4x3 iy1 ix −== + + + a) 3 b) 5/2 c) 3/2 d) 9/2 e) 7/2 OBJETIVOS ESPECIFICOSOBJETIVOS ESPECIFICOS:  Reconoce y clasifica una ecuación algebraica desarrollando la percepción y acumulando experiencias que servirán de soporte para futuras formalizaciones  Dado un conjunto de Ecuaciones de Primer Grado, trabaja creativamente y con actitud crítica situaciones problemáticas, utilizando una variedad de técnicas de cálculo y aplicando correctamente las propiedades que correspondan. COMENTARIO PREVIOCOMENTARIO PREVIO: Hace cinco mil años, en el país de los sumerios, cerca del Golfo Pérsico, se dieron las primeras dificultades matemáticas que necesitaban ser interpretadas bajo ciertas igualdades. Esto dio inicio a las primeras relaciones que, posteriormente, los matemáticos dieron el nombre de Teoría de Ecuaciones. Con el afán de resolver las ecuaciones se han creado nuevas teorías, nuevos conceptos, nuevos conjuntos numéricos. El método de resolución de las ecuaciones de primer y segundo grado fueron descubiertos por los matemáticos sumerios y babilonios (3000 años a.C) y por Diofante (329 – 410 d.C) fundador del Álgebra, por los hindúes y, finalmente por los árabes (siglo IX). Este método forma parte del más antiguo patrimonio matemático de la humanidad. La ecuación de tercer grado dio ocasión a Cardano (1501–1576) y a Tartaglia (1499– 1557) para inventar los números complejos en el siglo XVI. Ludovico Ferrari (1522–1565), discípulo de Cardano, encontró el método general de la resolución de la ecuación de cuarto grado. Posteriormente, René Descartes (1596–1650), sabio y filósofo francés, inventor de la geometría analítica descubre otra forma de resolver la ecuación cuártica. Como es lógico, los matemáticos trataron de resolver las ecuaciones de grado superior a cuatro (quinto grado, sexto grado,…., de grado n). Este estudio tenía un interés doble, ya que hubiera constituido un gran logro encontrar un método general de resolución para todas las ecuaciones de una incógnita, cualquiera sea su grado. Tras muchos intentos se llegó a la conclusión de que las ecuaciones de quinto grado o superior eran imposibles de resolver sólo usando cálculos algebraicos. Un médico italiano de Bolonia, Paolo Ruffini (1765–1822), había tratado de demostrarlo en 1798, en su teoría general de las ecuaciones; pero la demostración resultó incompleta. Al cabo de unos años, el joven matemático noruego Abel (1802–1829) descubrió en 1824 el teorema que lleva su nombre y dice: “Es imposible resolver algebraicamente las ecuaciones generales de grado superior a cuatro”. Este teorema fue reforzado por Evariste Galois (1811–1832), matemático francés, fundador de la teoría de los grupos. Dado que los matemáticos no lograron encontrar métodos generales de resolución para ecuaciones de grado superior a cuatro; trataron de responder ciertas cuestiones como:  ¿Cuántas raíces positivas posee una ecuación?  ¿Cuántas raíces reales o complejas posee una ecuación?  Dados dos números a y b, ¿cuántas raíces de una ecuación dada están comprendidas entre a y b? (problema de la separación de las raíces de una ecuación). Desde este punto de vista los dos teoremas fundamentales son el de René Descartes y el teorema fundamental del álgebra (K. Gauss – D′Alambert). Este teorema fue enunciado por Girard en 1625, sólo realizó una demostración incompleta por parte de D′Alambert (1746). La primera demostración completa fue establecida por K. Gauss (1799). Después Cauchy, Weierstrass y Kronecker dieron otras demostraciones. El teorema de Gauss – D′Alambert se enuncia “Toda ecuación polinomial de grado n posee por lo menos una raíz (compleja o real)”. CONTENIDO TEÓRICO: S4AL33B “El nuevo símbolo de una buena educación....” S4AL33B “El nuevo símbolo de una buena educación...." ECUACIONES DE
  • 7. 01 02COLEGIO DE CIENCIAS LORD KELVIN 4to Año Secundaria ALGEBRA 4to Año Secundaria 1. IGUALDAD DE NUMEROS REALES Es la relación matemática donde nos indica que dos cantidades tienen el mismo valor. Se denota por el signo =, que se lee igual. Veamos: 27 = 27 ; |9| = |- 9| ; A = B AXIOMAS DE LA IGUALDAD.- Enunciaremos los siguientes axiomas sobre la Igualdad de Números Reales. Axioma de Reflexividad: Todo número real es igual a si mismo. Si a ∈ R ⇒ a = a Axioma de Simetría: Si un número real es igual a otro, entonces el segundo es igual al primero. Si a = b ⇒ b = a, a; b ∈ R Axioma de Transitividad: Si un número real es igual a otro, y este otro es igual a un tercero, entonces el primero es igual al tercero. Si a= b ∧ b = c ⇒ a = c; a; b; c ∈ R 2. ECUACIÓN Una ecuación es una igualdad condicional entre dos expresiones matemáticas definidas sobre un mismo conjunto numérico, donde participa por lo menos una variable (cantidad desconocida llamada variable). Es todo enunciado abierto en que aparece el signo “=” y cuyo valor de verdad se determina mediante su correspondiente conjunto de valores admisibles para la variable (conjunto solución). Notación: A(x) = B(x) OBSERVACIÓNES.- Enunciado abierto: Es toda expresión que contiene por lo menos una variable, que para determinados valores de su dominio se convierte en un enunciado verdadero o falso llamado proposición. Variable: Es el símbolo que puede tomar un valor cualquiera de un determinado conjunto llamado dominio. A las variables que intervienen en la ecuación se les llama incógnitas Conjunto solución: El conjunto solución de una ecuación es el conjunto de valores (soluciones) que permiten que la ecuación sea una proposición verdadera. Si una ecuación no posee solución alguna, entonces definiremos a su conjunto solución como el vacío y lo denotaremos por φ o {} Ejemplo 1. Sea la ecuación: x3 = 4x. Si x=1: 13 = 4(1) → 1 = 4 Proposición falsa Si x=2: 23 =4(2)→8=8 Proposición verdadera Si x=-2: (-2)3 =4(-2)→-8=-8 Proposición verdadera Si x=0: 03 = 4(0) → 0= 0 Proposición verdadera De lo expuesto; vemos que 2, - 2, 0 son soluciones de la ecuación de acuerdo a la definición, luego: CS = {2, - 2, 0} Ejemplo 2: La ecuación 3x – 5 = 0, tiene como raíz o solución a: x = 5/3. Luego, su conjunto solución es: C.S.=       3 5 3. CLASIFICACION DE LAS ECUACIONES ALGEBRAICAS 3.1. DE ACUERDO A SU FORMA: Ecuación polinomial: Es una ecuación algebraica racional entera. P(x) = ax + b= 0 P(x) = ax2 + bx + c = 0 P(x) = ax3 + bx2 + cx + d = 0 P(x) = a0xn + a1xn–1 + a2 xn–2 + a3 xn–3 +...+ an – 1 x + an = 0 n ∈ Z+ ∧ {a0; a1 ; a2; a3; ...an - 1;an} ⊂ R ; a0; a1; a2; a3; ...; ; an - 1; an son los coeficientes. Ecuación fraccionaria: Es una ecuación algebraica racional fraccionaria. P(x)= 2x 7 + - 5x+11= 0 ....... CVA = R - {-2} P(x)= 0 1x 4 3x 5 1x 3 = − − + + + .......CVA =R-{-1,-3,1} Ecuación Irracional: P(x)= 03x2x 2 =−+− .Restricción de la ecuación: x-2≥0→x≥2. Luego CVA=x∈ [2,+∞> 3.2.DE ACUERDO A SU CONJUNTO SOLUCIÓN: Ecuaciones consistentes o compatibles: Son aquellas que tienen o aceptan por lo menos una solución. A su vez se dividen en: - Determinadas: Son aquellas que tienen un número limitado de soluciones. Ejemplo: x3 = x, CS = {1; 0; - 1} - Indeterminadas: Son aquellas que tienen un número ilimitado de soluciones. Ejm: Ejemplo: x + 1 = x + 1, CS = R Ecuaciones Inconsistentes o Incompatibles.- Son aquellas que no tienen solución, también se les denomina absurdas o imposibles. Ejemplo: x 1 = 0 CS = φ 4. ECUACIONES DE PRIMER GRADO 0 LINEALES EN UNA VARIABLE Son aquellas ecuaciones que tienen la forma: P(x) = ax + b = 0 Donde: a, b son los coeficientes, “x” es la incógnita. Para obtener la única raíz o solución de la ecuación, basta con despejar la incógnita, así tendremos que: x = a b − (presentación única solución). 5. ANÁLISIS DE LA ECUACIÓN PARAMETRICA EN VARIABLE “X”. ax= b.........(*) Caso I: Si: a ≠ 0 (no importa el valor de b), reemplazamos en (*), obteniéndose x = b/a una sola solución, con lo cual su conjunto solución es finito, luego (*) es compatible determinada. Caso II: Si: a = 0, b = 0, evaluando en (*) se tiene 0x = 0, indicando que existen infinitas soluciones, luego (*) es compatible indeterminada. CasoIII:Si: a = 0, b ≠ 0, al reemplazar en (*) se obtiene 0x = b que carece de soluciones, con lo cual su conjunto solución es vacío, luego (*) es incompatible. S4AL33B “El nuevo símbolo de una buena educación....” S4AL33B “El nuevo símbolo de una buena educación...." Nota: El conjunto de valores admisibles en una ecuación polinomial son todos los reales. Nota: El hecho de haber establecido el conjunto de valores admisibles (CVA), no implica haber resuelto la ecuación, sólo se le ha restringido.
  • 8. 01 02COLEGIO DE CIENCIAS LORD KELVIN 4to Año Secundaria ALGEBRA 4to Año Secundaria Ejemplo: En la ecuación paramétrica en “x”: (a – 5) (a + 3) x = (a + 2) (a + 3) Halle los valores de a para que sea: I) Determinada II) Indeterminada III) Incompatible Resolución I) (a-5)(a+3)≠    −≠ ≠ 3a 5a 0 ⇒ ∀a ∈ R -{- 3, 5} II)(a – 5) (a + 3) = 0 ∧ (a + 2) (a + 3) = 0 (a = 5; a = – 3) ∧ (a = – 2; a = – 3)⇒ ∴a= - 3 III)(a – 5) (a + 3) = 0 ∧ (a + 2) (a + 3) ≠ 0 (a=5; a=- 3) ∧ (a ≠ - 2; a ≠ - 3) ⇒ ∴ a= 5 6. ECUACIONES EQUIVALENTES: Dos o más ecuaciones de las mismas variables son equivalentes, si y solo si poseen el mismo conjunto solución. Ejemplos: P1 = 14 3 x2 2 x =+ → CS = {12} P2 = 5x – 36= 24 → CS = {12} Como los conjuntos solución son iguales, entonces P1 y P2 son equivalentes: Luego, para resolver ecuaciones en general y de primer grado en particular es necesario tener en cuenta lo siguiente: a) Si se divide ambos miembros de una ecuación por una misma expresión que contenga a la incógnita, entonces se perderán soluciones. Esto se puede evitar si la expresión que se divide (simplifica) se iguala a cero. Ejemplo: Resolver: (x+3) (x-2) = 4 (x - 2) Resolución Simplificando (x - 2) para no perder solución: x – 2 = 0 → x = 2 Luego, tendremos: x + 3 = 4 → x = 1 La ecuación tiene 2 soluciones x=2 y x=1 (de no haber igualado a cero, hubiéramos perdido la solución x=2). b) Si se multiplican ambos miembros de una ecuación por una misma expresión que contenga a la incógnita, entonces se puede introducir soluciones extrañas. Esto se puede evitar si previamente se simplifica por separado cada miembro de la ecuación. Ejemplo: Resolver: ( ) ( ) 4 2 23 = − −+ x xx Resolución Primero simplificamos (x - 2), y tendremos; x + 3 = 4 → x = 1 Observación.- Si hubiésemos trasladado (x - 2) a multiplicar, tendríamos que una solución sería x = 2, que es una solución extraña, pues no verifica la igualdad. c) Si se eleva ambos miembros de una ecuación a un mismo exponente, entonces se pueden introducir soluciones extrañas. Ejemplo: Resolver: 772 −=+ xx Resolución Elevando al cuadrado ambos miembros de la ecuación propuesta: ( ) ( )222 77 −=+ xx x2 +7=x2 –14x+49⇒ 14x = 42 ⇒ x = 3 Pero si reemplazamos; x = 3 en la ecuación dada tendremos: 444167373 2 −=→−=→−=+ Proposición Falsa (No cumple), luego: x = 3 es una solución extraña, y la ecuación es incompatible, pues no tiene solución: Observación: Siempre que se potencie los dos miembros de una ecuación. El valor o los valores obtenidos para “x” deben comprobarse en la ecuación original pues pueden no ser soluciones verdaderas. d) Si a ambos miembros de una ecuación le sumamos un mismo número o un mismo polinomio, la nueva ecuación es equivalente a la inicial. Observación: Si a ambos miembros se suma o resta una función arbitraria la ecuación resultante no necesariamente es equivalente a la inicial. La ecuación: x2 – 12 = 2x + 3 tiene por raíces: x = 5; x = - 3 Sumando a los dos miembros de la ecuación original: 5x 2 − Obtenemos: x2 –12+ 5x 2 − =2x+3 + 5x 2 − . Para lo cual x = 5 no es solución. Observaciones: 1. El conjunto solución de una ecuación depende del conjunto numérico en que se quiere resolver la ecuación, por ejemplo: Si queremos resolver en el conjunto de los racionales (Q), entonces el conjunto solución de la ecuación: x2 = 2, es vacío; pues no existe número racional cuyo cuadrado es 2. Si embargo si resolvemos en el conjunto de los reales (R), entonces el conjunto solución es { 2− , 2 }. De la misma manera, la ecuación x2 = – 1, no tiene solución en R, pero si la tiene en el conjunto C. Al despejar x se obtiene: x = 1− ó x = - 1− . Si definimos 1− =i (i es la unidad imaginaria del conjunto C), el conjunto solución es: {- i; i}. 2. Si p y q son expresiones algebraicas en una variable “x”, entonces un enunciado de la forma “p = q” se llama una ecuación algebraica en “x”. Si obtenemos una proposición verdadera cuando reemplazamos x por x0; entonces x0 es llamada una solución de la ecuación. x0 es un valor del dominio (conjunto de valores admisibles) para x. 3. Si el conjunto solución de una ecuación es todo el dominio para x, entonces la ecuación se llama una IDENTIDAD, por ejemplo: La ecuación: 2 1 x− = )1)(1( xx +− es una identidad; pues es cierta para todo número en el dominio para x, esto es, en el intervalo cerrado: [- 1, 1]. 4. Si en el dominio para “x” existen números que no son soluciones, entonces la ecuación se llama ecuación condicional o un enunciado abierto. Por ejemplo; en la ecuación: x2 = S4AL33B “El nuevo símbolo de una buena educación....” S4AL33B “El nuevo símbolo de una buena educación...." Para resolver una ecuación de primer grado es fácil, bastará con aplicar algunas propiedades básicas de los números reales hasta hallar el valor de la incógnita. Se debe tener cuidado, cuando la variable aparece en el denominador o cuando se presenta un término radical; es justamente en estos casos que aparece una raíz extraña en algunas ecuaciones.
  • 9. 01 02COLEGIO DE CIENCIAS LORD KELVIN 4to Año Secundaria ALGEBRA 4to Año Secundaria x , cuyo dominio para x es: [0, ∞> existen números en el dominio que no son soluciones, por ejemplo x = 4 ∈ [0, +∞>, y no es solución, luego se trata de una ecuación condicional. PROBLEMAS EXPLICATIVOS 01.Sayumi tenía 120 nuevos soles. Si gastó los 7 5 de lo que no gastó. ¿Cuánto dinero gastó Sayumi? Resolución Sea x la cantidad de nuevos soles que gastó Sayumi. Entonces (120 - x) nuevos soles es lo que no gastó. Luego: Gasto = 7 5 (No gastó) Entonces: x = 7 5 (120 - x)↔7x=600 – 5x ↔ 7x + 5x = 600 ↔ 12x = 600 ↔ x = 12 600 ↔ x = 50 Respuesta: Sayumi gastó 50 nuevos soles. 02.Walter llega tarde al colegio cuando había pasado un 8 1 de la clase de álgebra; 6 minutos después llega Jimmi y sólo escucha los 5 4 de la clase. Si la clase empezó a las 8:00 de la mañana. ¿A que hora terminó? Resolución Sea t el tiempo (en minutos) que duró la clase. Jimmi se pierde ( '6t 8 1 + ) de la clase, que equivale a 5 1 t (pues Jimmi sólo escuchó los 5 4 t). Luego: 5 1 t = 8 1 t + 6↔ 5 1 t – 8 1 t = 6 ↔ 40 t3 = 6 ↔ t = 3 6x40 ↔ t = 80’ Respuesta: Como la clase empezó a las 8:00 a.m. y duró 80 minutos entonces terminó a las 9:20 a.m. 03.Un río tiene una corriente de 3 kilómetros por hora. Si el bote de Aly Boydi tarda el mismo tiempo en ir 18 kilómetros río abajo y 15 km. río arriba. Calcule la velocidad del bote en aguas tranquilas. Resolución Sea V la velocidad del bote en aguas tranquilas, entonces (V + 3) es la velocidad del bote río abajo (con la corriente a favor) y (V - 3) es la velocidad del bote río arriba (contra la corriente), entonces tenemos: Distancia Velocidad Tiempo Río Abajo 18 V+3 3V 18 + Río Arriba 15 V – 3 3V 15 − Como el tiempo es el mismo: 3V 18 + = 3V 15 − ↔ 18 (V – 3) = 15 (V + 3) ↔ 18V – 54 = 15 V + 45 ↔ 18V – 15V= 45 + 54 ↔ 3V = 99 ↔ V = 3 99 ↔ V = 33 Respuesta: La velocidad del bote en aguas tranquilas es 33 kilómetros por hora. PRÁCTICA DE CLASE 01.Clasificar las siguientes ecuaciones algebraicas de acuerdo a su forma. S4AL33B “El nuevo símbolo de una buena educación....” S4AL33B “El nuevo símbolo de una buena educación...." Clasificación de las Ecuaciones De acuerdo a su forma De acuerdo a su C.S. Algebraica Poseer grado No Algebraica Compatible Incompatible (CS= O ) Polinomial P(x)=a o xn + a x n-1 1 + a x n-2 2 + ...+ a = 0 n Fraccionaria 3 x+2 = 5 Irracional x - 7 = 7 - x Se considera Se caracterizan por: Exponencial xx- 256= 0 Logarítmico Log 6 x - 1= 0 Trigonométrico Senx - x= 0 Puede ser Determinada (C.S. Finito) Indeterminada (C.S. Infinito) Se enfoca
  • 10. 01 02COLEGIO DE CIENCIAS LORD KELVIN 4to Año Secundaria ALGEBRA 4to Año Secundaria ECUACIÓN ALGEBRAICA CLASIFICACIÓN 7xx2x 23 −+− = 0 x2 2x 5 − − = 0 ECUACIÓN ALGEBRAICA CLASIFICACIÓN x24x −−− = 0 3x 3 5 2x x3 − −+ + = 0 02.Clasificar las siguientes ecuaciones algebraicas de acuerdo a sus soluciones: ECUACIÓN ALGEBRAICA CLASIFICACIÓN x3 = 9x 2x + 5 = 2x + 5 x + x 1 x 1 = ECUACIÓN ALGEBRAICA CLASIFICACIÓN x(x - 2) = (x - 1)2 5x = 5x 5x − - 2x +− 03.Encierra en una circunferencia V (Verdadero) o F (Falso). - El conjunto de valores admisibles en una ecuación algebraica implica que la ecuación ha sido resuelta. V - F - En una ecuación polinomial sus coeficientes son números naturales V - F - Una ecuación es una proposición matemática V - F - Una ecuación compatible indeterminada tiene infinitas soluciones. V - F 04.Una ecuación compatible: a) Tiene 2 incógnitas b) No tiene solución c) Tiene un número finito de soluciones d) Tiene un número infinito de soluciones e) c y d 05.Toda ecuación lineal presenta: a) 1 solución b) 2 soluciones c) 3 soluciones d) 4 soluciones e) N.A. 06.Se llama ecuación polinomial a la: a) Ecuación algebraica racional entera b) Ecuación algebraica racional fraccionaria c) Ecuación trascendente d) Ecuación irracional e) N.A. 07.Una ecuación se llama incompatible si: a) Tiene infinitas soluciones b) Tiene 3 incógnitas c) Tiene un número finito de soluciones d) Es irracional e) No admite solución 08.Resolver: x + 5 + 6x 4 x7 6x 4 − +−= − a) 6 b) – 6 c) 6 y – 6 d) Indeterminado e) Incompatible 09.Resolver: x– 4 + 2 x420x8x5 −+−=− a) 6 b) – 6 c) 6 y – 6 d) Indeterminado e) Incompatible 10.Resolver: 2xx 9x 2x 3x 2 2 −+ − = + − . Marque lo correcto: a) Tiene una raíz b) Tiene dos raíces c) Tiene tres raíces d) Indeterminado e) Incompatible 11.Respecto a la ecuación en x, a (a2 – 1) x = 0, establezca el valor de verdad de cada proposición: I. Es compatible para cualquier valor de a. II. Si a = –1, tiene infinitas soluciones. III. Si a = 0, tiene solución única. IV. Si a ∈ {0; 1; –1}, tiene una única solución e igual a cero. a) VVVV b) VFVF c) FFVV d) FFFV e) FVFF 12.Determine el valor de verdad de las siguientes proposiciones con respecto a la ecuación en x: ( ) 21)2(2 −=−−+−− axaaaaaa . I. Es determinado cuando a ≠ 1∧a ≠ -1 II. Es indeterminado cuando a =1∨a= -1 III. Es incompatible cuando a = 2 a) VVV b) VVF c) VFV d) FFV e) VFF 13.Luego de resolver la ecuación en “x”: a axaxax ax 2 15 423 5 3 3 2 − − = − + − ++ Es cierto que: a) La solución depende de a (a ∈ ℜ) b) Tiene una sola solución c) No tiene solución d) Tiene infinitas soluciones e) Tiene dos soluciones 14.Luego de resolver la ecuación en “x”.       ++= − + − + − cbaab cx ac bx bc ax 111 2 I. Si a + b + c = 0 la ecuación tiene infinitas soluciones con abc ≠ 0. II. Si a + b + c ≠ 0 siempre existe solución y es única. III. Siempre la solución es a + b + c. a) VVV b) VFV c) VFF d) FVV e) FFV EJERCICIOS PROPUESTOS Nº 03 01.Resolver: xx −=− 9)1( 2 . a) Incompatible b) 0 c) 5 d) 5, - 5 e) Indeterminado 02.Resolver: 116513 +=++ xxx . Indique la suma de sus raíces. a) 0 b) 5 c) 6 d) 7 e) 9 S4AL33B “El nuevo símbolo de una buena educación....” S4AL33B “El nuevo símbolo de una buena educación...."
  • 11. 01 02COLEGIO DE CIENCIAS LORD KELVIN 4to Año Secundaria ALGEBRA 4to Año Secundaria 03.Resolver: 6 10 1 3 3 7 2 5 + + + − + = + + + + + x x x x x x x x Indique: x3)x27( ++ a) 4 b) 2 c) - 27 d) 3 3 e) 2 04.Dada la ecuación en x: 6 75 3 21 + = + + + xx x x Dar el valor de verdad: I. La ecuación dada es lineal II. La ecuación tiene infinitas soluciones III. La ecuación tiene solución única IV. x = 32 + es solución de la ecuación V. La ecuación dada es ecuación polinomial a) FVFVV b) FVFVF c) VVVFF d) FFVVV e) VFVFV 05.Para que valor real del parámetro “n”, la ecuación del primer grado “x”: (2n – 1)x + 2 = nx – 3n2 será compatible y determinada. a) ∀n ∈ R b) 2 c) 3 d) ∀n ∈ R+ e) ∀ n ∈ R - {+1} 06.En la siguiente ecuación: (x+1) + (x+2)+(x+3) + ... + (x+n)= n2 , n entero positivo, el valor de x es: a) 2 )1( −n b) 2 )1( +n c) 2 n d) 2 3n e) 2 )12( +n 07.Si se define: P(n) = n + 3; f (m) = 3m. Calcular “x” en: f (P (f (P (2)))) – P (f (P (x))) = 75. a) 4 b) -11 c) 12 d) -15 e) -1 08.Resolver: 1 22 1 22 1 = −−+ + −++ xxxx a) 1 b) 4 c) 5 d) 2 e) - 1 09.Resolver: x + 2 25 x− = 7. ¿Cuántas soluciones tiene? a) 1 b) 2 c) 3 d) 4 e) 0 10.Hallar x en: x+++ 141221 = 5 a) 1 b) 4 c) 9 d) 16 e) 0 11.Si |– 9x| = 72. Calcular: |x – 3|. a) 0 b) {1, 2} c) {5, 11} d) 11 e) 5 12.Sea la ecuación en “x”: a3 x – a4 + 6a2 = (3a – 2)x + 8a – 3 e indicar el valor de “a” para el cual la ecuación presenta infinitas soluciones: a) 1 b) 2 c) 3 d) 4 e) 0 13.Resolver: 3 )81...941(285 306 1 ... 20 1 12 1 6 1 285 x x +++++=      ++++ a) 1 b) 7 c) 8 d) 9 e) 570 14.Si a ≠ b, resolver en x: a (x - a2 ) - b(x -b2 ) = 0. a) φ b) {0} c) {1} d) {a + b} e) {a2 + ab + b2} 15.Determinar el cardinal del conjunto solución de la ecuación: =− 2 )2(x - 9. a) 0 b) 1 c) 2 d) 4 e) N.A. 16.Resolver la ecuación: (x+1) + (x+2)+(x+) +... +(x+20) = 420 – x. a) 0 b) 5 c) 10 d) 12 e) 21 17.Hallar m y p para que la ecuación: 3mx–4p=2x+m. Sea: I) Incompatible II) Indeterminada Señalar la suma de soluciones de m: a) 2/3 b) 1/3 c) 1 d) 4/3 e) 5/3 18.Si: 2 )1( −x = 1 - x. El conjunto solución de la ecuación es: a) x = 1 b) x = 3 c) x > 1 d) x < 1 e) x = 2 19.Resolver la ecuación de primer grado: (m – 3)x2 + 5m + (m – 2)x – 14 = 0 a) 1 b) - 1 c) 0 d) 19 e) 15 20.Resolver: x – 7+ 5 11 −x = 3 – x + 5 11 −x a) 5 b) 5; - 5 c) - 5 d) Indeterminado e) Incompatible TAREA DOMICILIARIA 01.Compre cierto número de folletos de álgebra por 100 nuevos soles. Si el precio por el ejemplar me hubiese costado un nuevo sol menos, tendría 5 ejemplares más por el mismo dinero. ¿Cuántos folletos compre? a) 5 b) 4 c) 25 d) 20 e) 15 02.José tiene tres veces los años que tenía Ricardo cuando el tenía 16 años. Ricardo tiene 24. Hallar la edad de José. a) 25 b) 20 c) 40 d) 30 e) 35 03.En un reloj se lee: 8: 48 cuando en realidad son: 8:52, más tarde a las 9:42 se lee 9:34, y según esto; ¿A que hora se tenía una lectura correcta? a) 8:02 b) 8:00 c) 8:04 d) 8:25 e) 9:11 04.En una sala de juego para entrar se paga 1 dólar y para salir 1 dólar. Una persona juega en 3 salas y pierde en cada una la mitad de lo que tiene. ¿Cuánto tenía antes de empezar a jugar si al final se queda sin dinero? a) 20 b) 21 c) 22 d) 23 e) N.A. 05.El conjunto solución de: 9x 15x6x 3x 4x 3x 1x2 2 2 − +− = − − − + − , es: a) IR b) {3, - 3} c) {4, - 4} d) IR - {3, - 3} e) N.A. 06.Resolver: x+++ 133210 = 4 S4AL33B “El nuevo símbolo de una buena educación....” S4AL33B “El nuevo símbolo de una buena educación...."
  • 12. 01 02 ax2 +bx + c = 0 , {a; b; c} ⊂ R / a≠ 0 COLEGIO DE CIENCIAS LORD KELVIN 4to Año Secundaria ALGEBRA 4to Año Secundaria a) 12 b) 16 c) 25 d) 36 e) 9 07.Resolver la ecuación: 2 1 65 5 3 4 2 = +− − + − − xx x x x a) 1 b) 2 c) 3 d) 4 e) 5 08.Resolver la ecuación: 3 11 3 1 1 = + − − x x x a) 18 b) 9 c) 25 d) 16 e) 4 09.Resolver: 2 4 2 + =++ x xx . a) 1/2 b) 4/5 c) 2/3 d) 3/2 e) 3/4 10.Resolver: ab baax a bx b ax 2 )(32 −+ = − − + . a) 2 ba + b) ba ab + c) ba ba − + d) ba ab + 2 e) ba ba + − 11.Indique que pares de ecuaciones son equivalentes: I. x = 4; x2 = 16 II. x = 4; x2 = 4x III. x = 4; x = 16 IV. x = 4; 4x = 16 a) Las 4 posibilidades planteadas b) Sólo I y II c) Sólo II y III d) Sólo III y IV e) Sólo I y IV 12.El valor de x que satisface la ecuación fraccionaria: 3 1 1 1 1 1 2 1 1 1 1 + + = + + x . a) 3/4 b) 1/2 c) 2/3 d) 5/6 e) 7/6 13.Hallar el valor de x en: 64...... 3 3 32424 =∞÷÷÷÷ xxxx a) 2 b) 32 c) 16 d) 4 e) 64 14.Hallar el valor del parámetro “a” de modo que la ecuación a2 x + 2x + 2 = a2 + a + 3ax sea:  Compatible determinado  Compatible indeterminado  Incompatible BIBLIOGRAFIA 1. PERELMAN : Álgebra Recreativa 2. POTAPOV : Álgebra 3. SWOKOVSKI : Álgebra Universitaria 4. ACADEMIA CESAR VALLEJO: Compendio Académico 5. RYBNIKOV: Historia de la Matemáticas 6. ALFONSO MORALES: Matemática Resumida OBJETIVOS ESPECIFICOSOBJETIVOS ESPECIFICOS:  Dado un conjunto de Ecuaciones de segundo Grado, resolverlos aplicando correctamente las propiedades que corresponden. COMENTARIO PREVIOCOMENTARIO PREVIO: La historia del álgebra comenzó en el antiguo Egipto y Babilonia donde fueron capaces de resolver ecuaciones lineales (ax = b) y cuadráticos (ax2 + bx + c) así como ecuaciones indeterminadas como x2 + y2 = z2 con varias incógnitas. Los antiguos Babilonios resolvían cualquier ecuación cuadrática empleando esencialmente los mismos métodos que hoy se enseñan. También fueron capaces de resolver algunas ecuaciones indeterminadas. Los matemáticos alejandrinos Heron y Diofante continuaron con la tradición de Egipto y Babilonia aunque el libro "La aritmética de Diofante" es de bastante más nivel y presenta muchas soluciones es sorprendente para ecuaciones indeterminadas difíciles. Esta antigua sabiduría sobre resolución de ecuaciones encontró a su vez, acogida en el mundo islámico en donde se le llamó "ciencia de reducción y equilibrio" (La palabra árabe "ál - jabru" que significa reducción es el origen de la palabra álgebra). En el siglo IX el matemático al-juarizmi escribió uno de los primeros libros árabes de álgebra, una presentación sistemática de la teoría fundamental de ecuaciones con ejemplos y demostraciones incluidas CONTENIDO TEÓRICO:CONTENIDO TEÓRICO: ECUACIONES DE SEGUNDO GRADOECUACIONES DE SEGUNDO GRADO Llamadas también ecuaciones polinomiales de segundo grado, cuya forma general es: Frecuentemente a dicha ecuación se le llama: Ecuación Cuadrática y se caracteriza por presentar 2 soluciones (su incógnita “x” asume dos valores) MÉTODOS DE RESOLUCIÓN DE LA ECUACIÓN. Toda ecuación de 2do grado podrá resolverse al menos por una de las siguientes formas: A) Por Factorización Este método se aplica únicamente si el trinomio: ax2 + bx + c es factorizable por aspa simple, para lo cual se debe tener en cuenta la siguiente propiedad: Si: m . n = 0 ⇒ m = 0 ∨ n = 0 Resolver: ax2 + bx + c = 0 Factorizando se obtiene: a(x–x1) (x–x2) = 0 De donde:x–x1=0∨x–x2 =0⇒x= x1∨x= x2 ∴C.S = {x1; x2}, x1; x2 se llaman raíces de la ecuación polinomial cuadrática. Ejemplo: Resolver la siguiente ecuación: x2 – x – 12 = 0 Resolución La ecuación dada es: x2 – x – 12=0 Factoricemos al trinomio: x2 – x – 12 Según el criterio del aspa x2 – x–12=(x–4)(x+3) simple tendremos: x -4 x 3 luego la ecuación dada será: (x–4) (x+3) = 0 Finalmente de acuerdo a la propiedad señalada líneas arriba; se tendrá: S4AL33B “El nuevo símbolo de una buena educación....” S4AL33B “El nuevo símbolo de una buena educación...." ECUACIONES DE
  • 13. 01 02COLEGIO DE CIENCIAS LORD KELVIN 4to Año Secundaria ALGEBRA 4to Año Secundaria x – 4 = 0 ∨ x + 3 = 0 ⇒ x = 4 ∨ x= -3 Es decir el conjunto solución de la ecuación: x2 – x – 12= 0, es : C.S. = {4; -3} B) Por la Fórmula de Carnot Dada la ecuación: ax2 + bx + c = 0, sus raíces se obtienen utilizando la fórmula deducida por Sadi Carnot: a acbb x 2 42 −±− = Donde las raíces son: a2 ac4bb x 2 1 −+− = ; a2 ac4bb x 2 2 −−− = Luego el conjunto solución es:         −−−−+− = a acbb a acbb SC 2 4 ; 2 4 .. 22 Ejemplo: Resolver la siguiente ecuación: x2 + 3x – 1 = 0 Resolución De la ecuación se deduce que: a = 1 ∧ b = 3 ∧ c = –1 Reemplazando en la fórmula tenemos: )1(2 )1)(1(433 x 2 −−±− = Efectuando y reduciendo: 2 133 x ±− = Finalmente las raíces de la ecuación son: 2 133 x1 +− = ; 2 133 x2 −− = En consecuencia el conjunto solución es:         −−+− = 2 133 ; 2 133 .S.C ANÁLISIS DE LA ECUACIÓN. Para la ecuación: ax2 + bx + c = 0, se tiene: I) Si: a ≠ 0 ∧ {b ; c} ⊂ R , la ecuación es : Compatible Determinada II) Si: a = 0 ∧ b = 0 ∧ c = 0, la ecuación es compatible Indeterminada. III) Si: a = 0 ∧ b = 0 ∧ c ≠ 0, la ecuación es Incompatible. NATURALEZA DE LAS RAÍCES. A) DISCRIMINANTE (∆) Llamamos discriminante a la expresión subradical contenida en la fórmula de Carnot: ∆ = b2 – 4ac De este modo la fórmula que da solución a una ecuación de segundo grado queda así : a2 b x ∆±− = B) ANÁLISIS DEL DISCRIMINANTE Observando la relación anterior, resulta previsible que el valor y/o signo del discriminante determinará la naturaleza de las raíces de una ecuación de 2do grado. Veamos los siguientes casos: Primero: Si: ∆ > 0 En este caso las raíces de la ecuación serán reales y diferentes. Segundo: Si: ∆ = 0 En este caso las raíces de la ecuación cuadrática serán reales e iguales. Este caso se presenta cuando el trinomio “ax2 + bx + c” es un cuadrado perfecto. Tercero: Si : ∆ < 0 En este caso las raíces de la ecuación serán imaginarias y conjugadas. Debe notarse que las raíces imaginarias siempre se presentan en parejas, siendo una la conjugada de la otra. Cuarto: Si:∆ = k2 (cuadrado perfecto) Siendo a, b ∧ c números racionales, las raíces de la ecuación serán reales racionales. Pero si ∆ ≠ k2 , las raíces de la ecuación serán reales irracionales y conjugadas. PROPIEDADES DE LAS RAÍCES. Para la ecuación: ax2 + bx + c = 0 / a ≠ 0, de raíces x1 ∧ x2 , tenemos: I) Suma de Raíces: s = x1 + x2 = a b − II) Producto de Raíces: p = x1 . x2 = a c III) Diferencia de Raíces: d= x1-x2= a ∆ Para determinar la diferencia de las raíces se recomienda utilizar la siguiente identidad 21 2 21 2 21 4)()( xxxxxx =−−+ A) RAÍCES PARTICULARES En algunas ecuaciones las raíces se condicionan de tal modo que efectuando alguna operación elemental entre ellas, se podrá deducir alguna propiedad particular, como por ejemplo: Raíces Simétricas: Si x1 ∧ x2 son raíces simétricas, se podrá establecer lo siguiente: x1 = m ∧ x2 = –m ⇒ x1 + x2 = 0 Raíces Recíprocas: Si x1 ∧ x2 son raíces recíprocas, se podrá establecer lo siguiente: x1 = m ∧ x2 = m 1 ⇒ x1 . x2 = 1 B) RAÍCES ESPECIALES Llamaremos así a las siguientes raíces: Raíz Nula: Dada la ecuación cuadrática ax2 +bx +c=0 / a ≠ 0, si ésta presenta una raíz nula (x=0), se cumplirá que: c = 0. Raíz Unidad: Dada la ecuación cuadrática ax2 +bx+c=0 / a ≠ 0, si ésta presenta una raíz unidad (x =1), se cumplirá que: a + b + c = 0. RECONSTRUCCIÓN DE UNA ECUACIÓN CUADRÁTICA. Considerando a x1 ∧ x2 como raíces de la ecuación tal que: S = Suma de raíces P = Producto de raíces Entonces la ecuación que originó a dichas raíces se determina así: x2 – Sx + P = 0 PROPIEDADES IMPORTANTES. A. De las Ecuaciones Equivalentes S4AL33B “El nuevo símbolo de una buena educación....” S4AL33B “El nuevo símbolo de una buena educación...."
  • 14. 01 02COLEGIO DE CIENCIAS LORD KELVIN 4to Año Secundaria ALGEBRA 4to Año Secundaria Sean: a1 x2 + b1 x + c1 = 0 ...... (1) a2 x2 + b2 x + c2 = 0 ...... (2) Dos ecuaciones equivalentes, luego entre ellas se cumplirá la siguiente relación: 2 1 2 1 2 1 c c b b a a == PRÁCTICA DE CLASEPRÁCTICA DE CLASE 01.Resolver las siguientes ecuaciones a) + + 3 1x 5 2x − = 15 1x8 + b) x - 7212 =−x c) 34 4 9 1 32 2 222 +− = − + − −+ − xxx x xx x 02.Resuelve las siguientes ecuaciones cuadráticas: a) (x + 1)(x + 2)(x+3) = x(x + 4)(x + 5) b) 3 2 2 1 1 = + − + − + x x x x c) 3 72 x− - 3 16 x− = 2 d) 2 5 2 3 3 2 = − − + − − x x x x 03.Completar: a) 2x2 – 7x – 3 = 0 ∆ = ………………… b) 7x2 – 11x – 14 = 0 S = ………………… c) x2 – 5x + 6 = 0 =+ 21 x 1 x 1 ……….. d) 2x2 + 7x + 1 = 0 1 2 1 1 xx −− + = ……… e) 2x2 + x – 1 = 0 ( )( )1x1x 21 ++ = …. f) x2 + 2x – 1 = 0 ( )( )1x1x 21 ++ = …. 04.Relaciona correctamente: I) x2 – 4 3 x + 12=0 a)Raíces reales iguales II) x2 – 2x – 1 = 0 b)Raíces reales diferentes III) x2 – 2x + 3 = 0 c)Raíces complejas a) I A – II B – III C d) I C – II B – III A b) I B – II C – III A e) I A – II C – III B c) I C – II A – III B 05.Calcular “m” para cada uno de los siguientes casos, siendo la ecuación cuadrática: (m + 1)x2 – (3m – 5)x + 2m – 5 = 0 a) Suma de raíces es 5/2 m=……...... b) Producto de raíces es 9/4 m=……...... c) Raíces recíprocas. m=……...... d) Raíces simétricas m=……...... e) Una raíz es – 2 m=……...... 06.Calcular “n” para cada uno de los siguientes casos, siendo la ecuación cuadrática: (2n - 5)x2 + (3n - 5)x + n + 1 = 0 a) Raíces iguales b) Suma de las inversas de las raíces es –5/2 c) Diferencia de raíces es 0,5 d) Suma de los cuadrados de las raíces es 5/4 07.Formar una ecuación cuadrática con coeficientes enteros para cada uno de los siguientes casos: a) x1 = 7 x2 = 4 b) x1 = 2/3 x2 = - 3/5 c) x1 = 3 - 2 d) x1 = 4 + i e) x1 + x2 = - 7/3 ∧ x1 . x2 = 5/9 08.¿Para qué valor de “m” las raíces de la ecuación: x2 –(m+3)x+ 4 m 2 +1=0; se diferencian en 2? a) – 6 1 b) 3 1 c) - 3 1 d) 6 1 e) 3 2 09.La ecuación de segundo grado una de cuyas raíces es la fracción: x =  1 2 1 3 1 2 1 3 1 1 + + + + + ; está dada por: a) 3x2 – 5 = 0 b) 5x2 – 3 = 0 c)3x2 –x–5=0 d) 5x2 – x – 3=0 e) 2x2 - 4 = 0 10.Determine la suma de los valores que puede tomar “a” para que la ecuación: (a + 1) x2 + ax + 1 = 0; tenga una sola solución si “a” es un número real y diferente de –1. a) 12 b) 4 2 c) 4 d) 5 e) 6 11.Sea: {x1; x2} el conjunto solución de: 3x2 – x – 1 = 0. A continuación se establece que: P(n) = n n 2 n 1 xx + ; calcular: P(2) a) 7 b) 3 7 c) 3 d) 7 e) 3 12.Si la ecuación: x2 – 6x + n + 1 = 0, admite como raíces a x1 ∧ x2 , tal que : 5 3 x2 1 x2 1 21 =+ ; Encontrar el valor de n: a) 1 b) 2 c) 3 d) 4 e) 5 13.¿Para qué valor de “n” el discriminante de la ecuación: x2 – 8x + n = 0, es igual a 20? a) 44 b) 11 c) 33 d) 22 e) 17 14.Sabiendo que las ecuaciones: x2 + mx + n = 0 x2 + nx + m = 0 Presentan una raíz común, formar otra ecuación cuadrática cuyas raíces sean las no comunes de las anteriores a) x2 + x – 1 = 0 b) x2 + (m – n) x + mn = 0 c) x2 – x + 1 = 0 d) x2 – (m + n) x + mn = 0 e) x2 – mn = 0 S4AL33B “El nuevo símbolo de una buena educación....” S4AL33B “El nuevo símbolo de una buena educación...."
  • 15. 01 02COLEGIO DE CIENCIAS LORD KELVIN 4to Año Secundaria ALGEBRA 4to Año Secundaria 15.En la ecuación: 2x2 – (m - 1) x + m + 1 = 0, ¿qué valor positivo debe darse a “m” para que las raíces difieran en uno? a) 7 b) 8 c) 9 d) 10 e) 11 16.Sabiendo que: (p + q)2 y (p – q)2 son raíces de cierta ecuación cuadrática recíproca donde “p” y “q” son raíces de la ecuación: ax2 + bx +c = 0; a > b > 0, calcular a4 – b4 a) 2abc b) – 2abc2 c) 4abc2 d) – 4 ab2 c e) – 4abc2 17.Sabiendo que la ecuación: x4 – 9x + λ = 0 admite dos raíces que suman 3, calcular el producto de todas las raíces a) 3 b) 6 c) 9 d) 12 e) 18 18.Si las raíces de la ecuación en “x” x2 – 3x + m + 1 = 0 3x2 + 5x + m = 0 Son imaginarias y reales respectivamente determine el valor entero de “m” a) 0 b) 1 c) - 1 d) 4 e) 2 19.Determine a + b +c de modo que la ecuación: x3 – ax2 + bx + c = 0 Admita por raíces: a, b, c; abc ≠ 0 a) 1 b) - 1 c) 0 d) 4 e) 8 20.Resolver: 18 x 1 x3 x 1 x 3 =      +−      + Indicar la raíz de mayor valor a) 2 +1 b) 3-2 2 c) ( 2 +1)2 d) 2+3 2 e) (3+ 5 )/2 EJERCICIOS PROPUESTOS Nº 04 01.Indicar la mayor raíz de la ecuación: x2 - 3x + 2,16 = 0 a) 1,2 b) 0,8 c) 1,8 d) 0,3 e) 1,2 02.Si : x = ....221 +++ , puede decirse que: a) x = 3 b) 0<x<1 c) x>2 d) x =2 e)x es infinitamente grande 03.Cuál o cuáles de las siguientes ecuaciones: I. x2 – x – 1 = 0 II. x2 – 2x + 3 = 0 III. 3x2 + x – 2 = 0 No admite raíces reales. a) Solo I b) Solo II c) Solo III d) II y III e) I y II 04.Halle la menor raíz de la siguiente ecuación mónica de segundo grado: (m - 2) x2 – (3m - 8) x + m – 9 = 0 a) -2 b) -3 c) 2 d) 3 e) -1 05.Calcular el valor de “m-2n” si la ecuación cuadrática: 5 (m + n +18) x2 + 4(m - n) x + 3mn = 0 Es incompatible. a) -9 b) -18 c) 9 d) 18 e) -13 06.Calcular la mayor solución de la ecuación: (m - 2) x2 – (2m - 1) x + m – 1 = 0 Sabiendo que su discriminante es 25. a) 3 b) 0,5 c) 2,5 d) 1,5 e) N.A. 07.Calcular “m” para que la ecuación: 6x2 + (2m + 3) x + m = 0. Tenga única solución. a) 3 b) 3/4 c) 1/2 d) 3/2 e) 5/3 08.Indicar el valor de verdad de las siguientes proposiciones, en base a la ecuación: x (x - 1)2 (2x - 3)3 (x2 - 3 )2 = 0 ( ) Posee 4 raíces o soluciones ( ) Su conjunto solución posee 5 elementos ( ) Posee a x = 0 como raíz simple y a x = 3/2 como raíz triple. a) VVV b) FVV c) FFV d) VFV e) VVF 09.En la ecuación cuadrática: ax2 +bx+c = 0 Afirmamos: I. Si la suma de sus raíces es igual a su producto entonces b + c = 0 II) Si una raíz es la opuesta de la otra entonces b = 0 III) Si una raíz es el doble de la otra, entonces 2b2 = 9ac a) Las 3 afirmaciones son verdaderas b) I y II son verdaderas c) I y III son verdaderas d) II y III son verdaderas e) Sólo II es verdadera 10.Si “r” y “s” son las raíces de la ecuación: ax2 + bx + c = 0 ; el valor de : 22 s 1 r 1 + , es: a) b2 - 4ac b) a2 ac4b2 − c) 2 2 c ac4b − d) 2 2 c ac2b − e) b2 + 4ac 11.Si la ecuación: x2 – nx + 36 = 0, admite como raíces a : x1 ∧ x2, tal que: 12 5 x 1 x 1 21 =+ ; encontrar el valor de “n”. a) 25 b) 18 c) 12 d) 24 e) 15 12.Siendo : x1 ∧ x2 las raíces de la ecuación : 5x2 – 23x + 11 = 0 , el valor de: 9x2 1x3 . 9x2 1x3 2 2 1 1 − + − + ; es: a) 35 17 b) 35 143 c) 35 153 d) 35 183 e) 35 173 13.¿Para qué valores de “m” la ecuación: x2 - 2(3m+1) x + 7(2m+3) = 0, tendrá sus dos raíces iguales? a) 5 ; 2 b) 1 ; -3/2 c) 4 ; -2 d) 3 ; -1 e) 2 ; -10/9 14.La ecuación cuadrática cuyas raíces son: 2+ 2 ∧ 2- 2 , es: a) x2 + 2x – 1= 0 d) x2 + 4x +2= 0 b) 2x2 - 4x + 1= 0 e) x2 - 4x + 2= 0 S4AL33B “El nuevo símbolo de una buena educación....” S4AL33B “El nuevo símbolo de una buena educación...."
  • 16. 01 02COLEGIO DE CIENCIAS LORD KELVIN 4to Año Secundaria ALGEBRA 4to Año Secundaria c) x2 - 8x + 2= 0 15.Si “α” y “θ” son las raíces de la ecuación: x2 - 2x – 5 = 0, encontrar una ecuación cuadrática cuyas raíces sean: α2 y θ2 . a) x2 +14x + 25=0 d) x2 +14x+15= 0 b) x2 - 2x - 1= 0 e) x2 - 14x - 25= 0 c) x2 - 14x + 25= 0 16.Si x1 y x2 son raíces reales de: ax2 +bx+c=0 (a ≠ 0), calcular el valor de “m” para que la ecuación de raíces (x1 + m) y (x2 + m); carezca de término lineal a) – b /2a b) b/2a c) b/a d) – b/a e) b/3a 17.Determinar la ecuación de segundo grado cuyas raíces sean: una la suma y la otra el producto de las raíces de: ax2 +bx+c=0; a≠ 0 a) a2 x2 – a(b - c)x – bc = 0 b) a2 x2 – a (b + c)x – bc = 0 c) a2 x2 – a (b + c)x + bc = 0 d) a2 x2 + a (b - c)x + bc = 0 e) a2 x2 + a (b - c)x – bc = 0 18.Siendo x1 y x2 las raíces de la ecuación: ax2 + bx + b = 0; a ∧ b ≠ 0 Tales que x1 es a x2 como “b” es a “a” calcular: 21 xx a b b a R ++= a) 0 b) 1 c) 2 d) 3 e) 4 19.La ecuación P(x) = x2 + bx + c = 0; tiene raíces reales positivas distintas, entonces de las raíces de la ecuación: F(x) = 2 x + b x + c = 0; se puede afirmar: a) Son las mismas de P(x) b) Algunas son negativas c) Algunas son complejas d) Son todas positivas e) Son todas negativas 20.Hallar la ecuación de segundo grado de coeficiente principal 1 y de raíces m y n se sabe que:  x2 + (m – 1) x + m – 2 = 0; tiene una sola solución real.  x2 – (n + 1) x + 2n = 0; tiene una raíz igual a 3. a) x2 + 9x + 18 = 0 b) x2 – 6x + 18 = 0 c) x2 – 9x – 18 = 0 d) x2 – 9x + 18 = 0 e) x2 – 6x – 18 = 0 TAREA DOMICILIARIATAREA DOMICILIARIA 01.Resolver las ecuaciones: 1) x2 = 7 2) (x + 1) (x – 3) = 12 3) 15x2 – 34x + 15 = 0 4) (x + 3) (x + 5) = 13x2 5) x(x - 1997) = (x - 1997) Indicar la ecuación que posee la menor raíz a) 1 b) 2 c) 3 d) 4 e) 5 02.Sea la ecuación: [(m + n)2 – (m - n)2 ] x2 + (m - 1)2 x – [(m + n)2 + (m - n)2 ] = 0 siendo m ≠ 0 ∧ n ≠ 0 y x1 y x2 son sus raíces. ¿En cuántas unidades es necesario disminuir dichas raíces para que sean simétricas? a) 1/n b) – 1/n c) 1/2 n d) – 2n e) – 1/2 n 03.Hallar una de las raíces de la ecuación: a (b - c)x2 + b (c - a) x + c (a - b) = 0 Si x es la incógnita a) ba cb − − b) cb ac − − c) ( ) ( )cba cab − − d) ac ba − − e) ( ) ( )cba cab − − 04.Dada la ecuación: x2 - 2x + m = 0. Calcular “m” si una de las raíces es 1 + 2i, (i = 1− ); m ∈ R a) 2 b) 3 c) 4 d) 5 e) 8 05.Si la ecuación: x2 + px + q = 0; tiene por conjunto solución {r, s} si: r – s = 4 y r3 – s3 = 208; entonces p/q es: a) 2/3 b) 3/2 c) 2/5 d) 2/7 e) 1/7 06.Hallar el valor de “a” para que las raíces de la ecuación: x2 – (a + 3) + 1 4 2 + a = 0 se diferencien en 5 a) 5/3 b) 7/3 c) 10/3 d) 5/6 e) 20/3 07.Resolver e indicar la solución: 275232522 =−+++−+− xxxx a) 7 b) 13 c) 15 d) 5 e) 16 08.Calcular “m” para que la ecuación: 6x2 + (2m + 3) x + m = 0 tenga una raíz solamente a) 3 b) 3/4 c) 1/2 d) 3/2 e) 5/3 09.Sea la ecuación: 021 =++ xx Indicar el valor de verdad de las proposiciones: ( ) Si la ecuación admite solución, ésta debe estar comprendido en [-1; 0] ( ) La ecuación tiene dos soluciones reales ( ) La ecuación tiene una única solución a) VFV b) VFF c) VVF d) VVV e) FVV 10.Resolver: (1 + x) (1 + 2x) (1 + 3x) = - 15 Indicar la suma de las raíces no reales: a) 0 b) 1/2 c) – ½ d) - 1 e) 1/6 11.Sea el polinomio cuadrático: P(x) ≡ (n + 1)! x + n! (x) + (n - 1)!; n ∈ N, indicar verdadero o falso, si P(x) = 0, según corresponda: ( ) P(x) tiene raíces reales y diferentes ∀ n ∈ N ( ) P(x) tiene siempre raíces imaginarias y conjugadas ( ) Para algún n ∈ N, P(x) tiene raíces iguales a) FFV b) FVV c) VFV d) VVV e) FVF 12.Si r y s son raíces de la ecuación cuadrática: mx2 – 2(m – 1) x + m = 0 y cumplen r s s r + =4, halle la suma de todos los valores “m” que satisfacen la condición a) 1 b) - 4 c) - 1 S4AL33B “El nuevo símbolo de una buena educación....” S4AL33B “El nuevo símbolo de una buena educación...."
  • 17. 01 02COLEGIO DE CIENCIAS LORD KELVIN 4to Año Secundaria ALGEBRA 4to Año Secundaria d) 0 e) 4 13.El producto de multiplicar el término independiente con el coeficiente del término cuadrático de la ecuación que tiene por raíces el cuadrado de la inversa de las raíces de ax2 + bx + c = 0, a ≠ 0, es: a) ac b) a2c2 c) a/c d) 1/a2c2 e) c/a 14.Hallar la suma de los cuadrados de las raíces de la ecuación polinomial: F(x) = x3 – 3x + 6 = 0 a) 1 b) - 1 c) 4 d) 8 e) 6 15.Si x1, x2, x3 son las raíces de la ecuación: 4x3 + mx2 - 4x + m2 = 0 Además: cba cba x ++ ++ = 43 1 ; ca cba x + ++ =2 ; c cba x 2 3 ++ = , calcule un valor de “m” a) 0 b) - 1 c) 2 d) - 2 e) 1 OBJETIVOS ESPECIFICOSOBJETIVOS ESPECIFICOS:  Reconocer una ecuación polinomial e indicar la relación existente entre solución y raíz.  Resolver ecuaciones de cualquier grado aplicando los teoremas y técnicas adecuadas. COMENTARIO PREVIOCOMENTARIO PREVIO Al - Guarismi, el año 1 100 estudia ecuaciones del tipo: ax2 +e=bx, ax2 +bx=e , ax2 + bx + c = d; etc y da soluciones para cada caso. La época de oro de las matemáticas Italianas se da en el siglo XVI, con Scipiene del Ferro, Nicola Tartaglia, Girolamo Cardano, Ludovico Ferrari, Frencois Viette, etc, quienes resolvieron las ecuaciones de tercer y cuarto grado. Hecho de trascendental importancia en esa época. La historia da cuenta de que el profesor Scipiene del Ferro logré resolver la ecuación de tercer grado en 1515, pero no la dio a conocer siguiendo las normas científicas de su época. Aún así, confió sus resultados a Antonio Fiore. En 1541 Antonio Fiore se bate en duelo matemático con el profesor Nicola Trataglia para ver quién resuelve la ecuación de tercer grado, saliendo vencedor este último. Cardano quien era médico, adivino y matemático logra con tretas y promesas, que Tartaglia le hiciera conocer la solución de la ecuación de tercer grado. El mismo año Cardano publica su libro “Arte Mayor” en donde da la solución de la ecuación de tercer grado como suya y menciona que Tartaglia no es sino un redescubridor ya que del Ferro había dado la primera prueba hace 30 años. En la misma obra aparece la solución de la ecuación de cuarto grado, debido a Ludovico Ferrari, discípulo de Cardano. Posteriormente se dieron otras pruebas tanto de la ecuación de tercer grado (F. Viette) como de la ecuación de cuarto grado (R- Descartes) Después de los rotundos éxitos de los matemáticos Italianos viene nuevamente un largo periodo de estancamiento en la tarea de la solución de ecuaciones de quinto grado. Recién en 1825, el joven matemático noruego Niels Henrick Abel demostró que la ecuación general de quinto grado no es resoluble mediante la extracción de raíces y las operaciones aritméticas conocidas. Por otro lado en 1929 Evaristo Galois, probaría que las ecuaciones de grado superior a cuatro no son resolubles por radicales y dio las condiciones necesarias y suficientes para que una ecuación de cualquier grado sea resoluble por radicales. Actualmente existen técnicas que permiten resolver ecuaciones de cualquier grado. CONTENIDO TEÓRICO:CONTENIDO TEÓRICO: ECUACION POLONOMIAL EN UNA INCÓGNITA Es aquella ecuación que tiene la siguiente forma general: P(X) = a0 xn + a1xn – 1 + a2 xn – 2 .... + an-1 x + an = 0 Donde: a0; a1; a2;...... : an – 1 ; an : son sus coeficientes Si: a 0 ≠ 0→el grado de la ecuación es“n”(n ∈N) x → es la incógnita RAIZ DE UN POLINOMIO Dado el polinomio P(x). Se denomina raíz o cero del polinomio, al número “a” si y solo si el polinomio P(x) es divisible entre (x - a). El polinomio P(x) tiene una raíz de valor “a” P(x) = (x - a) q(x) Ejemplo: hallar las raíces de: P(x) = x3 – 6x2 + 11x – 6 Resolución Factorizando se tiene: P(x) = (x –1) (x–2)(x – 3) Luego las raíces o ceros de P(x). Son: {1; 2; 3} Observación: S4AL33B “El nuevo símbolo de una buena educación....” S4AL33B “El nuevo símbolo de una buena educación...." ECUACIONES
  • 18. 01 02COLEGIO DE CIENCIAS LORD KELVIN 4to Año Secundaria ALGEBRA 4to Año Secundaria Una manera práctica de hallar las raíces de un polinomio P(x), es formar la ecuación: P(x) = 0. Así: P(x) =(x –1) (x – 2) (x–3)= 0. CS = {1; 2; 3} En este ejemplo las raíces del polinomio P(x) coinciden con las soluciones de la ecuación P(x) = 0, lo cual no ocurrirá siempre. Raíz de Multiplicidad “k”: Dado el polinomio P(x) se denomina raíz de multiplicidad “k” (k ∈ Z+ ) del polinomio P(x). Al número “a”, si y sólo si el polinomio P(x) es divisible entre (x – a)k , pero no es divisible entre (x – a)k+1 , es decir si: P(x) = x4 – x3 – 3x2 + 5x – 2 Factorizando se tiene: P(x) = (x – 1)3 (x + 2) Luego las raíces de P(x) son: {1; 1; 1; –2} y se dice que:  “1” es una raíz de multiplicidad 3 (raíz triple)  “2” es una raíz de multiplicidad 1 (raíz simple) Formemos la ecuación: P(x)=0⇒P(x)=(x– l)3 (x+ 2) = 0 ⇒ (x – 1)3 = 0 ∨ x + 2 = 0 ⇒ x = 1 ∨ x = – 2 Luego: CS {1; –2} Observación: La ecuación antes expuesta tiene 4 raíces y dos elemento en su conjunto solución. Cuando un polinomio tiene raíces múltiples el número de raíces y el número de soluciones no coincide. Ejercicio: En la ecuación polinomial: x3 (x – 2)2 (x2 + 9) (x + 3 3 ) = 0 Señale: a) El número de raíces b) El número de soluciones c) Su conjunto solución TEOREMA FUNDAMENTAL DEL ALGEBRA Toda ecuación polinomial con cualquier tipo de coeficientes numéricos tiene por lo menos una raíz que generalmente es compleja. Corolario: Toda ecuación polinomial de grado n > 1. Tiene exactamente “n” raíces complejas en general. Luego dada la ecuación polinomial: P(x)=a0 xn +a1xn – 1 +.......+an–1x+an = 0: a0 ≠ 0 Se tiene: P(x) = a0(x – x1) (x – x2)...... (x–xn)= 0 Donde: {x1; x2; x3;..........; xn) son raíces de P(x) TEOREMA DE CARDANO – VIETTE Sea la ecuación polinomial: P(x) = a0 xn + a1xn – 1 + a2xn – 2 +...+ an – 1x + an = 0 a0 ≠ 0. Cuyas raíces son: {x1; x2; x3;............; xn} Se cumple las siguientes relaciones • Suma de Raíces: S1 = x1 + x2 + x3 + ............ + xn = – 0 1 a a • Suma de Productos Binarios: S2 =x1x2+x1 x3 + x2 x3 +...... + xn -1 xn = – 0 2 a a • Suma de Productos Ternarios: S3 = x1 x2 x3 + x1 x2 x4 + ...... + xn – 2 xn – 1 xn = – 0 3 a a • Producto de Raíces: Sn = x1 x2 x3 .............. xn – 1 xn =(-1)n 0 n a a Ejemplo: 01.En: 4x4 + 3x3 – 2x2 + 3x – 1 = 0 Calcular: 4 3 S S 02.En: 3x5 + 10x12 - 2x10 - 25x5 + 15 = 0 Calcular: S10 TEOREMAS SOBRE LA ECUACIÓN POLINOMIAL 1. Toda ecuación polinomial de coeficientes racionales y de grado n ≥ 2. Que tenga una raíz de la forma: “a + b ”, donde: a y b ∈ Q (b > 0) ∧ b ∈ I ; tendrá como raíz necesariamente al número (a – b ). 2. Toda ecuación polinomial de coeficientes racionales y de grado n ≥ 4: que tenga una raíz de la forma ba + , donde: a y b ∈ Q+ ∧ Iab,b,a ∈ . Tendrá como raíces necesariamente a los números: ba:ba:ba −−+−− 3. Toda ecuación polinomial de coeficientes reales y de grado n ≥ 2 que tenga una raíz compleja de la forma, a + bi. Donde a y b ∈ R (b ≠ 0). Tendrá necesariamente como raíz al complejo conjugado de dicha raíz es decir otra raíz será: a - bi Observación: Q: conjunto de los números racionales I: conjuntos de los números irracionales Ejemplos:  En la siguiente ecuación: 07 23 )( =++−= baxxxP x . a, b ∈ Q Hallar (a + b) si su raíz es: 3 + 5  Formar la ecuación de menor grado posible sabiendo que una raíz es 35 + y además sus coeficientes son racionales.  Dadas la ecuación: x3 + x2 + mx + n = 0. m, n ∈ R Donde: 1 + 7 i es una de las raíces. Hallar La suma de coeficientes de la ecuación. TRANSFORMACIONES DE ECUACIONES Sea la ecuación polinomial: :01 2 2 1 1 =+++++ − −− nn nnn o aaxaxaxaxa  Con raíces: {x1; x2; x3;................; xn} entonces: 1. La ecuación de raíces aumentados o disminuidos en un valor “k”, es decir con raíces: { }kxkxkxkx n ±±±± ;;;; 321  es: ( ) ( ) ( )1 1 1 =++++ − − nn nn o akxakxakxa  Ejemplos:  Halle la ecuación cuyas raíces son las de la ecuación: x2 – 2x – 8 = 0, pero aumentadas en 1. La ecuación es: (x – 1)2 – 2(x – 1) – 8 = O S4AL33B “El nuevo símbolo de una buena educación....” S4AL33B “El nuevo símbolo de una buena educación...."
  • 19. 01 02COLEGIO DE CIENCIAS LORD KELVIN 4to Año Secundaria ALGEBRA 4to Año Secundaria  Encuentre la ecuación cuyas raíces son los de la ecuación x3 – 2x2 + x – 5 = 0 disminuidas en 2. La ecuación es: (x + 2 )3 - 2(x + 2)2 + (x + 2) - 5 = 0. Efectuando se obtiene: x3 + 4x2 + 5x – 3 = 0. También se puede usar el siguiente método: x = 2 1 - 2 1 - 5 ↓ 2 0 2 x = 2 1 0 1 - 3 ↓ 2 4 x = 2 1 2 5 ↓ 2 1 4 Luego la ecuación es: 03x5x4x 23 =−++ • Encontrar la ecuación cuyas raíces son las de la ecuación: x5 – 3x3 + 2x2 + 1 = 0, disminuidas en 1. 2. La ecuación de raíces multiplicadas por un valor “k” (k ≠ 0); es decir con raíces: 0a k x a k x a k x a n1n 1n 1 n o =+      ++      +      − −  O también: 0kaxkaxkaxa n n 2n2 2 1n1 1 n o =++++ −−  Ejemplos: • Encuentre la ecuación, cuyas raíces son las de la ecuación: x2 – x – 6 = 0. Multiplicadas por 2 La ecuación es: x2 - 21 x - 22 . 6 = 0 024x2x 2 =−− • Halle la ecuación cuyas raíces son las de la ecuación: x3 + 2x2 - 5 x 6 = 0 multiplicadas por 3. La ecuación es: x3 +2 . 31 x2 + 5 . 32 x - 6 . 33 = 0 x3 + 6x2 - 45x - 162 =O 3. La ecuación de raíces invertidas es decir con raíces:       nXxxx 1 ;; 1 ; 1 ; 1 321  Es: 0axaxa o 1n 1n n n =+++ − −  Ejemplo: Dada la ecuación: x3 - 5x2 + 7x + 2 = 0. De raíces {a; b; c} entonces la ecuación cuyas raíces son:       cba 1 ; 1 ; 1 es 2x3 + 7x2 - 5x + 1 = 0 TEOREMA DE BOLZANO Dada la ecuación polinomial F(x) = 0. Donde F(x) es una función continua definida en [a; b] Si F(a). F (b) < 0. Entonces existe al menos una solución real: x0 ∈ < a; b > / 0)( 0 =xF F(b) F(a) F b a x 0 x y PRÁCTICA DE CLASEPRÁCTICA DE CLASE 01. Sean: x1; x2; x3 raíces de la ecuación: 2x3 – x + 5 = 0 Calcular: 321 1 3 1 xxx 3x 1x + − + a) 1 b) 2 c) -2 d) -3/2 e) 4/3 02.Sean: a, b, y c raíces de la ecuación: x3 – 4x2 + 2x + 4 = 0 Calcular: ab c ac b bc a 222 ++ a) 5 b) - 5 c) - 4 d) - 7 e) 2 03. En la ecuación: x3 - 63x + α = 0. Determinar un valor de α para que una de las raíces sea el doble de otra. a) 162 b) 180 c) 400 d) 800 e) N.A. 04.En la ecuación polinomial: P(x) = x3 + (m + 2) x2 + (m2 – 3) x + m2 + 2 = 0 De raíces x1 ; x2 ; x3. Calcular el valor de “m” de tal manera que la expresión: A = 2 3 2 2 2 1 xxx ++ tenga el máximo valor. a) l b) 2 c) 3 d) 4 e) 5 05.Hallar la relación que debe existir entre los coeficientes de la ecuación: ax3 + bx2 + cx + d = 0: a ≠ 0 . Si una de sus raíces es el negativo de la otra a) ab = cd b) ac = bd c)ad=bc d) a + b = c + d e) a+d=b+c 06.Sabiendo que: x = c es una raíz de la ecuación: ax5 + (b-ac)x4 - bcx3 - bx2 -(a-bc)x + ac = 0: (a>0) ¿Qué condición deben cumplir a; b y c para que las otras raíces sean reales? a) |b| ≥ a b) |b| ≤ a c) |b| ≥ 2a d) |b| ≤ 2a e) 2c=a+ b 07.Indicar el menor valor que debe tener el grado del polinomio P(x). Con coeficientes reales, tal que: (2 + 3 ) Sea una raíz simple, (3 + 2i) sea una raíz de multiplicidad 2 y ( 3 + 2 ) sea una raíz triple. a) 5 b) 6 c) 7 d) 8 e) 9 08.Hallar un polinomio mónico P(x) con coeficientes enteros y de menor grado posible una de cuyas raíces sea: 3 32 + . Indicar la suma de los coeficientes de este polinomio. a) 34 b) 24 e) - 24 d) 62 e) - 34 EJERCICIOS PROPUESTOS Nº 05 01.Hallar el valor de “k” si las raíces de la ecuación: x3 - 9x2 + kx - 24 = 0 Están en progresión aritmética. a) 12 b) 13 c) 24 d) 26 e) 28 02. Sea el polinomio: F(x) = x3 + 3x2 – 9 S4AL33B “El nuevo símbolo de una buena educación....” S4AL33B “El nuevo símbolo de una buena educación...."
  • 20. 01 02COLEGIO DE CIENCIAS LORD KELVIN 4to Año Secundaria ALGEBRA 4to Año Secundaria Además: F(m) = F(n) = F(p) = 0 Calcular:       ++ mn p mp n np m F a) - 5 b) - 1 c) 2 d) - 2 e) 4 03.Si: (2 + i) es una raíz de multiplicidad dos del siguiente polinomio: P(x) = x5 + ax4 + bx3 + cx2 + dx + 25 Hallar: a + b + c + d. Además: a ; b ; c ; d ∈ R. a) 17 b) 18 c) 19 d) -18 e) -17 04. La ecuación: x4 – 12x – 5 = 0. Contiene 2 raíces cuya suma es 2. Calcular la suma de las inversas de las otras dos. a) 0,2 b) 0,4 c) - 0,2 d) - 0,4 e) 5 05. Sea la ecuación polinomial: P(x) = ax3 + x2 + x + b = 0: a ≠ 0 Determinar los valores de “a” de modo que P(x) admita una raíz real “r” de multiplicidad 2. a) { }4 3 1 ; −−∞−∈α b) 3 1 ; −∞−∈α c) 3 1 ;∞−∈α d) { }0 3 1 ; −∞−∈α e) α ∈ R 06.Si la ecuación: x4 + mx3 + 2x + n = 0 m ∧ n ∈ R; admite una raíz triple. Hallar: m2 + n3 a) 3 b) 4 c) 5 d) - 3 e) -1 07.Se sabe que: x1 ; x2 y x3 son las raíces de la ecuación. x3 – x2 – 1 = 0. Encontrar una nueva ecuación cuyas raíces son: x1 + x2 ; x2 + x3 ; x3 + x1 a) 01yy2y 23 =−+− b) 01yy2y 23 =++− c) 01yyy 23 =−−− d) 01yy2y 23 =+−− e) 01yy2y 23 =−+− 08. ¿Cuál será la ecuación cúbica cuyas raíces sean el doble de los recíprocos de cada una de las raíces de la ecuación polinomial? Ax3 – Bx + C = 0 ; C ≠ 0 a) Cx3 - Bx + A = 0 b) Cx3 + 2Bx2 + 4A = 0 c) Cx3 + 2Bx2 – 4A = 0 d) Cx3 – 2Bx2 + 8A = 0 e) Ax3 – 2Bx + 4C =O 09.El producto de los coeficientes de la función polinomial de menor grado que pasa por los puntos: (0; 0); (1; 1) ; (2; 0) y (3; -1) es: a) -15/4 b) -14/9 c) 5/9 d) -15/9 e) -16/9 10. Sabiendo que: a b y c son raíces de la ecuación: x3 - 7x2 + 5x + 6 = 0 Calcular: M = (a+b-c)-1 +(b+c- a)-1 + (c + a - b)-1 a) 31/55 b) 9/55 c) 7/155 d) 29/155 e) 27/55 11.Si la ecuación: x5 - 10a3 x2 + b4 x + c5 = 0 tiene 3 raíces iguales. Hallar el valor de: ab4 - 9a5 a) c b) - c5 c) 0 d) c2 e) 1 12.Encontrar un polinomio mónico en "x" de coeficientes en Z que acepte a 33 32 − como raíz. Hallar la suma de coeficientes de dicho polinomio. a) 165 b) 168 c) 170 d) 174 e) 162 13.Formar la ecuación de menor grado posible con coeficientes racionales, en la que una de sus raíces sea. 2i3 + a) x4 – 2x2 + 25 = 0 d) x4 +2x2 -25=0 b) x4 + 2x2 + 25 = 0 e) x4 +x2 +25=0 c) x4 + 2x2 + 5 = 0 14.Calcular la suma de las raíces de: x3 +2x2 =x – 1 a) 2 b) –2 c) 3 d) – 1 e) 1 15.Calcular el producto de las raíces de: 2x3 + 6x2 = 5x + 8 a) –1 b) –2 c) 4 d) – 4 e) –6 16.Resolver: x3 + 2x2 – 11x = 12. E indicar una de sus raíces. a) 1 b) 2 c) 3 d) 4 e) 6 TAREA DOMICILIARIATAREA DOMICILIARIA 01.Si: ( )23)( 1 1 − = x F x y además a, b y c son raíces de la ecuación: x3 - 3x - 1= 0. Calcular S = F(a) + F(b) + F(c) a) 1 b) 3 c) 1/3 d) 9 e) N.A. 02. Halle las raíces r1 , r2 , r3 , r4 de la ecuación: 4x4 - ax3 + bx2 - cx + 5 = 0 a) 1/2 b) 1/4 c) 5/4 d) 1 e) N.a. 03. Halle las raíces r1 , r2 , r3 , r4 de la ecuación: 4x4 - ax3 + bx2 - cx + 5 = 0 Sabiendo que son reales positivos y que: 1 8 r 5 r 4 r 2 r 4321 =+++ Indique el valor de: r4 a) 1/2 b) 1/4 c) 5/4 d) 1 e) 2 04.Determinar el polinomio P(x) de grado 7. Sabiendo que: I)Para: x = 3: P(x) =PI (x) = PII (x) = PIII (x) = 0 y PIV (x) ≠ 0 II) Para: x = - 2 : P(x) = 0 : PI (x) ≠ 0 III) Para: x = 4 : P(x) = 0 : PI (x) =0 : PII (x) ≠ 0 IV) P(2) = - 32 Dar como respuesta el valor de P(5) a) - 1 12 b) 224 e) 32 d) - 32 e) - 224 05.Sean a . b y c raíces de la ecuación: x3 + px + q = 0 (a, b, c diferentes) expresar en términos de p y q a: S4AL33B “El nuevo símbolo de una buena educación....” S4AL33B “El nuevo símbolo de una buena educación...."
  • 21. 01 02COLEGIO DE CIENCIAS LORD KELVIN 4to Año Secundaria ALGEBRA 4to Año Secundaria M=(a -b)2 (b - c)2 (a - c)2 a) 23 q27p4 + b) 23 q27p4 −− c) 43 q2p + d) 23 q9p +− e) 23 q27p4 − 06. Sobre la ecuación: P(x) = x5 + ax4 + bx3 + cx2 + dx + c = 0 Donde: 2a2 < 3b ∧ {a; b; c; d; e}⊂ R Indicar verdadero (V) o falso (F) I) Todas sus raíces son reales II) Al menos dos raíces son complejas III) Una raíz es real a) VFF b) FFV c) FVF d) FFF e) VVV 07. Si: P(x) = (x - 1)(x - 3)(x - 5)+(x - 2)(x - 4) Indicar la alternativa más correcta: a) Tiene 3 raíces reales b) Tiene 3 raíces reales negativas c) Tiene 3 raíces reales positivas d) Tiene 2 raíces reales positivas y una es negativa e) N.A 08. Sea el polinomio: P(x) = x3 - 3x2 + 5 Indicar si es verdadero o falso: I. Sólo tiene una raíz real positiva II. Tiene 2 raíces complejas III. Tiene una raíz comprendida entre <-2; - 1> IV. Tiene un mínimo absoluto en x= 2 a) VVVF b) VFVF c) VFFF d) FVVF e) FFFV 09. Dada la función: P(x) = x4 + 2x3 + x2 - 8 Decir verdadero (V) o falso (F) en: I) Tiene un mínimo relativo en x = -1 II) Tiene 2 raíces reales III) Su menor raíz está ubicada en <-3;2> a) VVF b) VFV c) VFF d) VVV e) FFV 10. La única raíz real de: x5 + x - 10 = 0 se encuentran en: a) < 3/2; 7/4 > b) < 7/4; 2 > c) <1;2> d) <5/4; 3/2> e) < 1; 5/4 > SOLUCIONARIO Nº Ejercicios Propuestos 01 02 03 04 05 01. C C C C D 02. C C B A A 03. B D E B D 04. D D B A D 05. A D E C D 06. A C A A A 07. E C B D B 08. A B D B D 09. D B B A E 10. A B B A D 11. C C E B 12. B A E E 13. D E A 14. D E B 15. A C C 16. C B C 17. A D 18. D B 19. B C 20. E D GRUPO EDUCATIVO INTEGRAL copyright 2003 S4AL33B “El nuevo símbolo de una buena educación....” S4AL33B “El nuevo símbolo de una buena educación...."